SMLE April & May 2023

You might also like

Download as docx, pdf, or txt
Download as docx, pdf, or txt
You are on page 1of 713

SMLE

T10 By NaiF .
⬇️
https://t.me/saudismle
April & May-2023
(Daily Updated )
‫ مشرفين القنوات ا لي تنقل لنا التجميعات والتجارب في التيليقرام ا ل تنسو نا من دعواتكم الصالحة‬+ ‫فضاًل منكم ال تنسوني أنا والفريق‬

Exam Period:
From 26-April-2023 To 25-May-2023
File’s Link⬛J
https://docs.google.com/document/d/14dNr8QA3u2c_x06cWZWEXL3-wsQJOpxNZe9gwDli9XM/edit

◆ Last updated:
May 29 - 2023
( 3:05 am ).
⬛✓Ref.
Amboss
&
✓⬛Ref. Uptodate
&
⬛✓Al Smith App: we use it to help
us and consider it as a comment just.
◆ Note: Most of the images aren’t included in the numbering of
the questions here.
‫ال تنسوني أنا والفريق ومشرفين القنوات بالتيليقرام من دعواتكم الصالحة‬
App⬇️ Telegram via ‫وبذات األخ أنس الزهراني هل دور كبير جد ًا في إنشاء الملف‬
contact Edite: & Notes For @SMLE_Team10

This file done by some correctors:


1- Naif Al-Osaimi.
2- Anas Al-Zahrani.
3- Dr. Hasna.
4- Dr. Ahmed
AlMudhahka. 5- SM.
6- Alaa.
7- Salam.
8-Bader.
9- mohammed Al-thobaiti.
10- Noor.
11- Abdullrazaq Al
Shammary. 12- Eman.
13- suhaib
14-

◆ÇMCQ’s resources are Telegram Channels:


1- Fair
2- Silent
3- Bdran’s space
4- SMLE-B
5- ASMA
6- Paediatric SMLE
Qs 7- Psycho Smle
8- SMLE Brain Ethics
9- OB/ Gyn new Channel
10- Surger By Hamoud High Yield
11. Earth 🌍

📍‫️⬇ مهم‬
‫ قبل ما نعلن بالقناة ا نا صححنا الملف بشكل كامل وممكن بعض الجابات خطأ وتحسب علينا فيه‬10T ‫ناس تح مّ ل ملفات‬
‫طريقتنا هي ️⬇‬
‫مع الجابات والشرح ”تجميعات الفترة فقط“ نف ّرغ كل لاا سئلة الموجودة بقنوات التيليقرام المنتشرة بنفس الفترة الموجود بالتعليقات‪ ، ،‬كل كم يوم‬
‫وبعدها الفريق يبدا يصحح كل اسبوع تقريباً واحيانا كل يومين على حسب فراغ الفريق ‪ ،‬والسؤال المصحح يكون موجود عليه اسم المصحح‪,‬وا لي دب ون اسم معنا هت باقي ما مر عليه الفريق أو تعذّر تصحيحه ‪ ،‬أتمنى تفهمون ‪ .‬هالشي‬

‫——————————————————‬

‫‪Starting‬‬
‫◆ جاري التصحيح‬
‫واذا فيه تعديل وت اصلو معي مباشرة على الخاص في تيليقرام ️⬇‬
‫‪@SMLE_Team10‬‬

‫?‪1- diagnostic test for flank mass‬‬


‫‪CT‬‬

‫‪2- Patient presented with bleeding from his ear‬‬


‫‪A- mastoid bone‬‬
‫‪B- basal skull fracture‬‬

‫‪Similar:‬‬
‫‪RTA patient has bleeding from external auditory meatus. Where is the site of the‬‬
‫?‪fracture‬‬
‫‪A. Base of skull‬‬
‫‪B. Occipital fracture‬‬
‫‪C. Mandibular fracture‬‬

‫‪Similar recall:‬‬
‫‪1- https://t.me/c/1752626805/4323‬‬
‫‪2- https://t.me/c/1632086392/882‬‬
‫‪3- https://t.me/smlepaediatrics/40‬‬
‫‪4- https://t.me/c/1625198149/225‬‬
‫‪5- https://t.me/silent_2023m/2062‬‬
‫‪6- https://t.me/BdranSpace/180‬‬
‫‪7- https://t.me/smle22olds/3609‬‬
‫‪8- https://t.me/c/1222499606/226‬‬
‫‪9- https://t.me/QuickRecall/488‬‬
‫وسط الرابط ع دّ ة رسايل م تاليه تابعه لنفس السؤال وقناة حلوة لجراحة ‪-https://t.me/SMLE_Surgery/588 10‬‬

‫صار عليه كتير نقاش‬


‫نزف من الذن وتمزق غشاء الطبل ‪ Mastoid‬تكدم غشاء الطبل ونز ساىل شوكي من الذن ‪B‬‬

‫‪asal‬‬
Similar: 26 Apr

Patient presented with bleeding from his ear -


> A- mastoid bone
B- basal skull fracture

‫كذا هب العبارات ما قن در نحدد‬

‫ فيه الزم‬differential signs

‫السؤالين متشابهين وبتجميع نوفمبر احس الجواب غلط‬


‫؟ احد عنده فت سير‬
‫سبق وبحثت ‪ ..‬ما فيه إجابة واضحة‬

‫صح ‪ skull of base‬اذكر حمزة اظن سأل استشاري وقاله‬


Basilar of skull fracture
⬛J
‫اللي أعرفه ااذ كان فيه‬
bleeding
Subcutaneous hematomas: typically appear within hours to days of injury
◆ Raccoon eyes: ( bilateral periorbital ecchymosis ) subcutaneous hematoma around
the eyes
◆ Battle sign: subcutaneous hematoma overlying the mastoid process .
Mostly ⬛~Answer is *Mastoid bone fracture * ⬛✓

……………

‫بختصار متى قن ول‬


Mastoid bone fracture
‫ومتى قن ول‬
basal skull fracture

‫ترا متشابهه‬
‫ويوجد فوارق في ال‬
clinical signs

‫ومتى قن ول‬
Temporal bone fracture

————————————————-

3- Patient presented with few symptoms and worst headache of their life:

subarachnoid hemorrhage
4- Patient presented with symptoms of anemia ..
labs MCV : 68
Red blood cell’s count: 6
What’s the Diagnosis ?

A- Iron deficiency anemia


B- Thalassemia

Based on the MCV and RBC count values provided, the likely diagnosis for this patient
is Thalassemia

Thalassemia is a genetic disorder that affects the production of hemoglobin and can
lead to low red blood cell counts, which is reflected in the lab results of low MCV and
normal or high RBC count. Iron deficiency anemia, on the other hand, typically presents
with low RBC count and microcytic hypochromic anemia, which is reflected in lower
MCV values. However, further diagnostic tests may be needed to confirm or rule out the
diagnosis of Thalassemia and to determine the appropriate treatment plan.
A complete blood count (CBC) with differential and hemoglobin electrophoresis are
tests that can aid in the diagnosis of Thalassemia and further guide the management of
the patient's anemia.
5- 20s man is referred to a psychiatrist after receiving poor evaluation at work. He is an
im resident currently complaining of training. Several medical students and interns
have complained about his behavior and accused him of being unnecessarily hostile
towards them. Recently he yelled at his intern in front of several colleagues after his
supervising attending expressed frustration about how large their inpatient census
had
become. During discussion the patient admits to feeling very stressed at work and
mentions several instances of being verbally abused by his current attending physician
for the quality of his teams, He says that he is very angry at his current attending and
feels that it is very difficult to work with him. His behaviour would be best described as
which of the following?

a. displacement
b. projection
c. sublimation
d. splitting
6- 20 y old male, keeps counting blocks, walls, etc.
he keeps repeat counting, he feels guilty and anxiety when he stops counting, what is
dx?

A. Compulsion
B. Obsession
C. Depression

Compulsions are a type of behavior that an individual must do repetitively to reduce feelings of anxiety
and panic. As per the description, the individual is repeatedly counting blocks, walls, etc. and
experiences guilt and anxiety when they stop counting or cannot continue counting.

Obsessions, on the other hand, refer to repetitive thoughts, images, or impulses that are often
intrusive and unwanted and result in significant anxiety or distress. Depression is a mood
disorder characterized by feelings of sadness, helplessness, hopelessness, and loss of
interest in activities they once enjoyed.

Therefore, the best diagnosis in this case appears to be compulsion.


7- Patient is a heavy smoker and he is known to have epilepsy which is well
controlled on carbamazepine. the patient is counseled for smoking cessation. Which
of the following is contraindicated in this patient?

A. Bupropion
B. Varenicline
C. Nicotine replacement therapy The
correct answer is A. Bupropion.
Bupropion is a medication that is used for smoking cessation and depression. However, it can
lower the seizure threshold and, therefore, is contraindicated in individuals with a history of
seizures like in the presented case.

Nicotine replacement therapy and varenicline are still treatment options for smoking cessation in this
patient. Nicotine replacement therapy delivers a lower dose of nicotine than smoking and is often used
to reduce nicotine cravings that lead to smoking. Varenicline is a medication
that is used as a partial nicotine receptor agonist, which reduces the pleasure from smoking and
reduces withdrawal symptoms when smoking is stopped.

Nevertheless, appropriate counseling, support, and motivation are important aspects of smoking
cessation and should also be implemented for effective treatment.

8- Case of Young man w/ a hx of ibd is brought to the ed after changes in his


behaviour, his friend reported that the patient has not been sleeping, is acting irritable
and depressed, additionally, he has a rash on the dorsal surface of his hands, face and
neck. The patient appears apathetic and has episodes of diarrhea, vitamin deficiencies
cause his symptoms?

a Niacin /vit B3✓⬛⬛


‫ الشرح تحت‬b
thiamine /vitb1
c vitamin C
d vitamin B12 ( same symptoms except rash)
- Ibd= inflammatory bowle diseases
The appropriate answer is A .
.

Niacin or Vitamin B3 deficiency (Answer A) can cause a range of symptoms including


dermatitis, diarrhea, dementia, and depression
—————————————————-
Comment of Dr. Abrar:
Regarding question 8 in may questions bank.
Case of Young man w/ a hx of ibd is brought to the ed after changes in his behaviour,
his friend reported that the patient has not been sleeping, is acting irritable and
depressed, additionally, he has a rash on the dorsal surface of his hands, face and
neck. The patient appears apathetic and has episodes of diarrhea, vitamin deficiencies
cause his symptoms?

a Niacin /vit
B3⬛✓ b
thiamine /vitb1 c
vitamin C
d vitamin B12
Was answered b12.

◆ I would argue that, Niacin is the answer, since the area of rash they mention is specific for
pellagra.
Along with the diarrhoea, dementia.

◆ And for B12 deficiency the


Common neurologic manifestations include paresthesias, weakness, gait abnormalities, and cognitive
or behavioral changes.

◆ Pellagra skin lesion support the diagnosis for it.


- Malnutrition, specifically IBD can cause either B12 or B3 deficiency.
Both can cause neurological disturbances and behaviour changes.
But the location of rash, dorsal of hand and neck is specific for Niacin deficiency. Thats why I would
go with B3z

Niacin deficiency: 4Ds


Diarrhoea, dermatitis , dementia, Death.

I saw a similar questions while preparing for usmle step 1. I will send it to you.
The rash in the neck and dorsum of the hand is quite a giveaway for the questions.
.‫ ممكن تحط الخيارين انهم ممكن يكونو اجابه‬،‫هو بس اج هت اد‬

Naif: answer is A- Niacin ( Vitamin B3) ✅


Another recall: ⬇️
https://t.me/saudismle/501
9- 17yo girl with refusal to maintain a weight within a normal range for her height and
age, fear of weight gain, severe body-image disturbance and amenorrhea for >3 cycles,
what is the diagnosis?

a. anorexia nervosa
b. bulimia
c. body dysmorphic disorder
d. obsessive compulsive disorder
10- Bacterial vaginosis case and ask for treatment?

A. Oral antifungal
B. Topical antifungal

metronidazole vaginal gel, metronidazole pills, and clindamycin vaginal cream. Since around 70% of
BV cases return within a year of initial treatment

11- Uterine fibroid most common site?

Malaz:
Intramural

Fathyia Elian:
Sub mucosal?

12-

13-

Ask about time ¶


14- Uterine fibroid most common site?
The most common site for uterine fibroids is the myometrium, which is the muscular layer of the
uterus. More specifically, fibroids can develop anywhere within the uterine wall, but they usually
occur in the body of the uterus or at the uterine fundus near the top of the uterus.
Fibroids can also develop within the uterine cavity or on the cervix, but these types of fibroids are less
common.

15- A case of tension pneumothorax, what’s the next step in management:

A- thoracotomy
B- needle decompression
⬛✓ C- chest tube

Naif: ⬛✓needle decompression followed by Chest tube

Ref.Amboss

16- Patient presented with a breast mass (not sure of the size) , imaging
showed: cystosarcoma phylloid.
What’s the management ?

A- chemotherapy
B- Radiotherapy
C- Mastectomy
D- wide excision

(I got this question twice on the exam but different scenarios)

▼According size

-If more than 10 C ✓


-Less than 10 D ✓

The management of a cystosarcoma phyllodes breast tumor (also known as phyllodes tumor) depends
on the size and characteristics of the tumor.
For smaller tumors that are benign in nature or considered borderline, the treatment of choice
is surgical excision with clear margins (Option D).

For larger tumors or those with malignant features, more aggressive management may be required.
In cases where the tumor is malignant or there is a significant risk for recurrence, either a wide
local excision or a mastectomy (Option C) may be required depending on the size and location of
the tumor.

Therefore, the most appropriate management option for a cystosarcoma phylloid tumor is D.
.

17- 35 years old female, came for a regular follow up , her mom has a history of breast
cancer and her dad had colon cancer , which screening should she do at this moment?

A- colonoscopy
B- mammogram
C- fecal blood occult

Based on the patient's family history of breast and colon cancer, the most appropriate
screening option for her at this time would be a mammogram (Option B).

The American Cancer Society recommends that women with a first-degree relative (mother, sister, or
daughter) who has been diagnosed with breast cancer should consider scheduling a mammogram and
breast MRI 5-10 years before the age of the relative's diagnosis. In this case, since the patient's mother
has breast cancer, she should start receiving regular mammograms at an earlier age than the general
population.

While colonoscopy (Option A) and fecal occult blood test (Option C) are important screening tests for
colon cancer, the patient's young age and family history of breast cancer make mammogram a more
appropriate option at this time.

18- A 50 years old male not complaining of anything , screening modality?

A- sigmoidoscopy annually B-
colonoscopy every 5 years
C- fecal blood occult every year

According to the American Cancer Society, men at an average risk of developing colon cancer should
start screening at age 45-50. Colonoscopy is the most effective screening method as it can detect and
remove polyps before they become cancerous. The frequency of follow-up colonoscopies will depend
on factors such as the size of any polyps detected and the patient’s individual risk factors, but every 5-
10 years is generally recommended.
-

19- Long case of man w/ a week of progressive abdominal discomfort, abdominal


distention, nausea, episodes of vomiting, 6 weeks ago he was hospitalized for an
episode of acute pancreatitis. He used to drink heavily before the hospitalization but
he has stoped since then. BP 117/84, epigastric fullness and tenderness. normal cbc,
similar to this pic of k⎝CT abdomen is shown. What Is the most appropriate next
step?

a. Expectant management b.
endoscopic drainage
c. parenteral antibiotics
d. cholecystectomy

20-

man is brought to the ER unconscious following a motorcycle collision, on arrival his


vital signs show BP of 160/100, pulse 58, respiratory 10 and irregular. He does not
answer questions or open his eyes to voice or touch, but he withdraws his extremities
to pain, his pupils are symmetric, sluggish, bilateral breath sounds are normal and has
abrasions on the face and extremities, he had endotracheal intubation for airway
protection. His imaging studies of the head and cervical spine shows no abnormalities/
hematoma but diffuse cerebral edema is present, which of the following is the most
appropriate medications to this stage?

a hypertonic saline b
cyclosporins
c nimodipine
d glucocorticoids
21- 30y M, has hx of persistent abnormal liver function tests for which no cause has
been found, admitted for liver biopsy performed on his bedside, 30 minutes later, when
he is found collapsed, examination shows pulse rate of 120 bpm, unrecordable blood
pressure and his abdomen is distended, if the decision is to perform exploratory
laparotomy, when will be the most appropriate time for it?

a. within 1 week
b. within 4 hours
c. immediately
d. next day

, the most appropriate time for exploratory laparotomy would be “. c. immediately “

The patient had abnormal liver function tests that were unexplained and underwent a bedside
liver biopsy. Shortly afterward, he became hemodynamically unstable with tachycardia,
hypotension, and abdominal distension. These signs suggest that the patient may have developed a
significant bleed into the abdominal cavity (hemoperitoneum) as a complication of the liver biopsy.

An immediate exploratory laparotomy is necessary in this case to identify the source of bleeding and
control it. The longer the delay in performing the surgery, the higher the risk of further blood loss,
hemodynamic instability, and potential shock. Therefore, the most appropriate time for exploratory
laparotomy in this case is immediately.

Options A (within 1 week) and D (next day) are not appropriate in this case due to the
likelihood of ongoing bleeding and the potential for a rapid deterioration in the patient's
condition. Option B (within 4 hours) may be considered if the patient is stable, but immediate
intervention is warranted in this scenario given the patient's unstable condition.

22- Case of 75yrs man on PE, dre feels a firm solitary nodule in the prostate.
biopsy shows small closely packed glands that contain cells with prominent
nucleoli, diagnosis?

a, urotherlial carcinoma
b,
adenocarcinoma⬛✓
⬛ c, chronic prostatitis
d, prostatic hyperplasia

Naif: B is correct thats my answer


Ref. Amboss
⬛J
Based on the given information, the most likely diagnosis is b. Adenocarcinoma.

The patient is a 75-year-old man who had a prostate examination that revealed a firm solitary nodule.
The biopsy results show small closely packed glands containing cells with prominent nucleoli. These
findings are typical of adenocarcinoma of the prostate gland.
Urothelial carcinoma (a) is a type of cancer that arises from the cells lining the bladder or the upper
urinary tract. It is not typically associated with prostate nodules.

Chronic prostatitis (c) is an inflammation of the prostate gland that can cause urinary symptoms such
as difficulty urinating, pelvic pain, and painful ejaculation. It is not typically associated with firm
nodules in the prostate.

Prostatic hyperplasia (d) is a non-cancerous enlargement of the prostate gland that is common
in older men. While it can cause urinary symptoms, it typically does not present as a firm
solitary nodule.

Therefore, the most likely diagnosis in this case is adenocarcinoma of the prostate gland, which is
the most common type of prostate cancer.

23- 15Y/O boy comes with 2 days of fever and nasal discharge, malaise, fatigue and
myalgia,. temperature is 38.8 bp is 122/74, incidental urine dipstick testing shows 2+
proteinuria but no hematuria, pyuria or active urine sediment serum Creatinine is
with in normal limits. Which of the following is the most appropriate next step in
management?

a_ repeat dipstick testing on two subsequent occasions. b-


order 24-hour urinary collection for protein.
c_ reassure, with no further workup. d_
check something I forgot

On dipstick testing is to repeat the dipstick testing on two subsequent occasions (Option A).

Proteinuria can indicate the presence of an underlying renal disease. However, fever,
infections, and dehydration can also cause transient proteinuria. Therefore, initial findings of
proteinuria in an acute illness should be confirmed on two subsequent occasions to exclude the
diagnosis of persistent proteinuria.

Option B (24-hour urinary collection for protein) is typically reserved for confirmation of persistent
proteinuria or to assess the amount of protein lost in the urine.

Option C (reassurance with no further workup) would not be appropriate in this scenario, as there is
a possible underlying renal disease that needs to be investigated.

Therefore, the most appropriate next step in this case is to repeat the dipstick testing on two
subsequent occasions to confirm persistent proteinuria and determine the need for further evaluation.
In the meantime, the patient should be managed symptomatically for the acute
illness.
24- A developmental normal on the examination table is sitting without support and
babbling to herself, she picks up a cheerio with a raking motion and moves it from
her right hand to her left hand, how old is she?

a. 6months.
b. 4 months.
c. 12 months
d. 9 months.

child is most likely 6 months (Option A).

By 6 months of age, a child should be able to sit without support and babble to herself. Also, at this age,
a child's hand-eye coordination continues to develop, and she should be able to pick up objects with a
raking motion and move them between her hands.

At 4 months of age (Option B), a child may be able to briefly hold a sitting position and grasp objects
with both hands, but she is unlikely to sit without support for an extended period or have the ability to
move objects from one hand to another.

At 12 months of age (Option C), a child would typically have more advanced motor skills, such as
standing and walking with assistance, and would be able to use a pincer grasp to pick up small objects.

At 9 months of age (Option D), a child would be somewhere in between the 6-month and 12- month
milestones, with developing but not yet fully mature motor skills.

25- The case of a asymptomatic woman asks about management of hypothyroidism


during pregnancy, the patient is not pregnant but wishes to become pregnant soon, she
has a hx of hypothyroidism for several years and has been on a stable dose of
levothyroxine, TSH checked 3 months ago was 2.0 uU/ml. , which of the following is the
most appropriate recommendation for this patient?

a. Increase the levothyroxine dose when the patient becomes pregnant.


b. continue the current levothyroxine dose through pregnancy
c. decrease the levothyroxine dose when the patient becomes pregnant
d. increase the levothyroxine dose prior to pregnancy.
26- Patient with Kawasaki
One of the following is from the 5 symptoms of disease:

1_arthritis
2_Myocarditis
3_anterior uvitis
4_bilateral bullged conjunctiva injection without exaudate// ‫الخيار كان مت اما هيك‬//
27- A patient with dilated heart failure developed severe enteritis, diarrhea,
and vomiting a month ago and lasted 3 days, conscious and oriented, but
drowzy

Medicine:

Acei/ferosmide/BB
Lab
NA normal
K heigh
Cr heigh
Na urine low
Osmolality urine heigh

Management?

1_carful intravenous normal salin


2_increase ferosmide
3_spironolacton

Given the patient's history of dilated heart failure, severe enteritis, and electrolyte abnormalities (high
K+ and high serum creatinine), the most appropriate management option from the ones provided
would be to carefully administer intravenous normal saline, therefore the correct answer is 1.

The patient is likely experiencing acute kidney injury (AKI) due to dehydration and volume depletion
from the enteritis, which in turn has exacerbated the underlying heart failure. The high serum potassium
levels indicate that the kidneys are unable to excrete potassium effectively, while the low urinary
sodium and high urine osmolality suggest that the kidneys are conserving sodium and water in response
to hypovolemia.

In this situation, careful administration of intravenous normal saline is indicated to restore


intravascular volume, improve renal perfusion, and correct electrolyte imbalances. The aim is
to improve the patient's symptoms and prevent further complications from worsening heart failure
or AKI.

I
28- Man with Large pleural effusion
therapeutic pleurocentesis indication?

1_pleural glucose more than 6 2-


pleural protin less than5
3_localaized
4_forget??

Bad recall ‼️
.

29- case of a woman with recurrent abortion and lap positive with antiphospholipid
antibodies.
Which treatment to prevent venous thrombosis?

1_warfarine long life


2_heparin 3_enoxaparin
long life 4_rivaroxiban

30- Case (abdomin pain +red jelly


stool)=intussusception Most common site?

1_ileocolic
2_ileoilial

‫ اللحتياط‬2023 ‫هذا السؤال مكرر من الشهر الماضيه وذكرته في ملف مارس‬


Q2: Neonat have constipation and jelly stool Diagnosis by what

A.xray
B. us initially
C.not remembe

Similar recall:
1- https://t.me/smlemay/16370
2- https://t.me/silent_2023m/3092

Similar recall:↓
Child can’t Remember the age exactly, presented with jelly like stool, on abdominal palpation there
was a sausage shaped mass. What investigation is the best in reaching a diagnosis?
A- US. initial
B- CT.
C- contrast enema. Most diagnostic test , confirmatory tests→barium enema. Or the best /
D- MRI.

Dx is: Intussusception
Hx of Colicky abdominal pain and currant jelly stool,

Dx; initial; US
Most diagnostic test , confirmatory tests→barium enema

Tx; fluid then:


if child; Enema
if Adult; surgery

—————————————

Regarding intussusception; shock is the most common complication, sausage shape, target sign,

—————————————

Commonest cause of shock in children; hypovolemic shock

Another recall:
Child with abdominal pain current jelly blood stool where is the patholgy ? A-
Ilioileal
B- Jejenoilial
C- Iliocolon
D- Jejenojejonium
Answer is Ileocecal invagination (most common; accounts for 85–90% of cases)

Intussusception Classification:
• Ileocecal invagination (most common; accounts for 85–90% of cases)
• Ileoileal invagination
• Ileocolic invagination
• Colosigmoidal invagination
• Appendicocecal invagination (very rare)
◆ Answer is Ileocecal invagination (most common; accounts for 85–90% of cases)

Intussusception Classification:
• Ileocecal invagination (most common; accounts for 85–90% of cases)
• Ileoileal invagination
• Ileocolic invagination
• Colosigmoidal invagination
• Appendicocecal invagination (very rare)

Sams Almadani:

The idea that the smallest hole enter to the bigger this is the idea so it’s 1

Amboss
UTD

ileocolic ‫صح‬

‫ و الجابة الكثر دقة هي يعني الصح‬ileocecal


ileocecal junction⬛✓⬛

)ileocolon( ‫ واذا ماهي موجودة نختار‬ileocolic

31- Old man with surgery cancer colon after that few days begin symptoms and signs
DVT
Treatment?

1_heparin
2_enoxiparine
3_IVC filter
⬛J
Telegram comment: enoxaparin if no renal impairment, if renal impairment Unfractionated
heparin (UFH) , if there is ischemic changes:(tpa) Fibrinolytic therapy=thrombolytic
therapy.

The answer done by NaiF, resource is SMLE Notes High Yield J⬛telegram channel
https://t.me/c/1625198149/436 ⬛J⬛

◆ DVT:

Dx; Doppler US (Compressible USG)

Prophylaxis; Enoxaparin (40mg SC daily), in CKD; UFH (BID) ( therapeutic dose is 80mg) Tx;
If Unstable DVT or PE; tpa
Fibrinolytic therapy=thrombolytic therapy: Agents: Fibrinolytics promote the degradation of
thrombi by activating plasminogen to plasmin ~⬛Fibrin-specific agents: Tissue plasminogen
activator (tPA)

If Stable; Apixban≥ enoxaparin (LMWH) 80mg/kg or 40mg BID, heparin 10,000U IV

———

DVT on LMWH, then PE; same Mx (if already on therapeutic dose 80MG)

———-
DVT prophylaxis;

-after retinal surgery OR ICH ; compression

-after rectal surgery; Enoxaparin. Unless in recovery room (within 24 hrs); IVC filter

———-

History of DVT,
if pregnant; LMWH,

If not pregnant; no need

2nd event (DVT or PE); Lifelong anticoagulation

————

V/Q scan for; pregnant with PE and chronic thromboembolism pulmonary hypertension

PE in pregnancy; if leg symptoms; US, if no leg symptoms; CXR: if

abnormal CXR; CTA

if normal CXR; V/Q Scan.


——————————————————
.‫ افضل شي اقرأو عن الموضوع من اي مرجع كويس وممكن السؤال منقول غلط‬/ ‫️⬇هذا من السئله المح ّيرة‬
32- Patient presented with a breast mass
(fiproadenoma) With histology there is lobar in situ
carcinoma
What’s the management ?

A- chemotherapy
B- Radiotherapy
C- Mastectomy
D- hight risk follow up

Naif : Dx is Lobular Carcinoma in situ


a
————————————
33- Young mam present for her new baby (8w)
Good growth
(‫مو متذكرة العبارة اب لضبط بس شي بيشبه‬
spillited milk?)
What to do?

1_reassure not sure


2_pyloric echo
3_refer to Gastroenterologist

Also:
Child 10m from birth there are(spillited milk??!!! ‫ )العبارة من متاكدة مو‬Growth
and development good
Find story with caw milk
Whath to do?

milk thick to milk thin 1_refer


_2‫مامتذكرة الخيارات‬

34- A man complains of weakness in the left half of the body. On clinical
examination, there is paralysis of the facial nerve and hemiplegia
The first in the management?

1_aspirine
2_clopidogrile
3_head ct
4_MRI head

By NaiF: The first step in the management of a patient presenting with weakness on one half of the
body and facial nerve paralysis would be to perform a head CT. This is because these symptoms are
common signs of a stroke, and a head CT can quickly identify if a stroke has occurred. Depending on
the results, further imaging or medical treatment may be necessary. Aspirin and clopidogrel are
antiplatelet medications that may be prescribed in the case of an ischemic stroke, but these should not
be used as the first step in management without a proper diagnosis. MRI head may be needed for
further evaluation, but CT is usually performed first due to its greater availability and quicker results.

35- A man came with a complaint of rectal bleeding , examination there is blood
and there is no rectal mass.
Next ?

1_ Sigmoydoscopy
2_ Colonoscopy
3 _ fecal blood occulte

By NaiF: The next step in the management of a patient complaining of rectal bleeding with no rectal
mass on examination would be to perform a fecal occult blood test. This is a non- invasive test that can
identify the presence of blood in the stool even when it is not visible to the naked eye. If the test results
turn out to be negative, then further investigations may not be required. However, if the test is positive,
then further investigations such as sigmoidoscopy or colonoscopy may be needed to identify the source
of bleeding. The choice of the approach will depend on the results of the fecal occult blood test and the
individual case.
36- Young man check up
(lab) Mcv. Hb. Low
Retice normal
Which?

1_ID
2_SCD
3_thalasimia trait I’m not sure 4_anemia
with chronic disease


Pead History of pics with low MCV low hb 82 (normal 120)but high retics ' management Blood
transfusion
Im iron
ferrous Oral
‫ هذا‬ferrets Drops
‫السؤال نفسه‬

Thalassemia trait
In a young man check-up, a low MCV (mean corpuscular volume) and low Hb (hemoglobin) with
normal reticulocyte count would suggest a diagnosis of thalassemia trait. Thalassemia trait is a
genetic condition where the individual inherits one abnormal hemoglobin gene from one of the
parents. It leads to the production of fewer red blood cells and less hemoglobin, resulting in
microcytic anemia (low MCV) with normal or slightly reduced reticulocyte count.

Iron-deficiency anemia (IDA) is characterized by low MCV, low hemoglobin, and low serum iron level,
ferritin level, and transferrin saturation. In contrast, in anemia of chronic disease (ACD), low MCV is
typically not seen and reticulocyte count can be low or normal. Sickle cell disease (SCD) is also a
genetic disorder but is characterized by the presence of sickle-shaped red blood cells, and thus, a low
MCV may not be seen.

37- Manic disorder in which ?

1_Paranoid schizophrenia 2-
bipolar
3_Anxiety disorder

Bipolar disorder:
Manic disorder is a mental health condition characterized by elevated or irritable mood,
increased energy, decreased need for sleep, racing thoughts, excessive talkativeness, risk-
taking behaviors, and impaired judgment. It is also known as a manic episode, and it is a defining
symptom of bipolar disorder.

Paranoid schizophrenia is a chronic mental health condition where individuals experience


disturbances in thought, perception, behavior, and emotions. It is characterized by delusions,
hallucinations, disordered speech, and impaired social functioning, but it doesn't typically involve a
distinct period of mania.

Anxiety disorder is a group of mental health conditions characterized by persistent fear, worry,
and anxiety. It includes different types of disorders such as generalized anxiety disorder, panic
disorder, and social anxiety disorder. While anxiety disorder can lead to symptoms such as
irritability or restlessness, it doesn't typically involve a distinct manic episode.

38-38 YO female presents to the clinic after finding of hilar lymphadenopathy on CXR.
She has on and off cough, but denies any fever, headache, n/v, weight loss all negative.
Labs insignificant except for X-ray shows confirmed bilateral hilar lymphadenopathy CT
guided biopsy shows noncaseating granuloma
/There was hypercalcemia)
What is the best next step in management?

A-Observe
B-Start prednisolone✓⬛(
glucocorticoids) C-Start azithromycin
D-Start anti TB medication

Naif: Dx is Sarcoidosis⬛✓

Similar recall: https://t.me/SMLE_medicine/701


39-

pregnant 37 w management?

Dx is Complete placenta previa


Or centralis placenta previa

✓⬛Initiate immediate management of antepartum hemorrhage.


≥ 37 weeks: immediate delivery
< 37 weeks: delivery or expectant management
Severe, active bleeding or fetal distress: emergency cesarean delivery Light
or no bleeding and no fetal distress: expectant management Hospitalization
and observation for 24–48 hours
Consider tocolytics to inhibit uterine contractions (in consultation with obstetrics). [19][23] See also
“Management of preterm labor.”

◆ Route of delivery [6][24]:

- Placenta previa: lower segment cesarean delivery ✓⬛


- Low-lying placenta: Shared decision-making is advised.
Lower segment cesarean delivery: usually preferred
Vaginal delivery ± induction of labor: may be considered for stable mothers with reassuring fetal
status
‫‪40-‬‬
‫؟ سؤال طويل عن عالج المالريا‬
‫اخترت جواب‬
‫هيك بيشبه ‪artemether combination‬‬
‫شي‬
-
◆Treatment of Malaria is Very long you can read more about the topic from
Amboss J⬛
Free Amboss account for All
⬛J⬛ ‫ بوس ما حساب‬:
a7mad.med36@gmail.com
iamA7madBoss

41-
‫مريض قصور قلب ايسر وعند معايرة‬
‫اجا سؤال اول مرة يمر علي‬ natiuretic peptide ‫ قيمته كانت‬positive
‫؟السؤال ماهي الحالة التي تسبب ايجابية كاذبة‬

)ferosmid_1‫هي متذكرتا الني اخترتها‬


_2 ‫مابتذكر الخيارات‬

Similar recall:
42- 22man present with cough without sputum + low fever+ sweating+infiltration at
CXR for 7week and twice hemoptysis (dont mention loos weight)
Diagnosis?

1_lung cancer
2_tuberculosia
3_atypical pneumonia
4_

Bader:
In a 22-year-old man presenting with cough without sputum, low fever, sweating, and infiltrates on
CXR for 7 weeks with twice hemoptysis, the most likely diagnosis would be tuberculosis (TB). TB is an
infectious disease caused by the bacterium Mycobacterium tuberculosis, and symptoms can include
cough, fever, night sweats, weight loss, and hemoptysis. The presence of pulmonary infiltrates on CXR
is also consistent with TB.

Lung cancer is less likely in a young, otherwise healthy individual, although it cannot be entirely
ruled out without a more thorough evaluation. Atypical pneumonia is a possibility but may not fully
explain the chronicity of symptoms and hemoptysis. Other conditions such as bronchiectasis or
chronic bronchitis could also be considered in the differential diagnosis.

Overall, given the presentation of chronic cough, fever, night sweats, and hemoptysis in a young
individual with pulmonary infiltrates, tuberculosis would be the most likely diagnosis.
Further evaluation, including sputum cultures and/or a TB skin test or interferon-gamma release
assay, would be necessary to confirm the diagnosis.
——————————————————
The diagnosis cannot be accurately determined with the given information alone. However, some
possibilities include tuberculosis (Option 2) or lung cancer (Option 1).

Option 1, lung cancer, is a possibility since the patient has a persistent cough, low-grade fever,
sweating, hemoptysis, and an infiltrate on chest x-ray. However, lung cancer is less common in younger
patients.

Option 2, tuberculosis, is also a possibility given the patient's symptoms, including cough, low- grade
fever, sweating, hemoptysis, and infiltrate on chest x-ray. Tuberculosis is more common in young
adults and can cause long-standing respiratory symptoms.

Option 3, atypical pneumonia, is less likely given the duration of symptoms and the presence of
hemoptysis.

It is important to perform further diagnostic tests such as sputum culture, tuberculosis test, chest CT
scan, and possible bronchoscopy to determine the underlying cause of the patient's symptoms.

In summary, further diagnostic workup is required to determine the underlying cause of the
patient's symptoms, and a diagnosis cannot be accurately determined with the information given.

Naif:
TB⬛J ‫◆هنا بضيف ملخص لل‬
43- Case about achalasia.(women with dysphagia liquid more than
solid) First?

1_barium sollow⬛✓
2_endoscopie⬛✓⬛✓
⬛✓⬛✓
3_esophageal echo
4_

‫ اسئلة مشابهه بالتعليقات‬+ ‫الرابط هذا فيه نفس السؤال‬


Same Q: https://t.me/saudismle/823

Naif: if there is manometry in choices I will select it as a confirmatory test. If ask about
First , I will go with 2_endoscopie⬛✓

c3Achalasia
⬛J
Approach:
◦ In general, all patients with suspected achalasia should initially undergo upper endoscopy and/or
esophageal barium swallow; findings may support the diagnosis.
◦ Esophageal manometry is indicated to establish the diagnosis (confirmatory test of choice),
irrespective of the initial imaging findings.
◦ If manometry is inconclusive and an esophageal barium swallow was not obtained initially,
esophagram can also play a confirmatory role.
◦ Endoscopy should be performed to rule out pseudoachalasia because the presentation and
manometric findings of a mechanical cause of obstruction (e.g., a malignancy) may mimic achalasia.

————————————
‫️⬇ تواصلت مع استشاري جراحة عامه وهذا رده‬

:‫هذي رسالتي وردة‬


First ‫في الحاله هذي هو سأل عن‬

First➡ ️ endoscopy EGD ‫اذا مل يتم تشخيصه نبدا‬

️ ⬇ ‫اذا توقعنا‬
Best or Gold standard for achalasia ➡ ️ Barium sollow
‫اذا مت تشخيصه ونبغا نتأكد‬
Confirming ➡ ️ manometry

‫؟؟ صح‬

‫* ماشي بس الحقيقية الموضوع فيه فت اصيل كا تر من كدا* صيغة السؤال من لاا ساس خطأ‬: ‫ اسحاق‬.‫د‬
‫ك‬
1- Barium sollow. Look for amboss⬛J
Bader:
The first investigation that is recommended for a woman presenting with dysphagia for liquids more
than solids suspicious of Achalasia is a barium swallow (Option 1).

Achalasia is a motility disorder of the esophagus characterized by impaired relaxation of the lower
esophageal sphincter (LES) and absent or ineffective peristalsis of the esophageal body.

Barium swallow examination can provide functional assessment and detect anatomic defects, such as
incomplete relaxation, distal esophageal dilation, and narrowing of the lower esophageal sphincter. It
has a high sensitivity for detecting achalasia, and the findings are characteristic, including a dilated
esophageal body, "bird's beak" appearance of the distal esophagus, and retrograde contrast filling of
the esophagus.

Option 2, endoscopy may show anatomic abnormalities, such as strictures and masses, but it is not as
sensitive for detecting achalasia. Endoscopy may be necessary to rule out other conditions that can
cause dysphagia, such as esophagitis, eosinophilic esophagitis, or esophageal cancer.

Option 3, esophageal echo (also called Endoscopic Ultrasound) is a useful tool for evaluating the depth
of tumor invasion in esophageal cancer but is not recommended for the diagnosis of achalasia.

In summary, a barium swallow is the first investigation that is recommended for women presenting
with dysphagia for liquids more than solids with suspected achalasia. Other tests
like endoscopy or esophageal echo may be needed to rule out other potential diagnoses.

44- Very long case about construction of intestine


What is contraindicated?

No

Bad recall ¶u¶

‫يمكن مكتوبه غلط ما علينا‬

Random information ⬛J

In case of medgut valvus ( intestinal malrotation )Initial resuscitation: NPO.

◆ Contrast enema helps differentiate complete bowel obstruction from partial


bowel obstruction.
◆ Barium enema is contraindicated if bowel perforation is suspected (water-soluble
contrast enema can be used instead).
45- Case RTA. Pain in chest and fast there are fluids in abdomen
Treatment?

1_lapratomy
2_ct chest 3_
ctA 4_mri

Naif: ( my opinion) is : In this case FAST IS +ve( fluids in abdomen)= stable or not stable we should
do surgical exploration (Laparotomy⬛✓) immediately; may pt needs anastomosis may have
bleeding from arteries. To prevent an intestinal necrosis.
And also Pt may losses a lot of blood s, we have to prepare a 2 units of blood (Blood
Transfusions) for pt.
——————————————
◆ If Pt vitally unstable + Abdominal distended and rigidity + low HG + hypotensive +
tachycardia+ pale =immediately OR for Surgical Exploration [ß) laparotomy To Locking
for site of haemorrhage or spleen rupture.

◆ May needs to do intestinal anastomosis in some cases of Abdominal haemorrhage.

Just similar recall: https://t.me/saudismle/495

Another comment: ⬛J⬛ from Silent (telegram channe)


https://t.me/silent_2023m/6727?comment=37919
◆ IF STABLE > 2
◆ IF UNSTABLE > 1

Look for amboss ⬛J


-pneumonia) with (shingles about case Long Very 46‫لم يذكر التشخيص انت بتعرفو من القصة‬
Treatment?

1_steroid 2-
acyclovir
3_azitromycine

Naif:
◆ Read more about ttt.
◆ More details ⬛J
47- Patient after RTA pain in left chast with open wound 2*2cm ( ‫))بس اب الصغاء موجودات كان كانت شو ماحفظت‬he
is concious and alert.
What to do first?

1_ 3 sides dressing
⬛✓ 2_chest tube
3_ neddle

Naif: ◆If there are any signs of tension pneumothorax do⬛J


1- Needle decompression followed by a chest tube for tension pneumothorax
2- Placement of an occlusive dressing, taped on three sides, for a sucking chest wound
48- Long case and what treatment in HCV?

1_tinidiver
2_sofosbuvir_ledipasvier
3_interferon

49- Case about testicular torsion


The best

1_doppler
2_surgical exploration
Naif: diagnostic tool is duplex US of the Scrotum.
⬛~Testicular torsion is typically a clinical diagnosis. Do not delay definitive treatment for
diagnostic workup if clinical suspicion is high. I will go with 2_surgical exploration⬛✓

Look here⬛J⬛
Similar recalls:
1- https://t.me/silent_2023m/6781
2- https://t.me/BdranSpace/502
3- https://t.me/c/1243365384/2535
4- https://t.me/smle22olds/2753
5- https://t.me/silent_2023m/1681
6- https://t.me/smlemay/5966
50- Pregnant woman with Rhesus
immunization What is most spesific?

1_edma in skin and ascite for fetus


2_Doppler waveform spectral diastole of the middle cerebral artery , measurement of peak systole.

51- Patient with 3CS before now pregnant and placenta deeply in uterine wall

1_accreta⬛
✓⬛
2_inccreta
3_pericreta

⬛J ‫افهم لا صورة‬

full recall

Pregnant 38wk at hospital planned for elective CS , US showed “


Placenta ATTACHED DEEPLY “ into uterine wall , what’s Dx:
A- P. Increta
B- P.Accreta✓⬛
C- P. Penceta
D- Placenta previa

52- Case about nephrotic syndrome/edma+protein in


urine/ The most diagnostic test?

1_kideny biopsy
2_

similar Q:

Case of nephrotic syndrome Asking about most dignostic :


Serum Albumin ⬛S
Us kideny
Total protien
comment

24 HR URINE PROTEIN FOR CONFORMATION OF NEPHROTIC RANGE PROTIENURIA,


BIOPSY FOR CONFIRMING PATHOLOGY WHEN ETIOLOGY IS NOT CLEAR

53- Female 25 years old, presented to the ED with vaginal


bleeding, nausea, and right lower abdominal pain and tenderness.
History of open appendectomy due to perforated appendix. Her
? BP:90/50 HR:120 RR:25
What is the diagnosis??

A-threatened abortion
B-complete abortion C.
Ectopic pregnancy
D. Abscess

54- Patient present after surgical treatment for molar and these values
BHCG Before 180,000
After
8000
700
25
1
What to do now?

1_sent from clinic


2_bHCG after1week 3-
bHCG after1month

H)Hb1c normal glucose Fasting bid 1g metformine on DM2 with Patient 55- 5,5 (6,9 ‫المجال‬
‫ المرجعي يلي عاطينو بالسؤال القيمة العليا هي‬eigh
Random glucose heigh
What to do?

1_no change 2_increase


metformin
_3‫ اسم دوا بس ما قدرت احفظو تماما اخره‬tid
recall _4‫ كمان اسم دوا اولو‬sam

full

60 Y.O. female patient diagnosed with type 2 diabetes. She started metformin 1g BID.
After 3 months her FBG is normal. RBS is normal. HbA1c is 6.9. which of the following
is the best next step?

A. Increase metformin dose.


B. Add glimepiride.
C. No need to change.

D. Add DPP-4 inhibitor.

T2DM management
When to start:
A1C at or above target level (ie, >7.5 to 8 percent), metformin should be initiated at the time of diabetes
diagnosis.
A1C near target (ie, <7.5 percent), a three- to six-month trial of lifestyle modification before initiating
pharmacologic therapy is reasonable.
For patients presenting with A1C >9.5, insulin remains the preferred initial therapy.
Metformin monotherapy failure:
Insulin is the preferred second-line medication for patients with A1C >8.5 percent despite initial therapy
with metformin and lifestyle intervention.
For those close to A1C target (<8.5 percent), we prefer to add a sulfonylurea rather than insulin

Special considerations:
For patients in whom ischemic heart disease predominates we typically prescribe GLP-1 agonist
(liraglutide, semaglutide).
For patients in whom heart failure or chronic kidney disease predominate, we prescribe a low dose of an
SGLT2 inhibitor (empagliflozin, canagliflozin, dapagliflozin).
56- Case about SCD(picture for blood film)
+lab Treatment??

1_plasma exchange
2_steroid+IVIG
3_iron supplements

57- which of the following is more specific for SLE ?

anti smith

Naif:
Similar recall:
1- https://t.me/saudismle/616

Another Similar recalls:⬛J⬛


1- https://t.me/BdranSpace/111
2- https://t.me/FebruaryDoers/89

58- Pt has severe headache associated with neck pain, ct scan appears normal, what
next to diagnose her?

LP

Similar recall

Pt has severe worst headache ever for several hours associated with neck pain, ct scan appears
normal, what next to diagnose her?
A. mri brain
B. Lumbar puncture
C. MRA

Diagnosis : Subarachnoid hemorrhage


CT normal —> go to LP

Suhaib
UTD : Initial evaluation for subarachnoid hemorrhage must include noncontrast CT of the brain.
Lumbar puncture is indicated when there is clinical suspicion of subarachnoid hemorrhage and
the head CT is normal.
59- case of pseudomembranous colitis:

oral vancomycin

from nilsson

60- case of primary sclerosing cholangitis :

ursodeoxycholic acid

UTD : Ursodeoxycholic acid — UDCA, a hydrophilic bile acid, is the most extensively studied of all
medical treatments for PSC. UDCA, at a dose up to 15 mg/kg per day, is thought to exert its effects in
cholestatic conditions via protection of cholangiocytes against cytotoxic
hydrophobic bile acids, stimulation of hepatobiliary secretion, protection of hepatocytes against bile
acid-induced apoptosis, and induction of antioxidants.

61- case of pneumonia to confirm diagnosis

chest xray

SIMILAR RECALl poll

pt with clinical ss of pneumonia which of the following is confirmatory test?

A Sputum culture

B Blood culture

C X ray
D Ct

Pneumonia is a Radiological DX

62- most common type of lupus nephritis diffuse proliferative ttt:

pulse steroids + cyclophosphamide

Dr. Nagla Danial


‫مش الجابه‬
Mycophenolate most specific in kidney
Cyclophospamid for CNS
J⬛ ‫ام القرى‬
similar recall

Lupus nephritis class I (given) Whats the appropriate management


A reassure
B steroids

Reassure but Control BP if high

LUPUS NEPHRITIS
Patients typically have extrarenal symptoms of SLE at the time of diagnosis of LN.
ANA and anti-double-stranded antibodies are positive, and C3 and C4 complement levels are
depressed.
Classification and recommended treatment of LN is made after kidney biopsy: Class and II
(minimal or proliferative mesangial) lesions require no specific therapy.
Classes III and IV (focal and diffuse glomerular lesions) are treated with high- dose
glucocorticoids and either IV cyclophosphamide or mycophenolate mofetil. Class V
(membranous) LN has a course similar to idiopathic membranous glomerulopathy
63- case of proteinuria most definitive:

renal biopsy

64- case of encephalitis antineural ab lupus encephalitis ttt:

steroids + cyclophosamide

65- dr explained the importance of cardiac catheterization& alternative, pt refused.

-respect pt

full recall

patient came to the ER with signs and symptoms of myocardial infarction he was gonig
for PCI when the cardiologist was assessing his condition,he notices the patient was
depressed with a low mood, and refused the PCI What will you do?
1. Treat the patient regardless of the consent
2. Refer the patient for psychiatric assessment then take the consent
3.Respect the patient's choice after discussing the reasons for
refusing Last one i think take the consent from relatives
My Answer is 2

66- pt with hiv developed cough ppd 7, last one year ago 5,

start anti tb drugs


incompleat Q

similar Q:
HIV patient with ppd of 6 and unremarkable chest x ray
What preventive medication

izonizide and pyrizinamide for 3mo


Izonizide and pyrizinamide for 9
mo All 4medications of TB
No need for preventive medication

HIV child with ppd 5 or 6mm?


No need for treatment
Rifambicen and pyro
All tb treatment

Suhaib
Treatment of latent TB according to UTD and Amboss :-

1-isoniazid + Rifapentine for 3 months (3HP) given via Directly Obesreved Therapy (DOT) -> the
preferred regimen for HIV patients
2- Or Rifampin daily for 4 months (4R)
3- Or Isoniazid for 6 or 9 months (6H or 9H)

-If isoniazid used add pyridoxine (B6) for peripheral neuropathy

67- case of MRSA ttt:

vancomycin
precautions->contact

similar Q:

60y o male with prostatic valve depolved IE blood cultures done and were positive
MRSA what is antibiotic regimen ?

vancomycin

mportant precautions 🚦
TB + measles + chicken box > airborne
Meningitis + pneumonia > droplets
MRSA > contacts
68- pt doing ct guided biopsy developed severe dypsnea next initial step:

insert canula in 2nd left interspace

incomplete Q: from what i read its a chest tube

69- pt with DKA resident inserted cvp for fluids developed dypsnea :

pneumothorax

70- case of palpitation wt loss increased appetite:

before ecg do thyroid function

the diagnosis is hyperthyroidsm


71- ttt of adrenergic crisis?

phentolamine

72- diagnosis ms?

mri brain and spine


73- DM pt high BP?

enalapril

similar Q:

74- polymyositis rheumatica

high cpk

incomplete recall
75- case of mixed connective disease

A. anticentromer

comment
Anti RNP AB

76- upper resp infection presented by hematuria. recurrent attack. urine: protein,

rbcs minimal disease

full recall

30 man gross hematuria which started around the same time as a now resolved sore throat and a fever.
BUN 9 Cr 1.0 .urine specimen shows 15-20 dysmorphic red blood, +3 protein and 1-2 RBC casts.?
a. IgA nephropathy.
b. acute tubular necrosis
c. goodpasture syndrome.
d. minimal change disease

77- prostate cancer


PSA

similar Q:

65 old male his father had prostatic cancer ‫ يعمل ويريد‬check up ‫ و‬screening
palpatory examination ‫عمل وتم‬
smooth‫ و‬normal size ‫وكانت البروستاتا‬
palpated sulcus median And ‫ مش متاكدة من اسمه المهم يسال عن‬approch next most
A. psa
‫ شهور‬٣ ‫ فحوصات كل‬b.

resource → uptodate

78- ttt of campylobacter azithromycin?

ciprofloxacin

79- anti dm cardioprotective

A. metformin

comment:

- GLPT
incomplete Q

resource → uptodate

80- DM type 1 in honeymoon ttt?

Reduce insulin dose

similar recall

12 years old boy diagnosed with type 1 diabetes, he has been compliant with insulin since the diagnosis
2 months ago,.. with early morning hypoglycaemia, What is this called?
A. Somogyi phenomenon
B. dawn phenomenon
C. brittle disease
D. honeymoon period

resource → uptodate

81- ttt of trigeminal neuralgia,?

carbamazepine
82- case of siadh associated with hypokalemia 2.9 in addition to fluid restriction

give iv k

83- case of lung cancer hyponatremia SIADH.

fluid restriction
84- pt head trauma devolped polyuria

cranial Diabetes insipidus


85- initial management of DKA.

normal saline
86- case of shoulder pain cough, eye ptosis in heavy smoker pt,

do Chest xray
87- thyroid crisis after surgery ttt.:

iv iodine ،propranolol ، propylthyrocil ،steroids

88- painful genital ulcer lymph adenopathy:

HSV

89- 19yo boy w/ acute onset right testicular pain, nausea for 5 hours, swollen and
tender testis in a horizontal line w/ an absent right cremasteric reflex. What is the
definitive management?

a_manual detrosion
b_operative orchiopexy
c_ antibiotics
d_ analgesia

◆ Surgical intervention is recommended for suspected testicular torsion, regardless of


radiological findings.

◆ Testicular torsion is a medical emergency and should ideally be treated within 6 hours of the onset
of symptoms for the best chance of testicular salvage. Manual detorsion in the emergency department
may be attempted prior to surgery for immediate pain relief, but should not delay transferring the
patient to the operating room.

◆ Exploratory surgery [14][3]


• Indication: suspected testicular torsion
• Timing: ideally, within 6 hours of symptom onset [Surgery is also indicated > 6 hours after
symptom onset because the viability of the testis is difficult to predict.]
• Procedure
◦ Immediate surgical exploration of the scrotum with reduction (untwisting) and orchidopexy of the
affected testis
◦ Orchidopexy of the contralateral testis is recommended because the risk of testicular torsion on the
contralateral side increases with previous or current testicular torsion. [3]
◦ Orchiectomy if the testis is grossly necrotic or nonviable ( The patient should be informed before
exploratory surgery that orchiectomy might need to be performed depending on
intraoperative findings. Histopathological examination on the excised testis should be
performed ).

+I exclude C
+A&D in ER as Next
◆ My Answer is B

◆ Explaination ⬛J
+.+CThe TWIST score may be used to help guide decisions on imaging prior to surgical
exploration.
C+.+Testicular workup for ischemia and suspected torsion score (TWIST score)

Then⬛J

◆ Immediate surgical exploration of the scrotum with reduction (untwisting) and orchidopexy of the
affected testis ⬛✓
In This Q the

B is the nearest answer ß)[

◆ Do not delay urological referral in order to perform imaging or manual detorsion, especially if
clinical suspicion is high, as there is a significant risk of infertility and/or testicular loss if definitive
surgical management is delayed > 6 hours from symptom onset.

90- Long Difficult case of 43y/o man w/ 1 month hx of worsening frontal headaches,
blurred vision, occasional falls, the blurry vision is provoked when he leans forward,
his symptoms interfere with his sleep. no Hx of head trauma or seizure. occasionally
get a head cold at this time of year. which of the following is the most likely cause of
this patient’s condition?

a. high intraocular pressure


b. meningeal irritation
c. paranasal sinus inflammation
d. intracranial hypertension

C✓
because he talks about climate and symptoms of allergic rhinitis

C✓
due to he get this condition at the same TIME every year so its seasonal

91- 3 months F, w/ diarrhea, vomiting, ftt, edema, ascites, hypoproteinnemia. suspects


due to the genetics deficiency of an enzyme that is normally expressed by the
intestinal brush border cells, the absence of this enzyme results in the inability of the
infant to digest and absorb proteins in her diet. Administration of pancreatic enzymes
alleviates the patient’s symptoms, which of the following enzymes is most likely
deficient?

a. enteropeptidase
b. pepsin
c. trypsin
d. chymotrypsin

92- 50sy, man,decreased libido, inability to achieve, maintain erection for the past
several months, 5kg weight loss, drinks alcohol, BP is 110/70, bilateral Gynecomastia
and firm, small tests. normal TSH,t3,T4. whats diagnosis?

a, chronic liver disease.


b, testosterone insensitivity. c,
adrenal insufficiency.
d, exogenous thyroxine intake
93- 58y,F, Menarche was at age 14 and menopause occurred 4 years ago, A cousin
on the paternal side of her family died of breast cancer, which of the following is the
greatest risk factor for breast cancer in this patient?

a. family History of breast cancer


b. age at first pregnancy
c. duration of breast feeding
d. chronological age

I think it's B and there is a mistake it's age at first period


94- gait instability, right arm weakness. also had problems with balance and walking
for the past 2wks, no head trauma, mild headaches, fatigue, anorexia, night sweats and
chills over the last week. Hx of iv heroin use, and his last injection was a day ago,
temperature is high, BP is 142/75 Approximately.
multiple needle tracks on the arms. There is a grade III holosystolic murmur heard
loudest at the cardiac apex. Neurological examination shows right lower facial droop
and 2/5 strength in the right upper limb, has a wide based gait, difficulty with heel to
shin testing. Urinalysis shows red cell casts, proteinuria, cause of symptoms?

a. neurosyphilis
b. cerebral septic stroke
c. small vessel liptohyalinosis
d. progressive multifocal leukoencephalopathy

95- increasingly heavy vaginal bleeding and fatigue over the past 6 months, fm hx for
myocardial infarction, Abdominal exam shows firm, palpable mobile mass just blew the
umbilicus, on pelvic examination, and there is a moderate amount of old, clotted blood
coming from the cervical OS, hematocrit of 24% and hemoglobin is low, which
diagnostic test is most cost effective in confirming a diagnosis?

a. transvaginal US
b. hysterosalpingogram
c. abdominal and pelvic CT
d. diagnostic laparoscopy

https://t.me/smlemay/17416?comment=202932 (A
or c )

96- 51yo woman with no colon related strange sx, Her medical history is significant
for HTN and her mother developed colon cancer at age 85. Her examination is
unremarkable. She underwent colonoscopy during which 2 small 8mm hyperplastic
polyps were removed, which of the following is the most appropriate next step in
management?

a)colposcopy in 10yrs.
b)CT scan abdomen.
c) mismatch repair genetic testing.
d)fecal occult blood test in 6 months.

Answer is c https://t.me/smlemay/17415?

comment=202929

97- 30y woman with muscle weakness and weight gain began 3 months ago. Initially
face was getting fat and Most recently she has gained additional weight in the truncal
region and seems to have difficulty concentrating. BP is high, The PE is notable for a
facial plethora, the presence of supraclavicular fat pads and truncal obesity with
prominent purple stria principally in the abdominal region, what is the next step in
management?

a. 24 hours urine collection


b. plasma ACTH measurement
c. MRI of the head
d. CT of the abdomen

Answer is b https://t.me/smlemay/17417?comment=202922
S : Answer is A first to diagnose cushing as a next step, then further testing to know if it’s
ACTH-dependent or ACTH-independent
98- 20s F, w/ hair loss for 6 months, extensive facial and back acne. BMI IS 33 KG/M2,
thinning hair at the forehead with a receding hairline, the thyroid is nonenlarged,
abdomen is obese with no masses. urine p/g test is negative, serum prolactin, thyroid
function test are normal, which additional test is indicted in this patient?

a oral glucose tolerance test


b BRACA MUTATION TESTING
C diagnostic laporoscopy d
iron studies

hirsutism, acne, abd enlarged, DM = PCOS

Bader & Noor :


PCOS
Or
alopecia Androgenic ‫على م اعتقد‬
pcos ‫الردي من اسبابها‬

test glucose ‫في الحالتين‬


‫هو القرب و النسب اذا بوزتف و البيشنت بري دايبتك او دايبتك يعني عندها‬
insulin resistance
‫النها‬obese
99- man is in a motor vehicle accident, his right knee underwent a forceful collision with
the dashboard. His right foot is pale and pulseless. A radiograph shows a comminuted
distal femoral fracture with a posterior knee dislocation, which vessel was likely
injured?

a posterior tibial artery


b superficial fibular artery c
popliteal artery
d anterior tibial artery https://t.me/smlemay/17419?

comment=202925

100- 29,m, with acute rt upper quadrant pain. episodes each occurring while eating
meals. history is sickle cell dieses. right upper quadrant tenderness with
palpitation. Right upper quadrant US reveals numerous gallstones and edema of the
gallbladder wall. cause of gallstones in this patient?
a. chronic hemolysis
b. use of total parenteral nutrition
c. ileal disease
d. biliary infection

101- indication of non invasive mechanical vent :

exacerbation of copd

102- Dm ckd peripheral neuropathy:

pregabalin

103- case of gouty arthritis ttt :

colchicin

104- old age pt on antipsychotics analgesics presented drwsy rt abdominal pain Abg
metabolic acidosis :

acetaminophen toxicity
105- knee swelling aspiration most common organism:

staph aureus

106- nurse will be late for 5 min you asked female pt stay 5 min :

privacy
Similar recal
243- Male dr wants to examine a female patient and he asked the nurse to come with
him what did the doctor practice ?
A- privacy
B- justice
C- Nonmaleficence
https://t.me/BdranSpace/259

107- During exam of pt of different religion :

exam as any pt

108- case of ascites neutrophil count in ascitic fluid 500 best next step :

iv Antibiotic

109- post thyroidectomy hoarsenee:


recurrent laryngeal

https://t.me/silent_2023m/5086?comment=29922

110- pregnant woman no medical hx low tsh normal ft4 :

no ttt

111- obstructive sleep apnea :

cpap
112- comunity acquired pneumonia best single ttt :

Moxifloxacin

● Generally Tx depond of CURP-65, inpatient, outpatient Tx If


aske mono therapy ->> fluroquinines [Moxifloxacin]

113- Pregnant Hyperthroid Ecg SVT :

adenosine

114- pt traveled to Tailand fever, rash, LN :

dengue fever

● Distribution: tropical regions worldwide, particularly Asia (e.g., Thailand)


● Incidence
● Most common viral disease affecting tourists in tropical regions
∼ 400 million infections per year worldwide

Argument:

Doc1: Can be HIV??


1- dengue Fever widely distributed thorough tropic areas eg: Thalind

Doc2: How come ?? Dengue fever transmit by mosquito but HIV is by direct un protective contact
● ransmission route:
->> Vector-borne: mosquitoes most commonly from the species Aedes aegypti

115- Which one of the following indicates Rheumatoid arthritis in pleural effusions?

Glucose <1mmol
● RA affects the pleura in up to 5% of cases and, in keeping with other extraarticular
manifestations of the disease, is more common in men. Rheumatoid effusions characteristically
have a low glucose and pH. If the pleural fluid glucose is >28.8 mg/dl, rheumatoid is an
unlikely cause.

116- pt with lymphoma no symptoms coming f/up developed fever lumps.

diagnosis lymph node biopsy

Biopsy in essential for DDx

117- pt on chemotherapy fever neutropenia abdominal pain:

typhilitis

118- hospital needs quality accreditation :

safety measures

119- case of autoimmune hepatitis most definitive:

liver biopsy
120- porphyria cutaneous :

HCV
121- PAN associated with:

HBV
122- case of tonic clonic convulsion:

brain imaging
● If no focal neurological deficit -> MRI superior to CT
● It there if new focal neurological deficits, fever, coagulopathy -> CT

123- case of stroke first test :


non contrast ct brain

124- pneumocystis infection ttt :

TMP_ SMT
125- postrenal transplant deterioration of liver functions, viral infection
cmv ttt:

ganciclovir

In immunocompetent patients: No specific treatment is needed.


In immunosuppressed patients: Optimize antiretroviral therapy regimen to increase CD4 count above
100 cells/mm3
[ valganciclovir]: the dose/ route depots on which part is affected

126- empirical ttt of hsv encephalitis:

acyclovir
127- to decrease complications of meningitis:

dexamethasone

128- thyroid nodule FNA show high tsh low ft4 +ve Tpo

hashimoto
129- child examination multiple bruises:

inform authorities

● Need further information, could be child abuse, others hematological disease.

130- dr insist on visiting client in isolation what role did he break?

Privacy?[need check]

131- Resident teaching medical students infection control first to learn :

hand washing [ first thing to do]


-ECG of digitalis toxicity

Downsloping ST depression with a characteristic “reverse tick” or “Salvador Dali


sagging” appearance. Flattened, inverted, or biphasic T waves. Shortened QT interval.

-ECG hyperkalemia insulin + glucose


● intial step of Hyperkalemia ->>Ca Gluconate[ heart protection]
● Then insulin + Glucose to decrease serum K level.

-ECG inferior mi

-ECG j wave, hypothermia


132- 68Y/O man,HX of T2DM diabetes mellitus and coronary artery disease, undergoes
a left total knee replacement. on postoperative day 3. On postoperative day complains
of new abdominal discomfort. Over the past 12 hours, he has spontaneously voided
200ml of urine, medical chart shows infrequent recording of fluid input/output.
Abdominal diffuse discomfort and difficulty in appreciating masses due to obesity,
Laboratory results are as follows; hemoglobin low, serum sodium low, Bicarbonate; 21
mEq/L, blood urea nitrogen very high, serum Creatine: high, his renal function was
normal prior to the surgery which of the following is the most appropriate next step in
management of this patient?

a, bladder catheterization b,
Intravenous fluids
c, immediate hemodialysis d,
CT angio abdomen
● low UOP + high renal function tests ->> suggest pre-renal AKI, as a consequence of
Hypovolemia
● Most appropriate NEXT ->> IV fluids
● After IV fluid ->> insert bladder cath to monitor UOP

133- motorcyclist in motor vehicle collision,he suffered a direct blow to the lower
abdomen and pelvis; he has diffuse abdominal pain and a dull pain in the left
shoulder. no deformity of the left shoulder is noted and complete range of motion is
preserved. diffuse abdominal tenderness with guarding is present. which one of the
following injuries is most likely to be seen on CT scan of the abdomen in this patient?

a transection of the membranous urethra. b


transection of the anterior urethra
c renal laceration
d bladder dome injury

● Peritoneal irritation caused by interperitonal bladder injury ->> urin leakage + Lt,
shoulder pain caused by diaphragmatic iritation

134- 45Y Man and a 2 year history of dull, non-specific flank pain that subsides with
rest, the patient BP Is high. Abdominal examination is significant for palpable bilateral
renal masses. Laboratory tests are significant for creatinine of 1.9mg/DL and bun of
24mg/dL. Which test is recommended for this patient?

A. coronary angiography B.
stress echocardiography
C. chest X-ray
D. CT angiography of the heart
135- Woman in her 50s has an aortic valve replacement 8 months ago presents
complaining of fatigue, hemoglobin low and her reticulocyte count percentage is
high, wbc and platelets normal, what is the most likely finding on a peripheral smear?

a. schistocytes and burr cells


b. sickle cells and target cells
c. target cells and round macrocytic RBCs
d. heinz bodies and bite cells
136- 20s man with a rash, reddish burns began to develop on his chin 4 weeks ago,
painful, stools have becomes looser with intermittent crampy abdominal pain, lost 6kg
in a month.several erythematous tender, non-mobile nodules on the anterior lower
extremities, The abdomenis mildly tender to palpation ,which condition explain this
presentation?

a. celiac disease
b. inflammatory bowel disease
c. colon cancer
d. IgA vasculitis

IBD : cramping abdominal pain, wt loss, diarrhea recurrent oral lesions, extraintestinal
manifestation; erythema nodosum.
137- paramedics were called to the home of a 19yr young man who was found lying on
the floor of his apartment, unresponsive. At the scene the patient’s bp was 102/80, his
pulse 60 and respirations rate was 12 and shallow. The patient’s pupils were
constricted and non-creative, his skin was cool, cyanotic, at the hospital, doctors
were able to resuscitate the patient and he eventually maintained normal, vital signs
after several hours of treatment, which of the following most likely caused this
patient’s initial symptoms?

a cocaine.
b phencyline.
c methamphetamine. d
heroin.
● [ from First Aid step 3, 7th edition]

138- woman, who is 8 weeks postpartum, intermittent weakness of the upper and
lower extremities, says, my arms feel weak after carrying the baby for a few minutes
and I can’t lift him until I rest for some time. Yesterday evening, I couldn’t even get off
of the toilet, but today, I feel fine. had episodes of blurry vision, pregnancy was
complicated by gestational diabetes. Deep tendon reflexes and muscle strength of the
bilateral upper and lower extremities are normal. What's diagnosis?

a) myotonic dystrophy
b) multiple sclerosis c)
myasthenia gravis.
d) diabetic amyotrophy

MS ->> increased deep tendon reflexes [ BMJ best practice] MG -


>> Normal deep tendon reflexes [ BMJ best practice]
139- Patient with LF side heart failure, positive Natiuretic peptide, which of the
following cause false positive for NP?

A. Ferosmide
B....
C...
D..

COPD?[ is the answer ] check other causes of False -ve below


140- physician goes on Friday evening to check on the labor progression of her
patient. The physician is leaving town for vacation in a few hours, but an on-call
obstetrician will be available for deliveries. The physician recommends a cesarean
delivery. The physician’s inability to be at the delivery in a few hours, this situation
demonstrates which of the following ethical principle?

a. conflict of interest
b. beneficence
c. directive counseling
d. Justice
141- 50s, F, acute onset of abdominal pain followed by syncope. The patient was in an
exercise class when she suddenly had pain in left side of her abdomen quickly followed
by loss of Consciousness for 30 seconds. nausea, dizziness, diffuse abdominal pain,
k/c of htn, atrial fibrillation, diverticulitis and chronic lymphocytic. bp 80/54 pulse 120.
the abdomen is diffusely tender with decreased bowel sounds. hemoglobin
8.4g/dL(10.8 one month ago), platelets decreased ,WBC 27,800/mm3, lymphocyte 90%,
PT increasd , INR 1.2, PTT 37 sec, what is most likely diagnosis?

a. septic shock
b. splenic rupture
c. ischemic colitis
d. mesenteric artery occlusion
142- long case of 7yo girl, fever, fatigue, loss of appetite and SOB. imigrated from Syria
one year prior and the patient has yet to receive any childhood vaccinations. Today
she is febrile, hypotensive and tachycardic, cervical lymphadenopathy and grayish
pharyngeal exudate. A sample of the exudate is obtained and demonstrates
pseudomembranous exudation with subsequent cultures demonstrating gram-positive,
highly pleomorphic organisms. In addition, she is diagnosed with myocarditis, her
condition is most likely the result of the following processes?

a. Exotoxin induced intracellular protein ribosylation


b. enterotoxin induced damage
c. exotoxin induced inactivation of the 60s ribosomal subunit
d. autoimmune mediated damage.
143- 52s woman comes to the ed due to speech arrest and right arm weakness. The
symptoms began suddenly at breakfast but resolved after 30 minutes, the patient has
a history of relapsing-remitting multiple sclerosis. Three months ago she was
hospitalized due to blurry vision in the left eye, which was treated w/ corticosteroids
and improvement after several weeks. The patient’s other medical issues include HTN
and hyperlipidemia. She is currently not taking any disease-modifying therapy for her
MS. BP is 170/96 and pulse is 92. BMI is 35kg there is a mild afferent pupillary defect in
the left eye w/ diminished visual acuity, which of the following is the next step in
treatment of this patient?

a. initiate aspirin and stain therapy


b. initiate disease modifyigng agent
c. initiate intravenous heparin infusion
d. initiate plasma heparin

Case diagnosed as TIA [ many risk factors mentioned in the senoario] but MS still differential
diagnosis because MS relaps mimic stroke.
144- 42y,F, severe pruritus, antihistamine have not relieved it and she frequently
awakens at nights of severe itching. bilateral scleral icterus. Her Hx is remarkable
for celiac disease, adheres to a gluten free diet, total bilirubin 1.4mg/dl –direct
bilirubin 0.9mg/dL –alp is high – AST is high – ALT is high. diagnosis?

a. primary biliary cirrhosis


b. Wilson disease
c. hemochromatosis
d. autoimmune hepatitis
145- 61yo woman, for 3 months hx of constipation and pain in the left lower
abdomen. The pain is crampy, continuous and non-radiating and is not affected by
bowel movements or meals. The patient describes her stools as separate hard lumps
with early satiety, mildly distended abdomen with tenderness in the left lower
quadrant, what is the most appropriate next step?

a. pelvic ultrasound
b. barium enema
c. pelvic floor muscle exercises
d. fiber supplementation.

146- 4yo, delayed speech, you discuss with the mother that a speech delay can be the
result of a hearing deficit, what is the best predictor of this child’s hearing deficit?
a. birth weight of 1215g
b. mother with eclmapsaia
c. course of ampicillin
d. Second cousin deaf
147- Case about breaking bad news in OBGYN; 27yo at 18 weeks gestation comes for a
routine prenatal visit. The patient feels well and accompanied by her mother as her
husband was unable to get off work, us shows a cephalic singleton with severe growth
restriction, bilateral choroid plexus cysts, clenched fists and a large ventricular septal
defect, appropriate initial statement by the physician?

A. “there are some things about your ultrasounds that i need to discuss with you, is it okays to do that
now?”
B. “would it be okay if your mother stepped out of the room so we can talk about your results”
C. Let’s schedule a follow up visit to discuss the results when your husband can be there here with
you”
D. “could i speak to you in private about my concerns with your ultrasound findings?”

148- In her 20s female with easy bruising, oozing from her gums and nosebleeds 1 week
in duration, weakness and a high fever for 3 months. blood smear shows large
erythrocytes and a paucity of platelets and granulocytes, MCV is increased,
RETICULOCUTES are absent. bone marrow is aspired but appears diluted on a smear.
Dx?

a. aplastic anemia.
b. Megaloblastic anemia
c. multiple myeloma
d. Myelodysplastic syndrome

149- At 42 weeks gestaion is admitted to delivery, pregnancy has been uncomplicated


and fetal heart rate monitoring shows baseline of 130/min, absent variability and the
patient is informed of the need for an emergency cesarean delivery due to significant
risk for fetal asphyxia and death, however she refuses to undergo cesarean delivery,
reveals that the patient fully understand the risk refusing the procedure and continues
to insist on virginal delivery, which of the following is the most appropriate next step?

a) respect the patient’s decision and proceed with vaginal delivery.


b) proceed with emergency caesarean delivery because the fetal rights are more important than the
patient’s autonomy
c) proceed with emergency cesarean delivery because there is evidence of fetal distress
d) seek a court order forcing the mother to undergo emergency caesarean delivery.

150- admitted for a low anterior of rectal cancer, two days after surgery, the patient was
recovering uneventfully but suddenly become anuric, she has had a Foley catheter in
place since the operation with 50 mL/hr of clear yellow urine, vitals normal limits , what
is the explanation for sudden anuria?

a. catheter dysfunction. b.
acute renal failure
c. rectovesicular fistula.
d. hypovolemia

◆ ‼️ short term post op complications = Urainary rentetion As a


complication of UR is acute renal failure .
https://www.ncbi.nlm.nih.gov/pmc/articles/
PMC2945495/

151- In 60s woman was recently treated for uti, at first began to recover, 10 days
following the completion of a course of oral penicillin. She develops a rash, fevers,
hematuria and proteinuria, most likely diagnosis?

a Acute interstitial nephritis


b crescentic glomerulonephritis c
acute renal tubular necrosis d
pyelonephritis

152- I don't remember the case correctly but asks, Beside luteinizing hormone, which
hormones best interacts with and activates the LH receptor?

a. hCG
b. Fsh
c. Estradiol
d. GnRH
153- Long case of 31y man, fatigue weight loss, sweating over the last months.
opacities in the mediastinum. presences of CD15+ binucleate cells. most predictive of a
good prognosis in this patient?

a. Age
b. Serum albumin 3.9
c. presence of CD15+ cells
d. short duration of symptoms

154- 26yr woman presents to ed w/ fever, rash, the patient states that her symptoms
started about 6 hours ago. She woke up w/ a headache and nausea throughout the
morning. She developed vomiting and diarrhea. An hour before arrival, she noticed a
diffuse erythematous rash and she is currently menstruating and uses tampons. BP
86/45, heart rate 116, respirations 18, What organism is causing symptoms?

a. S. aureus
b. Klebsiella pneumoniae
c. n. meningitidis
d. E. coli

155- 51y, presents hot flashes, irregular periods and asks about the possibility
of entering menopause,which parameter is required to diagnose menopause?

a. cessation of menses for at least 12 months


b. increased serum FSH.
c. increased serum LH
d. pelvic ultrasound demonstrating decreased follicular activity

156- pt history of thrombosis and do sleeve surgery what the aideal INR FOR HIM?

A/1_1.5
B/2_3
C/3.5_4
D/NO NEED ANTICOAGULANT

-GBS 157 ‫سيناريو طويل ومطلوب العالج‬

158- LESION OR SAC AT THIGH ROUN MOVAEBLE FLUCTUATING?


1_LYPOMA
2_SARVOMA
3_ADENOMA
4_?

159-
‫سؤالين‬
SHOCK
‫واحد منهم مريض قلب وعمل عمليه في القلب وحاطط تحاليل منهم‬
WEDGE 4
‫مقالش‬
(HIGH OR LOW )
CARDIAC OUT P.2.5 ?

High pulmonary wedge pressure


Low cardiac output
High systemic vascular resistance

-160 ‫حاله واعراض وبعدين مطلوب‬


MOST COMON TYPE OF MALARIA ?

1_P .FALCIPARUM
2_P .MALARIA
3_P .OVAL
4_P .VIVAX

161- ‫ حالتين‬ECTOPIC PREGNANCY

WEEK 32 Q/ ? Action 150/90 ‫ وجايب صورة تحليل البول انها‬PROTINURIA 3+

1/admite and observation


2/IOL
3/ADMIT +MG SULFAT
4/SC

‫ ؟ نوع‬-TYPE DM 162 2 +‫تحاليل الكلى كلها عاليه‬

1_DIABITIC NEPHROUPATHY
2_ACUTE RENAL F.
3_??
163- 17 year femal fit and low wight and chronic diarrea and abd pain no bloody
‫؟ وحاطط تحاليل مفيش انيميا‬

1_celiac
2_crhons
3_uc
4_?

164- amyloisosis
‫مش متزكره السيناريو بس وجع في الضهر والجنب ووظايف الكلى اتاثرت‬

165- 30s,F, abdominal uterine bleeding, menstrual cycles were monthly until a year ago,
then began to have irregular menses with heavy bleeding and passage of clots but no
cramping, she has not had a menaces of period for the last 4 months, the patient has no
hot flashes, acne or abnormal hair growth, she takes levothyroxine for hypothyroidism
and her only surgery was tubal ligation after her last delivery, speculum exam is normal
and bimanual examination is limited by body habitus. Urine pregnancy test is negative
,Lab: TSH normal, prolactin normal, FSH normal. What’s diagnosis?

a. anovulation
b. primary ovarian insufficiency
c. uterine fibroid
d. endometrial polyp

166- which of the following disorders is associated with the same gene mutation of
congenital bilateral absence of the vas deferens, chronic pancreatitis or sinopulmonary
complaints?

a. klinefelter syndrome
b. sickle cell anemia
c. marfan syndrome
d. cystic fibrosis

167- 68yrs old woman with HX. of emphysema presents to the emergency department
w/ exacerbation of her baseline cough and dyspnea. Her respiratory rate is 22. The
rest of her vital signs are within normal limits, she is afebrile. Pulmonary auscultation
demonstrates loud expiratory wheezes and rhonchi. A cardiac examination is
unremarkable. An arterial blood is performed, which of the following results is
expected?
a_Ph of 7.34, pCO2 of 48, and Po2 of 63. b_
ph of 7.18, pco2 of 62 and po2 of 44. c_ph of
7.32, pco2 of 52 and po2 of 90. d_ ph of 7.48,
pco2 of 28 and po2 of 78.

Bader Alsawadi:
✓⬛⬛ C , emphesyma decreases the ph 0.3 persent , all Vitals are normal so B and D incorrect , A
type 1 resp failure so incorrect

He is vitally stable so C only the correct.

168- suture materials carries a chance for disease transmission?

a silk
b nylon
c Vicryl
d catgut
169- which of the following tests is most helpful in differentiating asthma from copd?

a. Spirometry before and after an inhaled bronchodilator


b. serum IgE measurement
c. Chest X-ray
d. Sputum eosinophil count

170- 17y man complains of intense left flank pain that radiates to the groin, stone
passage, which he has experienced so many times since childhood, his uncle had the
same problem, urinalysis shows hexagonal crystals, the urinary cyanide nitroprusside
test is positive, cause of this condition?

a)amino acid transport abnormality


b)abnormality of uric acid metabolism
c)excessive intestinal reabsorption of oxalate
d) parathyroid adenoma

171- 1year after cs section, complicated by postpartum hemorrhage and inability to


produce breast milk, loss of appetite, cold intolerance and constipation, menses have
not resumed after delivery, thin hair with dry skin, fatigue, which of the following
explains her symptoms?

a Sheehan syndrome b
adrenal insufficiency c
anemia
d hypothyroidism

Naif: Hypopituitarism, Im not sure It could be A- Sheehan syndrome.


I’m looking for Sheehan Syndrome, not clear no more information in Amboss app about
Sheehan Syndrome.

Hypopituitarism: caused by many things


One of them Sheehan Syndrome.

⬛J
◆ Sheehan syndrome: postpartum necrosis of the pituitary gland. Usually occurs following
postpartum hemorrhage, but can also occur even without clinical evidence of hemorrhage. [1][2]

During pregnancy, hypertrophy of prolactin-producing regions increases the size of the


pituitary gland, making it very sensitive to ischemia.

Blood loss during delivery/postpartum hemorrhage → hypovolemia → vasospasm of hypophyseal


vessels → ischemia of the pituitary gland → empty sella turcica on imaging [3]
Aya: ⬛'

⬛'Abrar: pathoma book for preparing USMLE.

172- man coughing up bright red blood, fever, pleurtic chest pain and cough productive
of thick brown sputum, he started oral antibiotics for these symptoms 2 days ago but
has seen no improvement, the patient received allergenic stem cell translation for acute
myeloid leukemia 6 wks ago, which was complicated by acute graft-versus-host disease
and prolonged neutropenia,bp is 102/66. right sided crackles. Lab results: haematocrit
low, platelets low, leucocyte low, cxr reveals a dense right upper lobe infiltrate, which
has grown in size when compared to the X ray from 2 days ago. CT of the chest reveals
several nodular lesions with surrounding ground glass opacities in the right upper lobe.
Sputum gram stain shows inflammatory cells, diagnosis?

a. Aspergillosis
b. Candidiasis
c. cytomegalovirus infection
d. pulmonary tuberculosis

Answer:
according to up-to-date “invasive aspergillosis occurs most frequently in the setting of
immunosuppression associated with therapy for hematologic malignancies, hematopoietic cell
transplantation, or solid organ transplantation”

173- 0ld women with dizziness and fatigue and her symptoms began 8 hours prior, she
also complains of orthostasis and gradually worsening blurry vision, two days prior,
n/v and decreased oral intake, those symptoms have since resolved, her past medical
history is remarkable for chf, htn, Dyslipidemia and T2DM, she is on multiple
medications including digoxin and spiromclactone, lisinopril, carvedilol, metformin,
simvastatin and aspirin and her BP is 95/65mmhg, bradycardia secondary to a 2:1
atrioventricular nodal black. potassium little high, BUN= 55 creatinine high, glucose
high. which of the following precipitated her current condition?

a/ renal insufficiency.
b/ hyperkalemia.
c/ medication interaction.
d/myocardial ischemia.

174- man visits his family physician for the first time since being released from
prison, he presents a routine examination and states that he was diagnosed with
hepatitis in prison but he does not remember which letter, you check his medical
records and find out that he has hepatitis B, he states that he does not want his wife
to know about his sickness because he doesn’t want to worry her, you encourage him
to talk to his wife and explain that there is a risk of him infecting her, the patient is still
adamant that he does not want his wife to know. What’s the next step to take in this
situation?

a) call the local health department and report the patient’s hepatitis B status.
b) call the patient’s wife and warn her that she is at risk for hepatitis B. c
listen to the patient and maintain confidentiality.
d/encourage to your patient to receive marriage counseling A

175- Old woman with chf is complaining of nausea, anorexia, weakness, chromatopsia
and hallucinations?

a hyperkalema
b hypermagnesemia
c atrial dysrhythmia
d hyponatremia

A (NCBI)
176- 50 something Man has occasional red urine for 3 months, he states that his urine
stream appears normal first but turns red by the end of voiding, he has also noticed
small clots in his urine, no fever, edema, flank pain, weight loos, but bp is high and
urinalysis is positive only for blood, cause of this symptoms?

a_bladder diseases b
-urethral injury
c_ nephrolithiasis
d -glomerular disease

A (red by end of voiding, bladder)

177- PRIMIGRAVID, uterine bleeding, abdominal pain, small amount of blood in


the vaginal canal and the cervical os is closed, positive fetal cardiac activity,?

a. missed abortion b_
inventible abortion
c_threatened abortion.
d_incomplete abortion

C (cervical os closed and poitive fetal cardiac activity) (amboss)

178- 19 YO gained 4.5kg over the last year and believes it is related to her ocp, the
patient takes low dose Ethinyl estradiol norethindrone daily, Her pain sxs resolve after
3 months on the pills, appropriate advice for this patient?

a- reassure that the weight gain is not related to combined OCPs.


b. discontinue OCPS And star non-steroidal anti-inflammatory drugs. c_recommend
switching from combined OCPs to a copper intrauterine device. d- recommend
switching from combined ocps to progesterone only pills.

-why B? Is NSAID an alternative to COCs!

Abdulrazaq Alshammari:
(She has dysmenorrhea this is why she is taking COCP)

179- epigastric pain, diarrhea, started 8 months prior and only partially improved
with eating and use of omeprazole, his hx recurrent Nephrolithiasis, EGD shows
multiple ulcers throughout the transverse portion of the duodenum and jejunum,
diagnosis?

a gastrinoma.
b vipoma
c somatostatinoma
d Carcinoid tumor

(also known as Zollinger-Ellison syndrome.)


180- postmenopausal, breast tenderness, breast exam shows bilateral, diffuse
fibrocystic changes. The abdomen is distended and a large right lower quadrant mass
is palpable pelvic us reveals a 12cm complex right ovarian mass with solid components
and multiple septations, enlarged uterus, endometrial stripe. Which of the following
tumor is most likely to be elevated in this patient?

a,Estradiol
b,Alpha-fetoprotein
c,hCG.
d,Lactate dehydrogenase

181- 35yo woman presents to the emergency department in critical condition, she has a
dark rash on her legs, her vital signs are temperature 35.0c, heart rate 110, Bp 70/50
and respirations 22., the patient is admitted to the icu, blood cultures are obtained and
grow gram negative diplococcic,the patient remains hypotensive, which of the following
is the most likely cause of the patient’s condition?

a hyponatremia
b decreased T3 uptake c
pituitary infarct
d adrenal artery thrombosis

Suliman ..:
Most likely diagnosis is Waterhouse-Friderichsen Syndrome (WFS) which caused by N.
Meningitidis (gram negative diplococci).

answer is D

WFS —> Addisonian crisis


WFS —> DIC —> Adrenal artery thrombosis I
think so..

182- 55years old man preprogressively worsening abdominal pain for the last 6
months, the pain is aggravated with eating and associated with fatigue, joint pain and
weight loss. his CT angiography reveals bilateral renal infracts and several abdominal
arteries with aneurysms and distal abrupt cutoff?

a. fibromyalgia nodosa b.
polyartertitis nodosa.
c. bacterial endocarditis
d. thromboangiitis obliterans
183- 77y man inability to void; hx shows nocturnal urinary frequency w/ weak
stream, distended bladder. Dre shows enlarged, non-tender prostate gland?

a chronic bladder outlet obstruction. b


cystitis.
c renal failure.
d prostatitis.

A no sign of inflammation
184- 30 man gross hematuria which started around the same time as a now resolved
sore throat and a fever. BUN 9 Cr 1.0 .urine specimen shows 15-20 dysmorphic red
blood, +3 protein and 1-2 RBC casts.?

a. IgA nephropathy.
b. acute tubular necrosis.
c. goodpasture syndrome.
d. minimal change disease.

answer: A

185- chronic cough, hemoptysis, l disuse crackles, CL high,, BUN high and Cr
high, urinalysis reveals microscopic hematuria and 4+ proteinuria, which test
should be performed?

a. anti- glomerular basement membrane antibodies


b. anti-parietal cell antibodies
c. anti-mitochondrial antibodies
186- 32y F/U of breast biopsy that confirmed high-grade malignant, FM HX is
significant for ovarian cancer on her mother.
mutation in the BRCA2 gene, scheduled to begin chemo, best contraceptive option?

a copper IUD
b ocp
c condoms
d vaginal ring

A hormonal contraceptives contraindicated in breast cancer (amboss) success rate for copper IUD is
higher than condoms

187- Very very Long case of female 65y obese, weakness, SOB, been sleeping in
reclining chair due to dyspnea when she lies down, hx of HTN,T2DM, more than 2 years
not taking her medication and she smokes a lot and BP 156/90, moderate respiratory
distress and diaphoretic, her jugular veins are dilated, tachycardia with an audible S3,
crackles in the bilateral bases. There is bilateral 1+ pitting lower extremity edema up to
the knee. EKG shows increased QRS amplitude. normal troponin and CK-MB levels?

a. chf b,
copd
c. myocardial infraction
d. pericarditis
A
S3 , dilated jugular veins, orthopnea, pitting edema (amboss)
188- 45yo woman with fatigue and muscle weakness in both lower extremities. has
weakness and cramping in the legs after walking a short distance. HTN, which is treated
with lisinopril, and mildly decreased strength in the proximal muscles of the lower
extremities with sluggish ankle jerks bilaterally. Lab show normal erythrocyte
sedimentation rate and an elevated serum creatine kinase level. appropriate
management?

a. antinuclaear antibody test


b. serum TSH AND FREE T4 tests
c. electromyogram
d. muscle biopsy

‼️ The patient's symptoms of fatigue and muscle weakness in the lower extremities,
particularly with walking, suggest a possible neuromuscular disorder.

The elevated serum creatine kinase level is also consistent with a muscle disorder, although it is
nonspecific. The mildly decreased strength in the proximal muscles of the lower extremities and
sluggish ankle jerks bilaterally support a possible neuromuscular disorder.

189- Long case of 65y/o man with cirrhosis secondary to hepatitis C with fatigue and
confusion over the past week, his history shows numerous hospital admissions in
the past year for variceal bleeding and symptoms ascites, which necessitated several
therapeutic Paracentesis, he reports that over the past 2 days, he has been urinating
very little, he is jaundiced, distended abdomen and hepatomegaly, laboratory tests
reveals a BUN and creatinine is high. Which of the following is the most appropriate
treatment for this patient’s underlying disease?

a/ liver transplant
b/ paracentesis c/
hemodylasis
d/ kidney transplant

A
indications for liver transplant: the development of ascites, hepatic encephalopathy, or GI
bleeding in patients with cirrhosis

190- Case of female in 50s with worsening headache and right sided weakness, started
having it one month ago and since then has had continuous throbbing pain over the
left side of her head associated with nausea, more recently she has noticed increasing
difficulty using her right arm and leg, and bp is 150 and significant right sided pronator
drift. Also CT shows partially calcified round extra axial mass compressing the left
frontal lobe. The mass appears Dural based and homogenously enhances on post-
gadolinium MRI. management option?

a. surgical resection
b. combined anti- tuberculous therapy
c. whole brain radiation
d. full body scan
191- Long case of old man passing out while getting out of the bed in the morning. The
episode lasted a few minutes and the patient rapidly recovered consciousness, now he
has generalized weakness, he had decreased appetite and watery diarrhea for 2-3 days
before admission. The patient takes atorvastatin for hyperlipidemia. strong family
history of coronary artery disease, blood pressure is 120/70 when supine and
98/8something when standing, jugular veins are flat when the patient is in a siting
position. which of the following laboratory values is the most sensitive indicator of the
patient’s condition?
a decreased urine output
d decreased serum sodium c
decreased hematocrit
d decreased plasma renin activity

C or A not sure
192- 57y with hematemesis, mild varicosity of the distal esophagus and a bleeding
site recognized and cauterized. heavy drinker for the past 10 years, what caused
this?

a it’s probably a consequence of alcohol consumption. esophageal varices b I’m


not sure
c this happened because you are an alcoholic
d let’s get you better first then we can discuss your disease in greater detail.

193- at 1 P.M with acute onset of pain in his left lower extremity for the past 2 hours,
hypertension, chronic atrial fib. his left lower extremity appears pale and cold and no
pulses can be palpated. What is the latest time the patient should undergo surgery
in order to prevent irreversibly of his condition?

a. 1:30PM
b. 7:00PM
C. 2:30PM
D. 5:00PM

treatment within 6hrs

194- 24yo woman G1P1 was treated with isotretinoin for acne during the first trimester
of her pregnancy. Which of the following fetal systems could have been affected by
the treatment?

a. gastrointestinal
b. renal
c. pulmonary
d. central nervous system
(Microcephaly)

195- 65y man with sudden chest pain, tachycardia and dyspnea, pain as severe,
sharp, tearing pain in his chest that he feels in his back and he has a history of HTN.
X-ray shows a widened mediastinum?

a, aortic dissection
b,pericarditis
c, myocardial infraction
d, cardiomyopathy

196- 61yo woman presents complaining of severe neck pain, she was playing golf
earlier that morning and turned her head quickly after teeing off, experiencing sharp
pain in her neck and along her left arm, she is a strict vegetarian and what is the cause?

a. fracture of a cervical vertebra


b. left rhomboid spasm
c. herniation of cervical intervertebral disc.
d. dislocation of a cervical vertebra B

or C
197- woman on whom you performed a tubal ligation during the previous day reports
well this morning and says she is ready to go home. However, you suspect that you
left a surgical towel in the patient’s abdomen during the procedure. Closer examination
of the operative note and post-surgery imaging confirms your suspicion, what is the
most appropriate course of action?

a. inform the patient that you forgot to remove a towel before ending the procedure and
perform a second operation to remove it
b. contact the hospital’s legal team before taking further action
c. convince the patient that she needs to return to the operating room for another procedure and
remove the towel after she is sedated
d. do not inform the patient and allow her to return home to avoid the increased risks of a second
operation.

B maybe

198- 20s female healthy but on PE has a diffusely enlarged thyroid gland and
irregular pulse. initial appropriate action?

a_ primarily beta receptors b-


nicotinic receptors
c_ angiotensin receptors
d_ primarily alpha receptors

Beta blocker

Female + diffuse enlargement of thyroid gland + irregular pulse = A-fib

199- 2yo w/ 9 day Hx of fevers, diarrhea, bilateral conjunctivitis. several other children
at daycare have been ill. preauricular adenopathy. how could this have been prevented?

a better hand washing practices


b avoidance of inappropriate antibiotics c
bavoidance of undercooked seafood d
appropriate immunization

adenovirus infection
200- recurring diaper rash, white curd-like substance covering he oral cavity.
A cutaneous injection of candidiasis antigens produces no reaction at 48
hours.

a. Chronic mucocutaneous candidiasis


b. Wiskott-Aldrich syndrome
C. Leukocyte adhesion deficiency
d. hyper IgM syndrome

maybe a

—————
Bilateral hydrosalpinx

Naif: https://www.google.com/search?
q=Bilateral+hydrosalpinx&client=safari&hl=ar-
sa&prmd=ivn&source=lnms&tbm=isch&sa=X&ved=2ahUKEwjtz5WS8dj-
AhUmTaQEHUQNBMQQ_AUoAXoECAIQAQ&biw=428&bih=738&dpr=3#imgrc=a374SJGPY
AZjvM&imgdii=d8CMaexulZCDVM&lnspr=W10=

—————
—————

201- 12yo boy patient is terminally ill with osteosarcoma, he asks you how long he is
likely to survive, his parents have asked you in the past to avoid discouraging their son
by withholding the true severity of his condition. What is the best course of action?

a. discuss the patient’s condition honestly without providing an estimated survival time.
b. explain that a positive attitude will improve life expectancy and that he should not worry about
his expected longevity while concentrating on getting well.
c. provide an estimate of survival time based on a literature review.
d. refer this question to the patient’s parents.

🌐 I think B is the better choice because it solves the problems of the parents (avoid discouraging)
while also helping the child redirect his attention towards getting better, which factually improves
life expectancy. It’s also a suitable answer to give a child.
D doesn’t make sense since the parents are not medical professionals to estimate survival rate. C
ignores the parents request. A is the closest after B in my opinion. I believe the best answer would be
to discuss the issue with all of them and come to an understanding, but this isn’t one of the choices.

202- 92yo woman with likely osteomyelitis and sepsis, she was diagnosed with
metastatic squamous cell lung cancer a month prior to admission, The patient will
probably require intubation to survive but she has a living will dated 3 weeks ago that
states she does not want any extraordinary measure taken to prolong life. Her
daughter just arrived and requests to have everything done for the patient, what is the
most appropriate course of action?

a. meet with the patient’s daughter and any other family members to discuss their rationale for
ignoring her wishes about terminal care.
b. transfer the patient back to the nursing home.
c. inform the patient’s daughter that her mother has a living will and that you will not intubate her to
prolong her life.
d. intubate the patient and transfer to ICU.

◆ C is the only option which considers the patient’s autonomy, which is essential in an ethical
question. Also, the recall does not mention anything that would make her incompetent to make this
decision for herself, so it should be respected and followed.

203- woman with amenorrhea, pregnancy test that was positive, History is remarkable
for hypertension, for which she was prescribed lisinopril but was not compliant with
treatment, blood pressure is 168/108. Work up confirms pregnancy. management?

a. educate patients about the importance of normalizing BP and prescribe nicardipine instead of
lisinopril.
b. reassure the patient and continue current care.
c. educate patient about the importance of normalizing BP and prescripe propranolol instead of
lisinopril.
d. educate patient about the importance of normalizing BP and prescribe losartan instead of
lisinorpil

🌐The HTN is clearly caused by patient non-compliance, so the best course of action would be to
educate and encourage her to take the same medications. No indication to change meds.

204- chronic non-productive cough worse in the evening and often awakes him from
sleep. mid-epigastric gnawing discomfort and occasional regurgitation of
undigested food, obese eats dinner at home before going to sleep. The
pathophysiology?

a. decreased lower esophageal sphincter tone.


b. increased lower esophageal sphincter tone
c. disordered peristalsis d
infection with H. pylori
AMBOSS: GERD : Reflux is primarily caused by an inappropriate, transient relaxation of the
lower esophageal sphincter (LES). Risk factors include obesity, stress, certain eating habits (e.g.,
heavy meals or lying down shortly after eating), and changes in the anatomy of the
esophagogastric junction (e.g., hiatal hernia).
🔆Typical symptoms: Retrosternal burning pain (heartburn), Regurgitation
🔆Extraesophageal symptoms: Chronic nonproductive cough and nighttime cough,
Hoarseness , Bronchospasm, Dental erosion
🔆Aggravating factors: Lying down shortly after meals , Certain foods/beverages

205- dizziness; she was unable to get out of bed and go to the bathroom, this morning
and has had a headache and nausea. The patient has a history of migraine headaches
and hypertension. Six months ago, she had an episode of transient right sided vision
loss and patient moves all extremities and deep tendon reflexes are symmetric, CT scan
of the head is shown in the image.she appears lethargic,next step in management of
this patient?

a. lumbar puncture
b. surgery
c. Loop diuretics
d. intravenous corticosteroids

🌐Assuming the diagnosis is Space occupying lesion, the answer could be D. Note question is asking
for next step, not best. The CT would give more details.
Management of Brain tumor from AMBOSS:
🔆Principles of treatment- Management of primary brain tumor: Maximal safe surgical
resection,
Radiation therapy: e.g., standard fractionated external beam radiation therapy,
Systemic therapy: e.g., chemotherapy, cancer immunotherapy, Offer clinical trial enrollment and
consider palliative care for patients with high-grade tumors (e.g., glioblastoma multiforme). Patients
with brain tumors are at increased risk of intracerebral hemorrhage; use shared decision-making for
anticoagulation treatment decisions.
Seizures: Primary prophylaxis is not recommended.
Secondary prophylaxis (i.e., history of seizures): Initiate treatment of epileptic seizures.
Vasogenic cerebral edema: Administer glucocorticoids (e.g., dexamethasone)
. —————

‫ الصور‬2

ECG
1 failure to thrive
1 moluscum contagiousum
‫ وكلهم‬pediatric in pneumonia lobar 1
‫اسم التشخيص او منجمنت‬

‫ بالنسبه الصورتين‬ECG
‫األولى وحدة جات مريضة عندها‬
MD HTN OLD AGE WITH ANGINA PAIN
‫ حطوا الدوية ا لي تستخدمها وطلبوا‬MANAGMENT INITIAL

:..Silent ‫ والثانية‬failure heart ‫وجابوا لستة ادوية وايش الدوا الناقص نضيفة لوصفة‬
‫اغلب ح الت لاا طفال و العنف ضد الطفال و الهمال جات في الطوارئ‬
tum ‫ والنساء جات في الطواريء وفي عيادة الرعاية الولية‬, molar , convulsions , PROM , adenocarcinoma vaccination, , bleeding , pregnant
or

‫ال على سؤال كم جا‬CONCENT for termination of pregnancy, tubal ligation in CS multiparty , , resuscitation of
DNR

‫ عن اسئلة جات‬antibiotics in gynecologist disease, vaginal discharge, pregnancy UTI IN

PEDIATRIC ‫مكررة اسئلة منها جات كمان‬

—————

206- Female 21? Years did cervical screening for last 5yrs and it was normal, now
she did it again and was normal, asking about when is the next time for screening

A: 6months
B: 1yr
C: 2yrs
D: 3yrs

AMBOSS: Cervical cancer - Papanicolaou test (Pap smear/cervical cytology): Screening interval:
Perform every 3 years if done alone
Perform every 5 years as co-testing

207- Pt hyperlipidimea took medication then developed flush, the Dr gave him
aspirin then it resolved, asking what is the medication he took:

answer is niacin

AMBOSS: Niacin- Mechanism of action: inhibits lipolysis and fatty acid release in adipose tissue
🔆Indication: high LDL cholesterol and lipoprotein(a) levels (> 50 mg/dL) despite statin
and ezetimibe therapy (or if statins are contraindicated)
🔆Adverse effects: Flushing and pruritus, Hyperglycemia, Hyperuricemia and gout (e.g.,
podagra), Paresthesias, GI upset, ↑ LFTs
🔆Contraindications: Liver failure, Gout, Hemorrhage, Gastric ulcer, Cardiovascular
instability
208- A lady came to clinic with her 6month child, asking what can he do at this age

A: turn from prone to supine


B: sit without support

🌐See the pictures below. ‘Rolls from prone to supine’ is at 6 months, ‘sits without support’ is at 7
months according to one table, and 6 according to the other one. From these 2 perhaps A is more
accurate.
209- Typical scenario of thoracic aorta rupture

AMBOSS: Thoracic aortic aneurysm rupture


🔆Risk factors :Large aneurysm diameter, Rapid aneurysm expansion, Trauma, Smoking
🔆Clinical features:Contained rupture -
Severe chest pain, Possible abdominal pain, often hemodynamically stable.
Free rupture - Possible loss of consciousness, Severe chest and possible abdominal pain, Hypotension
🔆Treatment: initial stabilization, emergency surgical repair.

210- Typical scenario of perforated esophagus case (chest pain after forceful

vomiting) AMBOSS: Esophageal Perforation-


🔆Clinical features: Mackler triad (esp. in Boerhaave syndrome)- Vomiting and/or retching, Severe
retrosternal pain that often radiates to the back, Subcutaneous or mediastinal emphysema: crepitus in
the suprasternal notch and neck region or crunching/crackling sound on chest auscultation (Hamman
sign)
🔆Imaging: Chest x-ray posteroanterior and lateral, upright AXR- Widened mediastinum,
Pneumomediastinum, pneumothorax, pneumoperitoneum, subcutaneous emphysema, Pleural effusion
Confirmatory tests: Contrast esophagography (gold standard): Contrast leak reveals the location
and size of the rupture.
CT chest and CT esophagography (with oral contrast)
🔆Treatment : initial: ABCDE survey, NPO, Broad Spectrum IV Antibiotics, Chest tube
placement, Parenteral analgesia, Urgent thoracic surgery and GI consult.

211- Duodenal perforation case (high spead mva with seat belt sign)

Radiopedia: The seatbelt sign is both a clinical and radiological sign. It is simply the presence of
ecchymosis and/or abraded skin in the distribution of a seatbelt (i.e. horizontal and/or
diagonal) extending across the abdomen evident after a motor vehicle accident.

🔆Epidemiology: A positive abdominal seatbelt sign, in combination with abdominal pain or


tenderness, results in a higher likelihood of intra-abdominal injuries; some cohorts
demonstrate a higher incidence of solid organ injuries 6 and others bowel/mesenteric
injury. There exists little consensus regarding the significance of the specific (i.e.
ecchymosis/abrasion) superficial injury present as it pertains to underlying injuries. CT of the
abdomen/pelvis is highly sensitive in these patients for identifying significant intraabdominal injuries
requiring exploratory laparotomy.

Similar recall (Earth - August 2021)


https://t.me/c/1243365384/3894
212- Patient came with dnr from another hospital

A: do not resuscitate
B: from another hospital is only valid for 24hrs C:
it is not valid
D: "I don't remember"

in general, a DNR order from another hospital or healthcare facility is valid and should be honored by
the receiving facility, but it is important to confirm that the order is appropriate for the patient's
current clinical situation and to follow any applicable laws or regulations.

Agree with the above explanation.

Similar recall with explanation: https://t.me/SMLEsecrets/1571

213- Primary dysmenorrhoea symptoms and resolve after 3days,asking about first
line management

NSAID

Wafa:
Similar recall: https://t.me/WafaOBGYN/840
📌Note to Remember (ACOG)
Management of Primary Dysmenorrhea
- NSAID (first line)
- OCP (second line, if a trial of NSAIDs does not provide adequate relief of dysmenorrhea
symptoms)
214- Typical case of adenomyosis

There is he of pelvic surgery sx dysmenorrhea

WAFA:
Similar recall: https://t.me/WafaOBGYN/289
📝📌Note to remember
Adenomyosis
Risk factors:
- Previous uterine surgery, C/S, D&C
- Childbirth
- Middle age Clinical
presentation:
- Commonly report symptoms similar to those reported by patients with (ddx) endometriosis.
- Common complaints include menorrhagia, dysmenorrhea, metrorrhagia, chronic pelvic pain and
dyspareunia
Physical Examination:
- Enlarged and tender uterus!!

215- Malaria parasite picture, asking for diagnosis

Malaria falciparum on blood smear:

216- Patient after rta can't put fork in her mouth, which part of brain affected

A: cerebellum
B: parietal`a C:
occipital
D: I don't remember

AMBOSS: Cerebellum functions- coordination.

217- Asking about the structure of gout crystals in the microscope

Needle shaped
They are needle-shaped and have a strong negative birefringence when viewed under polarized
light.

is a hallmark of gout crystals.

AMBOSS:
Gout -
🔆 Diagnostics: Arthrocentesis and synovial fluid analysis
Synovial fluid analysis is the gold standard for diagnosing gout.
Indications: Uncertain clinical diagnosis, Higher probability of septic arthritis
Characteristic findings: Polarized light microscopy: needle-shaped monosodium urate
crystals that are negatively birefringent
Synovial fluid cell count: WBC > 2000/μL with > 50% neutrophils

218- Pt with epilepsy came with shoulder dislocation, which is most common type

A: anterior subacromial
B: anterior "I don't remember"
C: posterior

AMBOSS: Seizures and Epilepsy.


🔆Complications:
Acute- Hyperthermia, cardiorespiratory deficits, and excitatory toxicity, which can cause
irreversible tissue damage, especially to the CNS and, in turn, increase the risk of further seizures
Postictal transient anion gap metabolic acidosis
Physical trauma: Tongue biting, Posterior dislocation of the glenohumeral joint due to
falling
Status epilepticus

219- Patient with AML, 2 days post chemo had some symptoms "side effects",
labs show hypocalcemia hyperkalemia hyperphosphatemia hyperuricemia
Dx: tumor lysis syndrome

AMBOSS:
Tumor lysis syndrome
🔆Definition: A potentially life-threatening oncologic emergency resulting from the rapid destruction
of tumor cells, which leads to a massive release of intracellular components, e.g., potassium (K+),
phosphate (PO43-), and uric acid, that can damage the kidneys and cause renal failure.
🔆Etiology: TLS most commonly occurs after initiating cytotoxic treatment in patients with
hematologic malignancies (e.g., ALL, AML, or NHL).
Can also manifest unrelated to therapy in patients with a very high tumor burden
🔆Pathophysiology:
hyperuricemia → urate nephropathy and risk of acute kidney injury
hypocalcemia → risk of seizures
Hyperphosphatemia: calcium phosphate crystals obstruct renal tubules → acute kidney injury
Hyperkalemia: → risk of cardiac arrhythmias
Think of “PUKE calcium” to remember the electrolytes affected in tumor lysis
syndrome: Phosphorus, Uric acid, and potassium (K+) are Elevated; Calcium is
decreased.

220- Pt hba1c 5.8 fasting sugar higher than normal range, on metformin 500mg

A: reassure
B: increase dose of metformin C:
insulin
D: add another medication I forgot

AMBOSS:
Diabetes mellitus : Management
A target of HbA1c < 7% is generally suitable for most nonpregnant adults.

221- Pt with retrosternal chest pain and heaviness but he noticed intensity of
pain changes as he change his position

A: reassure and follow up after 1 week B:


nsaid
C: troponin

AMBOSS:
🔆Clinical features: Acute pericarditis-Pleuritic chest pain. Acute, sharp retrosternal pain
caused by inflammation of the parietal pleura. Typically aggravated by coughing, swallowing, or deep
inspiration
Other causes of pleuritic chest pain include pulmonary embolism, myocardial infarction, and
pneumothorax.
Improves on sitting and leaning forward
Can radiate to the neck and shoulders (most commonly to the left side)
Pericardial friction rub: high-pitched scratching on auscultation: Best heard over the left
sternal border during expiration while the patient is sitting up and leaning forward.
Present in 85% of patients with acute pericarditis.
Pericardial effusion
Low-grade intermittent fever

Acute pericarditis classically presents with sharp, retrosternal chest pain that is
exacerbated by deep inspiration and lessened by sitting up or leaning forward.

🔆Treatment: The mainstays of therapy include anti-inflammatories


Medical therapy: Acute pericarditis is often self-limited but NSAIDs can alleviate symptoms
and prevent a recurrence. Consider anti-inflammatory therapy also for chronic pericarditis
(transient constrictive pericarditis may respond).
Pharmacotherapy: NSAID therapy (Aspirin, Ibuprofen, Indomethacin). Consider colchicine in
combination with NSAIDs or as a monotherapy.
Only consider prednisone in severe cases or in pericarditis caused by uremia, connective tissue
disease, or autoreactivity.

222- 60yr female with right breast non tender mass and no clear margins and
skin tethering

A: fibroadenoma
B: fibrocystic changes
C: carcinoma
D: ductal ectasia

AMBOSS
Breast cancer: Clinical features-
🔆Early stages- In early stages, affected individuals may notice a palpable mass with the
following characteristics: Typically single, nontender, and firm, Poorly defined margins,
Most commonly located in the upper outer quadrant (∼ 55%)
🔆Locally advanced disease- Morphology: changes in size and/or shape → asymmetric
breasts
Skin- Retractions or dimpling, Peau d'orange
Nipple- Inversion
Blood-tinged discharge
🔆Progressive disease- Ulcerations, Edema of the arm, Paget disease of the nipple
Signs of metastatic disease
Lymphatic spread- Lymphadenopathy
Nontender, firm, enlarged lymph nodes (> 1 cm in size), that are fixed to the skin or
surrounding tissue
Most commonly the axillary nodes and, in later stages, the supraclavicular and/or
infraclavicular nodes
Hematogenous spread-
Bone metastasis: Bone pain, Pathologic fractures, Spinal compression Liver
metastasis: Abdominal pain, distention, Nausea, Jaundice
Lung metastasis: Cough, Hemoptysis, Dyspnea, Chest pain
Brain metastasis: Headaches, Seizures, Cognitive deficits, focal neurological deficits

223- Pediatric patient came for dtap vaccination, his mom reports that his last
vaccination he developed severe leg edema and rash and shortness of breathe they had
to take him to ER

A: give vaccine
B: wait for "??" week
C: check his vaccine history
D: don't give vaccine

C or D
◆ Vaccine history needs to be checked in case the vaccine that caused the first reaction is not
mentioned (like this recall). But if the question clearly mentions that the previous vaccine causing the
reaction was the same (DTaP), then the answer is D - don’t give vaccine.
Anaphylaxis is a contraindication to receive the same vaccine.

224- Pt after thyroid surgery developed hoarseness and sob

A: bedside exploration
B: take to OR
C: intubate

Dr. Abeidi:
This is a wound hematoma causing extrinsic compression on upper airway.
Treatment is BEDSIDE evacuation of hematoma by opening the wound THEN taking the
patient to OR to achieve hemostasis.
So the answer must be: Bedside evacuation of hematoma by opening the wound

Similar recalls:
FAIR: https://t.me/dec2020q/519
SMLE-B: https://t.me/smlemay/2603 https://t.me/smlemay/2604
https://t.me/smlemay/3438
Earth: https://t.me/c/1243365384/3425

225- Female pt with frothy green vaginal discharge and itchy, what to give to her partner

A: No need B:
ceftriaxone
C:
metronidazole⬛✓
D: fluoroquinolone

Wafa:J⬛
Similar recall:
1- https://t.me/WafaOBGYN/1860
2- https://t.me/WafaOBGYN/1862
3- https://t.me/WafaOBGYN/1864
)c’ Trichomoniasis:
- Clinical presentation: Frothy, yellow-green discharge. Foul-smelling, vaginal itchiness,
strawberry cervix
- Microscope: Flagellated protozoa
- Treatment: Metronidazole, Treat Sexual partner(s)

226- Female with fishy odor discharge

A: bacterial vaginosis
B: candida
C: N gonorrhea

Wafa:⬛J
Similar recall:
1- https://t.me/WafaOBGYN/1876
2- https://t.me/WafaOBGYN/1874

Bacterial vaginosis:
- Clinical presentation: Gray/milky discharge. Fishy odor
- Microscope: Clue cells
- Treatment: Metronidazole. ✅

227- 5mm nuchal translucency, which is most common associated with

A: cardiac malformatio
B: neural tube defect

WAFA:
Pregnant of twins, one has increased nuchal translucency in Ultrasound. What will he have?
A. Congenital cardiac malformation ✅ ( all chromosomal syndromes have cardiac disease)
B. Turner syndrome
C. Neural tube defect
https://t.me/WafaOBGYN/54

Similar recalls:
SMLE killers : https://t.me/SmleKillers/2399
Earth: https://t.me/c/1243365384/4653
228- 58yr had rif pain and tenderness and diagnosed with appendicitis, and treated
conservatively

A: reassure
B: open appendectomy in 3 months C:
lap appendectomy in 2 week

-Interval colonoscopy would be better answer according to his age

◆ Better recall from SMLE with Muath. :

58 year old man with appendicitis was treated conservatively with antibiotics. He now presents with an
appendicular mass with no collection. How will you manage this case?
A. Interval laparoscopic appendectomy after 12 weeks
B. Interval open appendectomy after 12 weeks
C. No further intervention needed
D. Colonoscopy after 6 weeks ⬛✓

Young year old man with appendicitis was treated conservatively with antibiotics. He now
presents with an appendicular mass with no collection. How will you manage this case?
A. Interval laparoscopic appendectomy after 12 weeks ⬛✓⬛
B. Interval open appendectomy after 12 weeks
C. No further intervention needed
D. Colonoscopy after 6 weeks

Patient had appendiceal mass he was treated non-operatively and discharged home with
antibiotics, next step in management?
A. No further intervention
B. Open appendectomy after 2 weeks
C. Laparoscopic appendectomy after weeks (I forget how many week) ⬛✓

6-8 weeks or 12 weeks in another recall


https://t.me/JANISHERESMLE/335

Other recalls:
SMLE surgery: https://t.me/SMLE_Surgery/815
SMLE by Hamza: https://t.me/SmleByHamza/363

229- Old patient in icu and developed pneumonia and treated accordingly with abx for
12days, now suffering RUQ pain with tenderness, ultrasound shows acalcular
Cholecystitis

A: ercp drainage
B: urgent open cholecystectomy C:
conservative
D: cholecystostomy tube

🌐 Better recall from Quick recall channel:


Elderly pt. admiaed to ICU with acute MI, and developed pneumonia on tazocin, he also has RUQ
pain and tenderness. Management?
A- ERCP drainage
B- emergency cholecystectomy
C- convert from tazocin to meropenem
D- us guided cholecystostomy drainage

Abdulrahman: The answer is D. This patient most likely has acalculous cholecystitis. This
typically occurs in critically ill patients due to a combination of factors (e.g., bile stasis and
hypoperfusion). ERCP drainage and emergency cholecystectomy are not indicated in this case due
to the patient's inability to undergo surgery (elderly, ICU, and has pneumonia). Switching
antibiotics will not do anything in this case.
https://t.me/QuickRecall/308
Similar Recalls: https://t.me/c/1222499606/206
https://t.me/smle22olds/1300
https://t.me/Abeidi_Review/2924
https://t.me/c/1818479763/696

230- Pt came with amenorrhea for 6weeks . Ultrasound shows empty uterus and sac
in right side Hcg 4500 (she is vitally stable)

A: ruptured ectopic pregnancy B:


ectopic pregnancy
C: normal pregnancy D:
threatened abortion

◆ A would not be vitally stable. C and D will not show empty uterus.
WAFA:
Female with amenorrhea for 6 weeks, pregnancy test positive, presented with abdominal pain, (can't
recall if there was bleeding or not), US showed no intrauterine pregnancy with minimal fluid in cul-de-
sac and mass of 1.2 cm in tube. She was vitally stable. What's your action?
📌 According to Williams Obstetrics
- A small amount of peritoneal fluid is physiologically normal.
📌According to UTD
- A small amount of clear free fluid in the pelvis is a normal sonographic finding. There is no
established threshold for the volume of fluid that is physiologic and the volume is difficult to measure
sonographically. However, fluid that is anechoic and isolated to the pelvic cul-de-sac and adjacent
recesses is likely physiologic
- The presence or absence of peritoneal free fluid is not a reliable indicator of whether an ectopic
pregnancy has ruptured
**if large amount of fluid (indicates hemoperitoneum=ruptured ectopic pregnancy)—>
salpingectomy.

231- important Q⬛J


‫‪ C‬الجابه هنا‬
‫⬛‪J‬شوفو البروتش‬
232- Pt post gastric sleeve 3 days complains of fever, no abdominal pain complaint

A: intra-abdominal abscess B:
chest infection
C: UTI
D: wound infection
C - 3 days post op and also no abdominal pain.

233- WHO surgical checklist safety

A: before admission, before skin incision, at discharge


B: before induction of anesthesia, before skin incision, before pt leave OR C:
before admission, before induction of anesthesia, before pt leave OR D: before
admission, before induction of anesthesia, at discharge

https://cdn.who.int/media/images/default-source/ihs/patient-safety/safe-surgery/sssl-checklist- front-
thumb.gif?sfvrsn=246d8b0d_2

234- A pt with shock, which will decrease?

A: urine output
B: pulse pressure
C: respiratory rate

Ahmed: A is more convincing than B

235- Child came with toxication symptoms after ingesting a large amount of family
member medication (didn't mention name of drug, nature of symptoms, nor time
of ingestion)

A: activated charcoal
B: gastric lavage
C: ipecac syrup
236- Female pt came 15 days post delivery with bleeding and fever

A: endometritis
B: wound infection C:
retained placenta D:
mastitis

237- Pt k/c dm and HTN present with medial foot ulcer with discharge, popliteal
and distal pulses are intact

A: CT with contrast
B: CTA
C: MRA
D: Venous duplex

238- 7yrs child his mother complains of poor training to bathroom which muscles
is targeted in therapy

A Perianal
B Pelvic floor
C Rectus muscle
D Detrusor

comment: ??

‫هذا السؤال الحظت اإلجابات بالكومنت نفس الشي‬

Target of therapy —> pelvic floor


Affect muscles —> Detrusor

‫ يوافق على‬AI chat ‫حتى‬


‫اإلجابة شوفو شرحه إللجابة‬

The muscles targeted in therapy for improving bathroom training in children are primarily the pelvic
floor muscles. The pelvic floor muscles play an important role in controlling bowel and bladder
function, and it is common for children to have difficulty with toilet training if these muscles are
weak or not functioning properly. Strengthening exercises for the pelvic floor muscles can be helpful
in improving bladder and bowel control in children.
239- 21yr female 9w pregnancy came with abdominal pain, on examination uterus is
5 weeks and absent fetal cardiac activity.

A: misoprostol
B: mifepristone
C: expectant management D:
D&C

)’ cNaif:

Dx is Missed abortion ✓

✓ Answer is C- expectant management ( A management strategy that involves serial clinical


monitoring (i.e., intermittent screening for symptoms of disease).

◆ Missed abortion:
• Death of a fetus before 20 weeks' gestation without expulsion of any POC
• No bleeding
• No expulsion of the products of conception
• No fetal activity
• Closed cervical os

◆ Ttt:
• Depends mostly on patient preference.
• Expectant management (option for women < 14 weeks gestation)
• Medical evacuation: combination of mifepristone and misoprostol
• Surgical evacuation (D&C): if spontaneous evacuation does not occur after 4 weeks or in cases of
septic abortion or heavy bleeding

240- 20days baby with fever and poor feeding and decreased movement

A: admit and observe and discharge as soon as fever subsides B:


admit and do full workup and start antibiotics
C: reassure and follow up
241- child with fever, dark urine, irritability. O/E: tender abdomen with no
organomegaly. Investigation WBC 16000, urinalysis show proteinuria +2 and
erythrocyte 18.

A furosemide
B ceftriaxone
C salt restriction
D steroid

242- Pt with history of haematemesis did OGD and showed multiple gastric ulcers.
What to order

A: fasting gastrin level


B: acid ph
C: I don't remember

243- Patient did a right inguinal hernia repair month ago, now presenting with
a shrunken right testicle. What is the cause of his presentation?

A. Mesh prolapsed into the scrotum


B. Thrombosis of the pampiniform plexus
C. Tight external ring of the inguinal canal D.
Testicular artery ligation

Ahmed (According to UTD): interruption of testicles’ blood supply which typically occurs after
transection of indirect hernia from cord structures, can lead to ischemic orchitis and in turn, testicular
atrophy.

comments:

● Dr Muhammad Alamzeb Khan:

Both B and D

● Majed Almalki:

D
Injury to pampiniform plexus > ischemic orchitis within 1 week
Injury to testicular artery > testicular atrophy in protracted time

Ischemic orchitis manifest as enlarged painful testes

● Bader Alsawadi:

B > complications of testicular infarction


—————

—————

244- Child with fever and sore throat, and there’s exudate membrane cover the
tonsils, and neck lymphadenopathy

A: Coronavirus
B: H Influenza
C: RSV
D: mononucleosis
245- Pregnant woman 32 week came to clinic for follow up, her blood tests came back
as rubella antibody negative (reference value written have to be positive). What to
do?

A: give anti D Immunogloblin


B: check rubella antibodies C:
give rubella vaccine
D: reassure and follow up in 2 weeks

Ahmed: choices are not clear.


A - There is no hematological indication to give Anti-D Ig B -
Antibodies were just checked, why check them again?
C - It's contraindicated to give rubella vaccine during pregnancy
D - According to UTD: risks of congenital defects after maternal infection with Rubella are limited to
the first 16 weeks of gestation. “Little, if any, risk of Congenital Rubella Syndrome is associated with
infection after 20 weeks’ gestation.’’

246- Pt with ascites and edema, severe muscle wasting, hyperkeratosis, lab
show: decreased glucose, decreased albumin

A. severe protein deficiency ✓⬛


B. severe carbs deficiency
C-Rickets
D-Mild malnutrition

Naif: kwashiorkor✅
Similar recall:

Pt with ascites and edema, severe muscle wasting, hyperkeratosis, lab show: slightly
decreased glucose, significantly decreased albumin,
A. (kwashiorkor)⬛✓
B. (marasmus)
C-Rickets
D-Mild malnutrition

2 year old skinny, loss of fat, muscle wasting and NO EDEMA?


A.kwashiorkor
B.Marasmus⬛

247- A child presented to your clinic with his mother for regular follow up, the
only new complaint is epistaxis more than usual of his brothers.
His past medical history reveals atopic dermatitis treated with topical
steroid and multiple hospital admission due to chest infection

A) Wiskott-Aldrich syndrome.
B) Idiopathic thrombocytopenia purpura.
C) Kartagener syndrome.
D) Cystic fibrosis.
E) Drugs effect.

Naif: A) Wiskott-Aldrich syndrome.⬛✓


248- Pregnant Pt 29w came with BP 180/110 and proteinuria then suddenly she
started seizing

A: tonic clonic seizures


B: preeclampsia
C: eclampsia⬛✓⬛

Naif: Eclampsia: new-onset seizures (tonic-clonic, focal, or multifocal) in the absence of other
causes E.g., epilepsy, ischemic or hemorrhagic stroke, drug use ; a convulsive manifestation of
hypertensive pregnancy disorders.

249- Pt with rickets, asking which vitamin is deficient

A: D1
B: D2
C: D3
D: D4

Naif:
‫بحث في مصادر خارجية ومافيه شي محدد والنوعين الساسيه هي‬
D2&D3

‫ اقرب‬D1&4 ⬛J ‫تختلف بالتركيب الكيميائي‬


‫اجابه وهلال اعلم‬
*Cholecalciferol*, also known as vitamin D3 and colecalciferol, is a type of vitamin D that is made
from cholesterol by the skin when exposed to sunlight; it is found in some foods and can be taken as a
dietary supplement.[1]

D3 : Biological Active
‫هذا اللي فهمته‬

Amboss ⬛J
Vitamin D3 is active form
Vitamin D3 deficiency
⬛J⬛ In adults:
*osteomalacia* In
children: *rickets*
Symptoms of hypocalcemia (e.g., *tetany*)
Neurological manifestations [13][14][15][1]
Tetany: increased neuromuscular excitability (when caused by respiratory alkalosis =
hyperventilation-induced tetany)

250- A woman at 10 weeks gestation comes to the office for an initial prenatal visit, the
patient has no chronic medical conditions and no prior surgeries, while discussing
what to expect during the pregnancy and delivery, the patient states that she would like
a primary cesarean delivery performed, the patient’s mother has struggled for years
with urinary and fecal incontinence after having 3 vaginal deliveries and the patient
does not want the same complications, the physician discusses the complications
associated with cesarean delivery. Including the increased risk of complications in
future pregnancies, based on the lack of medical and Obstetric indications, the
physician is uncomfortable agreeing to perform the procedure, however the patient
states that she understands the risk and still wants a cesarean delivery, which of the
following is the most appropriate management of this patient?

A) Refer the patient to another obstetrician-gynecologist B)


Perform a cesarean delivery on maternal request
C) Tell the patient the procedure will not be covered by her insurance
D) Wait and discuss further when the patient is in active labor

Ahmed: CS delivery on patient’s request is a well known and prevalent cause of CS


worldwide.
According to UTD: Performing a cesarean birth on patient request in a well-informed patient is
considered medically and ethically acceptable.

The dilemma in the question is regarding the statement: “the physician is uncomfortable
agreeing to perform the procedure”, So it could be A?

251- Elderly diabetic underwent major upper abdominal surgery 72h ago.
Temperature: 38
Oxygen saturation: 87%

What is the most likely diagnosis?

A. Pneumonia
B. Basal atelectasis I’m not sure
C. Pulmonary embolism
D. Sub-phrenic collection

Why??!! Oxygen saturation: 87%

If fever in first day after postoperative answer B


After 3 day after postoperative UTI
After 5 day dvt , PE
After 7 day wound infection
More than 7 day drug indice

Naif:

Similar recall:
Q: Patient had fever in the morning after he went through a surgery, what’s your
diagnosis?
a) Atelectasis
b) Wound infection
c) DVT
d) UTI

Comment:

Naif: Atelectasis: 72 hours postoperative


-Hypoxemia, potentially low PaCO2, and respiratory alkalosis

surgery of ‫اول شي الزم نعرف‬


type
‫والمدة اللي ظهرت فيها العراض‬
In general B- wound infection probably

- postoperative UTI: fever + urinary symptoms

Also UTI depends on site of surgery, where ? Or


*maybe pt do urethral catheter *

In this case No signs of DVT or Atelectasis

PE typically with chest pain with Dyspnea ، Tachycardia and tachypnea with signs of DVT
Atelectasis: 72 hours postoperative
-Hypoxemia, potentially low PaCO2, and respiratory alkalosis
-Chest x-ray and CT: evidence of lobar collapse
◦ Large number of affected alveoli or rapid onset → acute dyspnea, chest pain, tachypnea,
tachycardia, and cyanosis
• Dull percussion note, diminished breath sounds, and decreased fremitus over the affected lung
• Possibly tracheal deviation towards the side of lesion

Similar recall:
Elderly diabetic pt after ( i think major ) abdominal surgery ( by 72 ) hours complain abdominal
tenderness or chest pain
Temp 38
Pulse 110 not sure
Scenario just like this no more

1 atelectasis⬛✓⬛
2 pneumonia
3 PE ⬛✓⬛
4 sub phrenic collection

—————

—————
—————

252- ◆Correction of following question◆

A woman got abused by her husband, she refuses to call the police. What is the most
appropiate action?

A) Respect her wishes ‫صح‬


B) Educate her about violence ‫صح اجابه اقرب‬
)husband her to Talk C ‫مالي دخل اكلم زوجها‬
D) Ignore her and report the case to the police ‫صح‬

Same recall: https://t.me/smlebrain/222

Naif :
.‫ صح تهديد بالقتل ااذ هي قدمت بالغ‬D ‫ممكن رافضه أ لنها مبتزة وتكون‬
A‫صح‬
‫ كل ن‬B ‫اشوفها كا ثر صحه‬
‫ اجابه دب ون مرجع‬, ‫هذا مجرد تحليل مني‬

‫لكن اذكر في‬


Amboss
‫ بإبالغ الجهه المسؤله‬: ‫ قي ول ا لزم تقدم لها الرعاية الصحيه كامله مث تستشيرها على انفراد‬.

‫يمكن هي تجهل حقها االق نوني للذ ك‬


B ‫✅مناسبه‬

——————————————
⬛J ‫تحليل عقالني‬
Telegram comment: I couldn’t find any Saudi guidelines on the subject of domestic violence but
according to western guidelines, you cannot report domestic violence to the police without the patient’s
consent. However, child abuse must be reported regardless of the mother’s wish. The answer is either A
or B, but if educating her about violence means providing useful
information on how to report domestic violence and where to find support, then B is the correct answer.

Maram: Amboss ⬇️

—————

‫هنا تجميعات عشوائية غير واضحة في نفس هذة الفترة ( فترة أبريل ومايو‬
)2023
‫️⬇ اقرأو عن الموضوع وأهم النقاط عنها مثالً ملخص‬
Summary
Itiology
Clinical features
Diagnostic
Management &Treatment
Prognosis
Prevention &Screening

‫ملف األطفال‬:
1- adenovirus in preschool
‫ أتذكر‬-T.B for positive father his with contact years 3 child 2
‫إختيارات نعمل لطفل إيه‬
croup viral -3
‫سؤالين‬
4- ‫ أتذكر‬juvenile arthritis 5- ‫حالة‬
‫ في لاأ طفال‬A Hepatitis

-7 ‫سؤالين‬
‫ منهم سؤال‬bronchiolitis
O2 sat
80%
‫نعمله إيه‬
intubation and mechanical ventilation?

8- ‫سؤالين‬
milestones

_9‫كام سؤال تطعيمات‬

10- ‫حالة‬
Tonsillitis

‫النسا‬
endometriosis ‫جابوا حاجات كتير بصراحه من أول‬
‫وصوره لها و‬
adenomyosis ‫ و‬vaginal discharge ‫ و‬infertility ‫ و‬picos ‫ و‬labour ‫ و‬preclampsia ‫ و‬induction of ovulation

‫الباطنه‬
‫سؤال على صورة‬
sudden visual loss for 20 min then normal ‫عيان في‬
DM
hyperlipaedemia ‫و‬
‫و الختيارات‬
retinal detachment ‫أو‬
TIA

‫ و‬HT
RHF with ‫ سؤال‬losartan ‫جروب يسمى إيه‬
‫ و‬emphysema
TB for test ‫ و عيان‬consciousness lost and 70% sat O2 emphysema ‫نعمله إيه‬
‫ و مريضه جايه من الفلبين‬induration 10cm

paedia in IDA ‫أسئله عاديه عن إمتى تعمل‬


for Screen

‫الجراحه قت ريبا كلها من مذكره مارس و واإلثيكس بالنص‬

NSTEMI ‫حالة‬

bronchiolitis ‫صور أشعة صدر كبار و أطفال‬

neurolupus ‫و سؤال عن‬

‫و سؤال عن‬
OA distal phalynges in hand

‫و سؤال عن‬
phyloid breast tumour ‫ و‬wide tumour exision

mass ‫و سؤال عن‬


brain

pt ‫و سؤال عن‬
‫ لم‬epileptic
‫يستجيب ل‬
‫ نديله إيه‬diazepam

‫و سؤال عن‬
febrile convulsions

‫ سؤال و‬schizophrenia

‫و سؤال‬
dementia
‫ سؤال و‬peptic
ulcer

hpylori ‫و سؤال‬

‫و سؤال‬
Type of cancer in labia minora ‫غري‬
‫ب سؤال كان‬

‫ طفل و‬supracondylar fracture ‫مفيش و‬


distal pulse
‫و طبعا الحل يخش عمليات على طول‬

‫و سؤال عن‬
open book fracture pelvis

‫الجراحه قت ريبا كلها من مذكره مارس و واإلثيكس بالنص‬

◆ Similar recalls of NSTEMI & STEMI:

1-https://t.me/c/1222499606/301

2- https://t.me/silent_2023m/6213

3- https://t.me/silent_2023m/6214
—————————————————-

Q: Epileptic pt : no response to diazepam??


✓⬛⬛ Repeat every 5–10 minutes if no response.

Naif:
◆ Early status epilepticus (5–20 minutes): first-line therapy Administer
push dose. Repeat every 5–10 minutes if no response.
• IV benzodiazepine (lorazepam OR diazepam)
• IM midazolam if IV access is NOT available
• Intranasal midazolam
• Buccal midazolam
• Rectal diazepam
————————————————-
⬇ ‫يتبع لتجميعات العشوائية الغير واضحة‬
◆ Pelvic fracture
—————————————————

◆ Vulvar and vaginal cancer


————————————————-

—————

appendicolithiasis or Facalith
Ask about management or Ttt: appendectomy Im not sure

Screenshot MCQs Collections. ⬇️


—————

253-

‫ الختيارات‬Diamnitionc‫هذا جى بس‬
‫كانت‬
8
<8
12-8
12>

Multiple pregnancy

Monozygotic twins;

Dichorionic and Diamniotic; 1-3 days

monochorionic/diamniotic; 4-8 days


Monochorionic and monoamniotic ; 8-13 days Vs.

Dizygotic twins;

Dichorionic and Diamniotic regardless of the sex

‫ اعطونا رأيكم بهالسؤال من ملف ابريل و مايو نفس السؤال‬J 2023⬛


https://t.me/saudismle/823 ‫⬛ شوفو التعليقات واعطوني رأيكم بالتعليقات هنا‬J ⬛
254- Case about achalasia.(women with dysphagia liquid more than solid)
Fist ?
1_barium
sollow⬛✓
2_endoscopie⬛✓
3_esophageal echo

Naif: if there is manometry in choices I will select it as a confirmatory test. If manometry not found
in choices I will go with 2_endoscopie✅ as first.

If we suspicious Achalasia 1- Barium sollow. Look for AMBOSS⬇️

-2022 ‫⬛'هذا في ملف عبدالرحمن القويز‬


2021
——————————————————

3cAchalasia⬛J⬛
Approach
◦ In general, all patients with suspected achalasia should initially undergo upper endoscopy and/or
esophageal barium swallow; findings may support the diagnosis.
◦ Esophageal manometry is indicated to establish the diagnosis (confirmatory test of
choice), irrespective of the initial imaging findings.
◦ If manometry is inconclusive and an esophageal barium swallow was not obtained initially,
esophagram can also play a confirmatory role.
◦ Endoscopy should be performed to rule out pseudoachalasia because the presentation and
manometric findings of a mechanical cause of obstruction (e.g., a malignancy) may mimic achalasia.

‫تواصلت مع استشاري جراحة عامه وهذا ردة‬

‫في الحاله هذي هو سأل عن ال‬first

First⬛~ endoscopy EGD


Gold standard for achalasia ⬛~ Barium sollow

Confirming manometry

‫؟؟ صح‬

‫* ماشي بس الحقيقية الموضوع فيه فت اصيل كا تر من كدا* صيغة السؤال من لاا ساس خطأ‬: ‫ اسحاق‬.‫د‬

- 255- Important old Q:

- pt on chemotherapy fever neutropenia abdominal pain ?

- typhilitis⬛✓
Naif: Answer: Neutropenic enterocolitis (typhlitis) ✅

Gastrointestinal causes of acute abdomen⬛J


◆ Neutropenic enterocolitis is an inflammation of the ileocecal region seen in patients with
neutropenia, most often in the context of *recent cancer therapy.*

——————————————————

‫هنا تجميعات عشوائية غير واضحة في نفس هذة الفترة ( فترة أبريل ومايو‬
)2023
‫️⬇ اقرأو عن الموضوع وأهم النقاط عنها مثالً ملخص‬
Summary
Itiology
Clinical features
Diagnostic
Management &Treatment
Prognosis
Prevention &Screening
10T ️ ⬇ Amboss & ‫حسابات مجانية رابط رسالة من قناة‬
Uptodate
‫ وهنا رابط القناة‬https://t.me/saudismle/1021
‫علشان اذا ما فتح رابط الرسالة‬
Telegram app Channel (T10): https://t.me/saudismle

-Screening for SCA?


-Ecg PE ?

-Pseudocycts ? Milestones

one Q ?

Q: Pictures / read about the topic + looking for pictures and understanding them:⬛J
endometriosis + open fracture + Failure to thrive / ‫صور‬

—————
—————

.†_•n Keep in mind:

Obsession = thoughts, thoughts thoughts 3c3c3c3cc3


Compulsive = actions, actions actions, actions Sy) (v¶ k ⎝directed by
obsessive thoughts

—————

256- Old female has pupic itch and bloody discharge pea shaped swelling in labia

A bartholin cyst
B bartholin gland carcinoma C
abcess
B,✓

Old , bloody disch , pubic itching .

recall: Full ‫⬛طيب شوفو السؤال المشا هب وشوفو نقاشنا بالتعليقات‬J ⬛


https://t.me/silent_2023m/7033

Q: Old female has pupic itch and bloody discharge pea shaped swelling in labia
A bartholin cyst
B bartholin gland carcinoma C
abcess
D Squamous cell carcinoma

Naif: Most common D- squamous cell cancinoma✅


‫‪Dr.Khaled Belal:‬‬

‫ملحوظه‬
‫عشان ‪ reference‬اإلجابه ح تغير على حد معلوماتي و إرجعوا ل ‪ carcinoma cell squamous‬لو جه في اإلختيار السؤال ده جالي‬
Dr. Mohammed Yaman: The scenario you presented describes an elderly female with
pubic itch, bloody discharge, and a pea-shaped swelling in the labia. Based on this
information, the most likely diagnosis is option A) Bartholin cyst.

A Bartholin cyst is a fluid-filled sac that develops in the Bartholin gland, which is
located near the opening of the vagina. Symptoms of a Bartholin cyst can include pain,
swelling, redness, and a discharge. In some cases, the cyst can become infected and
develop into an abscess, which can cause more severe symptoms such as fever and
chills.

Bartholin gland carcinoma is a rare type of cancer that can develop in the
Bartholin gland, but it typically presents with different symptoms such as a
painless lump or mass in the vulva.

An abscess is a collection of pus that can develop in response to an infection, and it


can occur in the Bartholin gland or other areas of the vulva. Symptoms of an abscess
can include pain, swelling, redness, and fever.
Squamous cell carcinoma is a type of skin cancer that can develop in the vulva, but it
typically presents with different symptoms such as an ulcer or growth on the vulva that
does not heal.

Therefore, the correct answer to your question is option A) Bartholin cyst. It is


important to evaluate the patient for any signs of infection, such as fever or chills, and
to refer her for further evaluation and management by a gynecologist or other
healthcare provider. Treatment for a Bartholin cyst may include antibiotics, drainage of
the cyst or abscess, or surgical removal of the gland.

Maram: similar recalls ⬛J

257- 68 old man ( don’t sure about the age ) c/o abdominal pain, vomiting, jaundice /
lap high ALT , AST + other test all high and ( CA19-19 very high) dx :

Liver tumor
Cancer of head of pancreas

Naif: CA19-19 very high⬛J⬛ not specific ◆


258- A hypertensive patient was prescribed Hydralazine 25mg by his physician.
Instead he took Hydroxyzine 25mg. Which of the following is the likely type of error?

Look-a-like error
Sound-a-like error
Illegal prescription
Handwriting error

.‫في اختالف عاإلجابات‬


‫رأيي الشخصي واللي فهمته على حسب صياغة السيناريو ااذ كان في وصفه طبيه او قراءة ف الخطاء بيكون‬
look alike
alike sound ‫اذا بالصوت فيكون تشابه صوتي‬
It’s A according to FDA and ISMP

comment:

-The scenario you presented describes a patient who was prescribed Hydralazine 25mg but
mistakenly took Hydroxyzine 25mg instead. Hydralazine is a medication used to treat high blood
pressure, while Hydroxyzine is an antihistamine used to treat itching and anxiety. Both of these
medications have similar names, which can lead to confusion and errors.

Therefore, the likely type of error in this scenario is a look-a-like error. Look-a-like errors occur
when two drugs have similar names, packaging, or appearance, leading to confusion and medication
errors. These errors are particularly common when medications are prescribed, dispensed, or
administered in a busy healthcare environment where distractions and interruptions can occur.
It is important for healthcare providers to take steps to minimize the risk of look-a-like errors, such as
using tall man lettering to distinguish similar drug names, avoiding the use of abbreviations, double-
checking the medication name and dosage before prescribing, dispensing, or administering
medications, and educating patients on the importance of medication safety.

259- Old female has pupic itch and bloody discharge pea shaped swelling in labia

A bartholin cyst
B bartholin gland carcinoma C
abcess
D Squamous cell carcinoma

^Most common D- squamous cell cancinoma ‫منقول‬/

‫ملحوظه‬
‫لو جه في اإلختيار‬
squamous cell carcinoma
reference ‫اإلجابه ح تغير على حد معلوماتي و إرجعوا ل‬
‫عشان السؤال ده جالي‬

-The scenario you presented describes an elderly female with pubic itch, bloody discharge, and a pea-
shaped swelling in the labia. Based on this information, the most likely diagnosis is option A)
Bartholin cyst.
A Bartholin cyst is a fluid-filled sac that develops in the Bartholin gland, which is located near the
opening of the vagina. Symptoms of a Bartholin cyst can include pain, swelling, redness, and a
discharge. In some cases, the cyst can become infected and develop into an abscess, which can cause
more severe symptoms such as fever and chills.

Bartholin gland carcinoma is a rare type of cancer that can develop in the Bartholin gland, but it
typically presents with different symptoms such as a painless lump or mass in the vulva.

An abscess is a collection of pus that can develop in response to an infection, and it can occur in the
Bartholin gland or other areas of the vulva. Symptoms of an abscess can include pain, swelling,
redness, and fever.

Squamous cell carcinoma is a type of skin cancer that can develop in the vulva, but it typically presents
with different symptoms such as an ulcer or growth on the vulva that does not heal.

Therefore, the correct answer to your question is option A) Bartholin cyst. It is important to evaluate
the patient for any signs of infection, such as fever or chills, and to refer her for further evaluation and
management by a gynecologist or other healthcare provider. Treatment for a Bartholin cyst may
include antibiotics, drainage of the cyst or abscess, or surgical removal of the gland.

260- Women came with severe itching white vaginal discharge and you
saw pseudohyphae what is your Dx?

Candida

261- Female with recently inserted IUD c/o abdominal pain and brown discharge. Dx?

A. uterine rupture
B. bacterial vaginitis
C. PID
D. ovarian torsion

262- 70-year-old male K/C of HTN with Gradually decline in cognitive state, MRI
showed Periventricular white matter Hyperintensities, what is the diagnosis?

A- Vascular dementia
B- Alzheimer disease

263- Female patient presented with signs and symptoms of SBO and mass irreducible
medial to pupic tubercle. What is your provisional diagnosis?
A-inguinal hernia B
-femoral hernia C-
obturator hernia D-
incisional hernia

264- Female patient complaing of menstrual irregularity,hypertension,


Hirtusm With high androgen in the blood
US shows multiple small cysts in the ovaries What is the most likely diagnosis?

A— gynecomastia
B— Kallmann syndromes
C— klinefelter syndrome
D— Stein-Leventhal syndrome

comment:

-The scenario you presented describes a female patient with menstrual irregularity, hypertension,
hirsutism, high androgen levels in the blood, and multiple small cysts in the ovaries on ultrasound.
These findings are consistent with a diagnosis of Stein-Leventhal syndrome, also known as
polycystic ovary syndrome (PCOS).

PCOS is a common endocrine disorder that affects women of reproductive age and is characterized by
a combination of clinical and biochemical features, including menstrual irregularity, hirsutism, acne,
obesity, infertility, and insulin resistance. The hallmark feature of PCOS is the presence of multiple
small cysts in the ovaries on ultrasound.

Gynecomastia is the development of breast tissue in males and is not related to the symptoms and
findings described in this scenario. Kallmann syndrome is a genetic disorder that affects the
hypothalamus and is characterized by delayed or absent puberty and anosmia (lack of sense of smell).
Klinefelter syndrome is a genetic disorder that affects males and is characterized by an extra X
chromosome, which can lead to infertility and other physical and mental health problems.

Therefore, the correct answer to your question is option D) Stein-Leventhal syndrome (polycystic
ovary syndrome). This diagnosis is supported by the combination of clinical and biochemical
features, including menstrual irregularity, hirsutism, high androgen levels, and multiple small cysts
in the ovaries on ultrasound.

265- 11-month-old boy is not moving his right hand and when he presented to
the hospital it appeared that he had right ulnar and radial fracture presented after
2days history of fall on an outstretched hand when the doctor asked the
parents why they didn’t bring him earlier they said that they didn’t see any
bruises. child is dressed well, well-nourished and has had regular well baby
visits.

A) Treat the child and inform the authorities B)


Believe the parents and treat the child
C) Treat the child and order head CT
D) Call the police

comment:

-The scenario you presented describes an 11-month-old boy who presented to the hospital with a
history of a fall on an outstretched hand and now has a right ulnar and radial fracture. The parents did
not bring the child earlier because they did not see any bruises. In this case, the best next step would
be to believe the parents and treat the child.

It is not uncommon for parents to delay seeking medical attention for their child's injuries, especially if
there are no visible signs of trauma. Additionally, the child is well-dressed, well- nourished, and has
had regular well-baby visits, which suggests that the parents are providing appropriate care for the
child.

However, it is important to document the history and physical examination findings in the medical
record, including any concerns about the delay in seeking medical attention. If there are any
suspicions of child abuse or neglect, it is the healthcare provider's responsibility to report it to the
appropriate authorities.

Therefore, the correct answer to your question is option B) Believe the parents and treat the child.
The child needs appropriate medical care for the ulnar and radial fractures, and it is important to
maintain a non-judgmental and supportive approach to the parents to build trust and ensure the best
possible outcome for the child.

266- 38 Y.O. Pt was having GERD and on PPl 40 mg. His symptoms was controlled. He
stopped the medication 4 months ago and now his symptoms is back again but worse.
All labs and P/E is normal. what is your next step?

A. Resume same dose.


B. Give higher dose PPI 80mg.
C. Give ranitidine and PPI.
D. Refer him for upper Gl endoscopy.

comment:

-The scenario you have presented describes a 38-year-old patient who had been taking a PPI (proton
pump inhibitor) 40 mg for GERD, but stopped taking it 4 months ago. The patient's symptoms have
returned and are now worse. The next step in management would be to refer the patient for an upper
gastrointestinal endoscopy.
An upper gastrointestinal endoscopy is a procedure in which a thin, flexible tube with a camera at the
end is inserted through the mouth and into the esophagus, stomach, and duodenum to visualize the
upper digestive tract. This procedure can help identify any structural abnormalities or damage to the
esophagus or stomach, such as erosive esophagitis, Barrett's esophagus, or gastric ulcers, which may be
causing the patient's symptoms.

While resuming the same dose of PPI or giving a higher dose may provide symptomatic relief, it is
important to identify the underlying cause of the patient's symptoms to guide further management
decisions. Ranitidine and PPI combination therapy may also be effective in some cases, but the most
appropriate next step in this scenario would be an upper gastrointestinal endoscopy to evaluate for any
structural abnormalities.

Therefore, the correct answer to your question is option D) Refer the patient for upper gastrointestinal
endoscopy. This procedure will help identify any underlying structural abnormalities or damage to the
upper digestive tract and guide further management decisions for the patient's GERD symptoms.

267- 24 YO female patient noticed that her abdominal bloating and diarrhea
improved after she starts gluten free diet
What is the best next step?

A. Gastroenterology referral
B. Refer to nutritionist
C. Resume gluten diet and order celiac serological blood test
D. Send stool culture

comment:

-The scenario you presented describes a 24-year-old female patient who noticed that her
abdominal bloating and diarrhea improved after she started a gluten-free diet. These symptoms
are suggestive of a possible gluten sensitivity or celiac disease, which is an autoimmune disorder
that can cause damage to the small intestine in response to gluten consumption.

The best next step in this scenario would be to resume the gluten-containing diet and order celiac
serological blood tests. These tests, such as tissue transglutaminase antibody (tTG-IgA) and deamidated
gliadin peptide (DGP) antibody, can help confirm or rule out celiac disease as a cause of the patient's
symptoms. It is important to note that these tests should only be done while the patient is consuming
gluten, as a gluten-free diet can lead to false-negative results.

If the celiac serological blood tests are positive, then a referral to a gastroenterologist would be
appropriate for further evaluation and management. A nutritionist referral would also be beneficial to
help the patient navigate the gluten-free diet and ensure adequate nutrient intake.
Therefore, the correct answer to your question is option C) Resume gluten diet and order celiac
serological blood test. This will help confirm or rule out celiac disease as a cause of the patient's
symptoms and guide further management decisions.

268- child 4 month burn in gentali


1% third degree

a. refer to burn unit


b. Abx ointment
c. dressing outpation
d. cream not sure

comment:

-A third-degree burn is a serious injury that requires prompt and appropriate treatment to prevent
complications and promote healing. In the scenario you presented, a 4-month-old child has a third-
degree burn in the genital area. In this case, the most appropriate next step would be to refer the child to
a specialized burn unit for further evaluation and management.

Burn units are specialized facilities that are equipped to provide comprehensive care for patients with
severe burns. Burn units typically have a multidisciplinary team of healthcare professionals,
including burn surgeons, nurses, physical therapists, and social workers, who work together to
provide optimal care for patients with burn injuries.

In addition to referral to a burn unit, other management strategies may include antibiotic ointment to
prevent infection, dressing changes to promote healing and prevent further injury, and pain
management to keep the child comfortable. However, it is important to note that the management of
third-degree burns requires specialized expertise and resources that may not be available in a general
healthcare setting.

Therefore, the correct answer to your question is option A) refer to a burn unit. The child's condition
requires specialized care and management in a burn unit to ensure the best possible outcome.

269- A woman with a left breast mass for 9months. Mammogram: speculated mass with
suspicious microcalcification and axillary lymph node involvement, BI-RAD V (probable
malignancy), next step?

A. Excisional biopsy
B. core Biopsy
C. Modified radical mastectomy
comment:

-The scenario you presented describes a woman with a breast mass, suspicious microcalcifications, and
axillary lymph node involvement on mammography, which is classified as BI-RADS category V,
indicating a probable malignancy. In this case, the next step in the management of this patient would be
a core biopsy.

A core biopsy is a minimally invasive procedure in which a sample of breast tissue is obtained using a
needle under image guidance. This procedure can provide a definitive diagnosis of breast cancer,
including the type and grade of the cancer, as well as information about hormone receptor and HER2
status, which are important for guiding further treatment.

Excisional biopsy and modified radical mastectomy are other surgical options for obtaining a tissue
sample and treating breast cancer. However, these procedures are more invasive than core biopsy and
are typically reserved for cases in which the diagnosis cannot be made with core biopsy or when the
tumor is too large to be removed with a less invasive procedure.

Therefore, the correct answer to your question is option B) core biopsy. Core biopsy is the
recommended next step in the management of suspicious breast lesions as it provides a definitive
diagnosis with minimal invasiveness and can help guide further management and treatment
decisions.

270- 4 days after abdominal surgery presented with hypotension despite


fluid resuscitation and warm periphery

Temp is 36.6

(no other clues infos or labs)

Diagnosis?

A. Septic shock
B. Cardiogenic shock
C. Anaphylactic shock
D. Hemorrhagic shock

Telegram comment:
https://t.me/silent_2023m/7052

Dr. Mohammed Yaman :


-The scenario presented describes a patient who underwent abdominal surgery and now presents
with hypotension despite fluid resuscitation and warm periphery. The patient's temperature is
normal at 36.6 degrees Celsius. Based on this limited information, the most likely diagnosis in this
scenario is septic shock.
Septic shock is a life-threatening condition that occurs when an infection leads to a systemic
inflammatory response, causing hypotension and reduced tissue perfusion. Patients with septic shock
may present with fever or hypothermia, tachycardia, tachypnea, altered mental status, and signs of
organ dysfunction. Hypotension that does not respond to fluid resuscitation is a hallmark feature of
septic shock.

The absence of fever in this scenario does not exclude septic shock, as not all patients with septic shock
will have fever. Additionally, the recent surgery increases the risk of infection and the development of
sepsis.

Therefore, based on the information provided, the most likely diagnosis in this scenario is option A)
Septic shock. It is important to promptly recognize and treat septic shock to prevent complications and
improve outcomes. Treatment typically involves early administration of antibiotics and aggressive
fluid resuscitation, as well as other supportive measures such as vasopressors and mechanical
ventilation if needed.

Naif:
periphery ‫الوح دي ة اللي فيها‬
‫ هي‬Warm
◆ *Distributive shock:*
1- Septic shock
2- Anaphylactic shock
shock Neurogenic 3-

‫ يعني نستبعد‬B&D

‫يبقى‬
A- septic shock (with fever & sings of inflammation) C-
Anaphylactic shock ( no fever )

‫عموماً السؤال ناقص معلومات ما اقدر احدد او اقرب الجابه‬


Wound infection ‫ حصل ممكن‬Septic ‫الى‬
‫ اقربها‬Anaphylactic: no evidence
Maram:
shock ‫احسه ناقص عشان قن ول‬
‫ الزم يكون في‬anaphylactic
History of allergy ‫ال‬
‫سؤال في‬
‫ مافي‬shock septic
‫شي دي ل على‬
infection
‫والمريض عنده حراره‬
————————————————-
SMLE-oriented Types of shock:

1- Cardiogenic:
- History of Cardiac cause as ischemic heart disease.
- Increased Pulmonary capillary wedge pressure PCWP. ( ‫) حاجه أهم‬
- High vascular resistance.

2- Hypovolemic:
- History of RTA, Bleeding.
- low PCWP.
- High vascular resistance.

3- Neurogenic:
- History of RTA, Spinal injury.
Bradycardia ( ‫ ) أهم حاجه‬-
- Warm extremities

4- Septic:
- Septic focus: Post op or Pnemonia and sepsis or Obstetrics complications and also fever.
- Tachycardia
- Increased COP ( ‫) حاجه أهم‬
- Warm extremities.

5- Anaphylactic:
- Wheezing, Urticaria and allergic Sx.
- History of Bee sting, allergic medication or allergic food.

271- Salicylate toxicity


Q: Case of aspirin overdose, what’s the electrolyte disturbance that will happen?

A- respiratory alkalosis followed by metabolic acidosis B-


metabolic alkalosis followed by metabolic acidosis
C- respiratory acidosis followed by metabolic alkalosis
D- metabolic acidosis followed by metabolic alkalosis

comment:

-Aspirin (acetylsalicylic acid) is a commonly used medication for its analgesic, anti-
inflammatory, and antipyretic properties. However, an overdose of aspirin can lead to
salicylate toxicity, which can cause several clinical manifestations and electrolyte
disturbances.
One of the most common electrolyte disturbances seen in salicylate toxicity is respiratory alkalosis,
which is characterized by a decrease in carbon dioxide (CO2) levels in the blood due to
hyperventilation. This occurs because aspirin stimulates the respiratory center in the brain and causes
hyperventilation, which leads to a decrease in CO2 levels in the blood and an increase in blood pH,
leading to respiratory alkalosis.

As the salicylate toxicity progresses, it can lead to metabolic acidosis due to the accumulation of
salicylate in the body and its effects on the acid-base balance. Therefore, the correct answer to your
question is option A) respiratory alkalosis followed by metabolic acidosis.

It's worth noting that other electrolyte disturbances seen in salicylate toxicity include hypokalemia,
hypoglycemia, and hypocalcemia. Treatment for salicylate toxicity typically involves supportive care,
such as maintaining adequate hydration and electrolyte balance, as well as specific therapies such as
activated charcoal and sodium bicarbonate to enhance salicylate elimination and correct metabolic
acidosis.

272- Mother brought her 9 months old infant and they’re investigating iron deficiency
anemia. What is suspected to be increased?

A. Hemoglobin
B. MCV
C. Total iron binding capacity

comment:

-Iron deficiency anemia is a condition in which the body doesn't have enough iron to produce
hemoglobin, the protein in red blood cells that carries oxygen to the body's tissues. This can lead to a
decrease in hemoglobin levels and a decrease in the size of red blood cells (microcytic anemia) as
well as an increase in the total iron binding capacity (TIBC).

Therefore, the correct answer to your question is option C) Total iron binding capacity. TIBC is a blood
test that measures the amount of iron that can be bound by the proteins that transport iron in the blood.
In iron deficiency anemia, TIBC is increased because the body is trying to compensate for the low
levels of iron by increasing the amount of iron-binding proteins in the blood.

In contrast, hemoglobin levels and mean corpuscular volume (MCV) are typically decreased in iron
deficiency anemia due to the lack of iron needed to produce hemoglobin, the protein that gives red
blood cells their color and size. So, options A and B are not the expected findings in iron deficiency
anemia.

273- case of child he had mumps do written what is the most affected part ?
A-Lung
B-Testis
C-Parotid

comment:

-Mumps is a viral infection caused by the mumps virus, which primarily affects the salivary glands,
especially the parotid glands located in front of the ears. The virus is spread through respiratory
droplets from an infected person, and symptoms typically appear 2-3 weeks after exposure.

The most commonly affected part in mumps is the parotid gland, which can become swollen and
painful. Other symptoms may include fever, headache, muscle aches, fatigue, loss of appetite, and
pain or swelling in the testicles (orchitis) for males who have reached puberty.

Therefore, the correct answer to your question is option C) Parotid gland. It's worth noting that while
mumps primarily affects the salivary glands, it can also rarely cause complications such as meningitis,
encephalitis, pancreatitis, or hearing loss.

274-

According to hamza C

Superficial , less than 5 cm = excisional

275- 14 years girl anorexa nervoda had low all electrolyt (vomting not sure) ask
about loss of weight…

a. malabsorbation
b. refeedless $
c.adequt nutration not sure

comment:
-The given scenario describes a 14-year-old girl with anorexia nervosa who has low levels of all
electrolytes. Anorexia nervosa is an eating disorder characterized by a distorted body image and an
intense fear of gaining weight, leading to self-starvation and weight loss.
Electrolytes are minerals in the body that help regulate various bodily functions, including heart rhythm,
nerve and muscle function, and fluid balance.

The most likely cause of low electrolyte levels in this scenario is refeeding syndrome. Refeeding
syndrome is a potentially life-threatening condition that can occur when a severely malnourished
person begins to eat again, leading to a rapid shift in fluids and electrolytes in the body. This can cause
a range of symptoms, including low levels of electrolytes such as potassium, magnesium, and
phosphate. Vomiting can also contribute to electrolyte imbalances.

Therefore, the correct answer to your question is option B) refeeding syndrome. It is important to
address refeeding syndrome in a systematic and controlled manner under the supervision of a healthcare
professional to prevent potential complications and ensure adequate nutritional support.

—————

*contraception of endometriosis.

*hydatiform mole
*ectopic pregnancy

-Pic Ecg hypo k


-Pic endometriisis
-Pic bacal cell concer in valva
-Pic MI ecg

—————

276- Type of sport recommended for premenstrual syndrome

1. Aerobic
2. Flexibility and mobility
3. Stability and balance
4. Coordination and agility
277- Old man came with sings and symptoms of endocarditis with murmur and
asking what is the most common organism

A) Staph, epidermis
B) klebsiella pneumoniae
C) streptococcus species
D) S. Pnumonia

comment:

-Endocarditis is an infection of the inner lining of the heart chambers and heart valves. The most
common organism that causes endocarditis depends on several factors, including the patient's age,
underlying medical conditions, and risk factors for endocarditis. However, in general, the most
common organism that causes endocarditis is Streptococcus species.

Therefore, the correct answer to your question is option C) Streptococcus species. However, it's worth
noting that other organisms, such as Staphylococcus aureus and Enterococcus species, are also
common causes of endocarditis in certain patient populations.

278- Female diagnosed myoma in uterus she have bleeding want to be pregnant?

1- NSAID’s
2- Ocp
3- Progesterone only
4-Combined

1. NSAIDs (nonsteroidal anti-inflammatory drugs) such as ibuprofen or naproxen can help reduce
the bleeding associated with myomas. However, they do not treat the underlying condition and are
not recommended for long-term use.

2. Oral contraceptive pills (OCPs) can help regulate the menstrual cycle, reduce bleeding, and
potentially shrink the myoma. However, OCPs are not recommended for women who are actively
trying to conceive.

3. Progesterone-only treatments, such as a progesterone-releasing intrauterine device (IUD) or


injections, can also help reduce bleeding and potentially shrink the myoma. These treatments are
generally safe for women who want to become pregnant, as they do not affect fertility.

4. Combined hormonal treatments, such as combined estrogen and progesterone pills or patches, are
generally not recommended for women with myomas, as they can stimulate the growth of the myoma
and increase the risk of complications.
279- Pt post surgery he was an well after surgery but in 5th his vital becom worth spo2
80 ‫ ? نازلة الحيوية عالماته وكل‬NEXT

1- intubation
2-laparotomy

280- Case of patient post op with chest pain and abdominal pain and the patient was
vitally stable what is the next step ?

1-xray
2-us
3-laparotomy
4-forget

281- Which of the following Occur with ocp?

A- decrease the risk of ovarian cancer


B-increase the risk of breast cancer C-
decrease endometrial cancer
D- increase risk of ectopic pregnancy

-The use of oral contraceptive pills (OCPs) has been shown to decrease the risk of ovarian and
endometrial cancers. However, studies suggest that OCPs may slightly increase the risk of breast
cancer, particularly in women who use them for a long time (more than 10 years). OCPs do not
increase the risk of ectopic pregnancy, but they do slightly increase the risk of a rare type of ovarian
tumor called a "borderline" or "low malignant potential" tumor. It is important to note that the overall
benefits of OCPs in preventing unintended pregnancies and reducing the risk of certain cancers
generally outweigh the risks associated with their use.
However, you should always talk to your doctor about the potential risks and benefits of any
medication you are considering taking.

-So the correct answers are:


A- decrease the risk of ovarian cancer C-
decrease endometrial cancer

And the incorrect answers are:


B- increase the risk of breast cancer D-
increase risk of ectopic pregnancy
—————

—————

282- Lactating women present with right breast pain for 6 day on examination,
hot tender swelling lateral to
the right areola Pt started to take floxacillin Ultrasound showed Cystic lesion, thickened
content, What
next?

A. Incision and drainage


B. Repeated aspiration
C. Excisional biopsy
D. Reassurance or antibiotic

283- Patient had a stab wound in the chest. Came with weak thread pulses, raised
JVP, Equal Bilateral Air
Entry in both lungs. What's the diagnosis?

A. Pneumothorax
B. Cardiac tamponade
C. Pulmonary contusion
pericardiocentesis ‫ و الجابات كلها غلط ال‬,‫جات حاله اعطاني العراض حق كاردياك تامبونيد دب ون يعطيني التشخيص‬

284- Picture of Two huge echinococcosis hydatid cyst but size wasn’t
mentioned, asking what most
appropriate initial step?

A. Albendazole
B. Surgical deroofing
C. Percutaneous drainage
D. Liver resection

‫ يسأل ايش النكست ستيب اخترت‬,


‫ جات حاله‬liver abscess with (septations (echinococcus)
‫يقولي المريض عنده‬
albendazole

285- Cirrhotic patient complicated came with portal hypertension, came with upper
GI bleeding due to varices
diagnosed by Upper Endoscopy, what to give after resuscitation?

A. Somatostatin
B. Terlipressin
C. Octreotide
D. None

varices esophageal bleeding ‫عنده مريض‬


endoscopy
‫يسأل ايش اللي نعطيه اول قبل دن خله‬

286- There was a question about a lady known case of gallstones for 15
years presented with abdominal pain
and fatigue There was air in biliary system, what is the diagnosis?

A. Gallstone ileus
B. Acute Cholecystitis
C. Acute pancreatitis
‫ التشخيص‬+ abdominal distension and pain ‫ ال يثن ن اعطوني بتي كل سيناريو‬gallstone (ileus ( ‫جاء حالتين اثنين عن نفس‬
pneumobilia
‫ اتوقع انه واضح‬,‫كانو يسألوا عن التشخيص‬

287- 50 years old female presented to ER with sudden left lower chest pain
and epigastric pain, after forceful
vomiting. On examination there was decreased breath sound in the left lower chest
What is the most
likely concern?

A. Pneumonia
B. Pneumothorax
C. Aspiration
D. Esophageal Perforation⬛✓⬛

Similar recall: from Hamza file.

Comment:

A > sure not


B > no hx suggesting any 1ry or 2ry causes of pneumothorax C >
not a viable option
D > could be

D>Boerhaave syndrome⬛✓

Naif: Topic summary J⬛


https://t.me/saudismle/937 T10 message link ( in comments)
https://t.me/saudismle T10 channel link

288- Female P6, presented with an abdominal bulge, painless, no other symptoms,
been there for 3 years.
On examination, there’s a midline bugle that increases when the patient leans to get up
from laying
down, negative cough test.
CT: showed no intra or extra masses, abdominal fascia is intact no defects.
What’s the most likely diagnosis?

A- incisional hernia.
B- divarication of recti.
C- spigelian hernia.
D- weakness in the transversalis fascia.
289- Something years old patient came with history with middle line neck swelling
that move up and down
with tongue protrusion what is the diagnosis?

A. Brachial cyst
B. Thyroglossal cyst
C. Zenker diverticulum
290- Patient Has ischemic heart disease came with sudden severe abdominal pain
in the left iliac fossa and
tenderness

A. Diverticulitis
B. Mesenteric ischemia
C. Perforated viscous

291- Patient with abdominal pain, vitally stable, imaging shows: increase thickness of
sigmoid and thumb print
due to edema. What is the next step?

A. Colonoscopy
B. Diagnostic laparoscopy

ischemic colitis ‫جاء السؤال هذا بس كان يسأل ايش التشخيص و الجابة‬

292- Patient involved in RTA with isolated head injury, he remains in coma for 5
days after the accident, what is the appropriate to resuscitate him in early
management?

A. NGT
B. Gastrostomy tube feeding
C. Central line
D. Peripheral line

293- Patient post RTA with cervical emphysema, decrease air entry
and pneumomediastinum, what's the
diagnosis?

A. Tension pneumothorax
B. Tracheobronchial injury
C. Simple pneumothorax
D. Open pneumothorax
294- Male patient complains of episode of hematemesis after repeated vomiting.
what is the diagnosis?

A. Mallory Weiss syndrome


B. Erosive gastritis
C. Peptic Ulcer Disease

similar recall:⬇️

Pt with vomiting for 24hrs due to food poisoning came to Er complaining of 2 fresh blood , when do
NGT show occult blood, What’s the most likely diagnosis?
1. gastritis
2. dieuflory lesion
3. peptic ulcer
4. mallory weiss syndrome

Male 40s patient, had food poisoning and kept vomiting for 3 days with diarrhea. Came to you because
of vomiting blood. What's the diagnosis? in other recall 24 hours vomiting after food poisoning
develop hematemesis on endoscopy blood streak on stomach Dx?

A. Gastritis
B. Mallory weiss syndrome
C. Perforated ulcer
D. Esophagealvarices
Answer is: B

young man with multiple episodes of vomiting, last one was bloody, vitally stable,
what's the most appropriate treatment?
A. Conservative management

B. Upper GI endoscopy

C. Sengstaken tube
D. PPI
answer: B, Mallory Weiss tear.

By Maram note: / Dr. Mohammed Alharbi: ⬇️


https://t.me/IMCrashSMLE/186

young man with multiple episodes of vomiting, last one was bloody, vitally stable,
what's the most appropriate treatment?
Conservative management
Upper GI endoscopy
Sengstaken tube
PPI

Answer:
Upper GI Endo

The diagnosis of Mallory-Weiss syndrome should be suspected in patients with upper


gastrointestinal bleeding and a history of vomiting or retching. An upper endoscopy
establishes the diagnosis, rules out other etiologies, and allows for therapeutic
intervention.

Medical intern Naif: B ✅ we should be confirm Diagnosis by Upper GI endoscopy.

Dr. Albeidi’s Review: ⬇️


https://t.me/Abeidi_Review/4764
Young man presented with multiple episodes of vomiting the last one was bloody
he is vitally stable ( BP 110/76 ) What is the most appropriate?
A. conservative management
B. Upper GI endoscopy
C. Sengstaken tube

Should go with A as the Pt stable


? Or B for diagnosis ◆* ?
Azhar: This is a tricky question.
It describes a case of Mallory- Weiss Syndrome. Majority of patients stop bleeding
without any specific intervention, hence, treatment is “conservative”. But you won’t be
able to confirm diagnosis and adopt a conservative approach unless you do an upper
GI endoscopy.
So it is the question of who is first, the chicken or the egg.

295- Old age patient present with fresh blood per rectum, you did rectal
examination and your finger full of
blood, but anorectal colonoscopy was normal. CBC was: Hb 8. What is the investigation
that has a high
diagnostic value?

A. Upper GI endoscopy
B. Tch 99
C. CT
D. US

surgery by Hamzah answered it B


but according to uptodate the answer would be A see the approach for hematochezia below also, despite
Tch 99 is the most sensitive radiographic test for GI bleeding it is not use if there is an active bleeding
296- A 65-year-old male comes to the clinic with a mild intermittent urinary
flow reduction Rectal
examination, urinalysis and prostate specific antigen studies are normal (see report).
Ultrasound prostate:
Enlarged median lobe. Which of the following is the best way to investigate?

A. Annual renal function monitoring


B. Periodic PSA measurement
C. Beta-blocker therapy
D. Cystoscopy

A may be the answer if post void urine was high , but US normal

‫ بس هذا شوي قريب هل ااذ ما خانتني الذاكرة‬,‫جاء سؤالين عن ىبروستات بس ما اتذكرهم‬

297- Old patient underwent rectal surgery then developed Deep Venous Thrombosis
(DVT), what is the management?

A. Enoxaparin
B. Heparin
C. Warfarin
D. IVC

similar recall
Old pt after rectal surgery he is not doing well after in the recovery he start to have leg pain he
developed DVT from the popliteal to the femoral ..
A. Enoxiparen
B. Heparin
C. Warfarin
D. IVCP✓
Because Still in recovery room
-after rectal surgery; Enoxaparin. Unless in recovery room (within 24 hrs); IVC filter

298- 48 years old female with diffuse goiter, labs showed high T4 low TSH, US
showed bilateral thyroid nodules, right 3x4 in size, left is 1x2 size, what to do?

A. FNA both
B. FNA the larger one
C. Total thyroidectomy

surgery by Hamzah

299- PPH Medication?

A. Oxytocin 20 units mixed with 500ml D5 IV


B. Ergo 0.5mg IM
C. Ergo 0.2 mg IV

from Dr.Wafa's notes

Medications used in PPH Management:

1) Oxytocin 10-40 units in 500/1000ml NS/RL or 10 units IM (1st line) (For patients in
whom infusion of sodium chloride must be avoided, 5 % dextrose solution may be used)
2) Methylergonovine 0.2 mg IM (2nd line, Contraindicated in preeclampsia and HTN)
3) Carboprost (hemabate) PG F2a 0.25mg IM (3rd line, Contraindicated in Asthma)

4)Misoprostol 600-1000 micrograms PO, or rectal (4th line)

300- 41-year-old P5 +3 presented to the clinic complaining of abnormal uterine


bleeding her Menstrual period
is regular every 30-day associated with blood clots and pain that is not relieved by
simple analgesic she had previous myomectomy, she is a known case of PCOS And her
BMI is 40?
A-Adenomyosis
B-Endometriosis
C-Uterine fibroid
D-Endometrial hyperplasia

from Dr.Wafa's notes

Let’s Exclude!!

- Adenomyosis? Previous uterine surgery, multiparity and her age (all are risk factors
for adenomyosis!)
- Endometriosis -> will present with dysmenorrhea mainly (presentation goes with
adenomyosis more)
- Uterine Fibroid -> will present with bleeding only
- Endometrial hyperplasia -> will not present with dysmenorrhea

301- G3p0, A2 now at 5 weeks presented with spotting on examination open os and no
active bleed. History showed 2 abortions at 2nd trimester, last one with D&C
diagnosed as incompetent cervix.
Your diagnosis now for the third pregnancy of this patient?

a.Asherman syndrome
b.Incompetent cervix
c.Chromosomal abnormalities

from Dr.Wafa's notes

According to ACOG:

- Approximately 50% of all cases of early pregnancy loss are due to fetal chromosomal
abnormalities - In rare cases, after a D&C has been performed after a miscarriage, bands of
scar tissue, or adhesions,

may form inside the uterus. This is called Asherman syndrome

302- What is the method to detect TB is


sputum? Alaa: the ANSWER IS Ziehl-neelsen
staining

from Um-alqura

What is the method to detect TB is sputum? THE


ANSWER IS Ziehl-neelsen staining
EXPLANATION
Constitutional symptoms + respiratory symptoms
Looks like lung cancer but think of TB in patient in prison or returned back from
high risk area (e.g. india)

Chest x-ray finding:- Upper lobe cavity

Precaution:- Airborne (once you suspect,

isolate)
If there is associated pleural effusion, send sample for ADA level, AF culture
N.B: strong association between SILICOSIS and TB

Diagnosis:- sputum AFB


PPD and IGRA is not the right choice for symptomatic patient
It is only for screening for latent TB
Treatment:- Isoniazid + Pyridoxin, Rifampicin, Ethambutol, pyrazinamide (RIPE +
VIT D)

303- A patient presented with moderate pleural effusion. What is the indication of
chest tube insertion?

A- Loculated pleural fluid


B- Negative gram stain of the pleural fluid
C- Glucose > 80
D- pH > 7.35

from Um-alqura

304- Facial plethora and JVP distended, which of the following is associated with
this condition

The answer is Small cell lung cancer

305- A 4yr child with asthma, The most accurate diagnostic test for asthmatic
patient is?

A- his response to medication


B- Spirometry
C- Peak flow meter
D- Chest x-ray
E- IgE test

Similar recall:
from Um-alqura
Maram By /‫ نستخدم‬5 ‫ لاأ طفال األكبر من‬b
⬇ ️ High Yield ‫قناة‬

⬇ ️Bdran’s space:
https://t.me/BdranSpace/127
306- Hypertensive patient recently diagnosed as bronchial asthma, which
medication you will stop?

A- Propranolol cause bronchoconstriction and worsen symptoms in pt with asthma


B- Amlodipine
C- Captopril
D- Perindopril

307- Asthmatic patient on SABA + LABA + ICS and still uncontrolled, what will you add
in the management?

A- Low-dose oral prednisolone


B- Leukotriene inhibitor
C- Medium-release theophylline
D- Inhaled ipratropium

308- Child with forigen body aspiration (peanut) and doctor prepare him to
do bronchoscopy, where did you expect its location?

A- Larynx
B- Trachea
C- Right main bronchus because it is straight D-
Left main bronchus

309- Elderly medical and surgical history were normal not taking any medication
brought by his wife complaining from gradual memory loss and tremor, but can take
care of himself dressing and eating and personal care were not affected there was also
personality change he became more aggressive and agitated, examination showed
normal findings. What is the diagnosis?

1 - Lewy body dementia⬛✓⬛


2 - Alzheimer’s dementia
3 - vascular dementia

Naif:
Dementia ‫اللي حط السؤال بيلخبطنا ويشككنا‬
‫ الجابه الصحيحه‬J⬛ ‫ ب‬Alzheimer’s
1- Lewy body dementia⬛✓⬛
tremor ‫الن الوح دي اللي معه‬
‫هو‬:
Dementia with Lewy bodies (DLB):
• Visual hallucinations and parkinsonian motor disorders
• Attention impairment

Ref.Amboss

Similar recall:
1- https://t.me/silent_2023m/3136 ( in comments some important discussions & explanations
Also there are some masseges links of Similar recalls)

310- 81 year old man presented for memory loss and difficulty in recognizing his
grandson, he doesn’t have cardiovascular risk factors. He drinks alcohol occasionally
Which of the following is the diagnosis:

A- Alzheimer disease
B- lewy body dementia comes w motor symptoms like parkinson disease + visual
hallucinations
C- Alcoholic induce encephalopathy comes w excessive drinking
D- Parkinson comes w motor symptoms

311- Old age pt with cardiomegaly. he's asymptomatic Echo showed: EF 40%. what
is the best Mx

A- digoxin
B- Lisinopril
C- echo after 6 months
D- furosemide
Bader H. : Lisinopril is an ACE inhibitor used to treat heart failure with a low EF. It
helps to improve symptoms, reduce hospitalizations, prolong survival in patients with
systolic heart failure with an EF less than or equal to 40%.

312- A 67 year old male presents with insomnia, irritability and palpitations for
3 months. He is known to have HTN, depression and Atrial fibrillation. He is on
amiodarone, fluoxetine, and enalapril.
Although he complains of palpitations, he has no increased heart rate. Clinically, he is
unremarkable.
Vitally stable. What is the most appropriate next step?

A. Add propranolol
B. Do thyroid function tests
C. Substitute antidepressant drug
D. Refer to psychiatry

Similar recall : https://t.me/IMCrashSMLE/3068


https://t.me/BdranSpace/104?comment=230

313- A diabetic lady presenting to the clinic asking about travelers’ diarrhea
prophylaxis. Mild Cr Elevation, BUN is high, urea is high. What prophylaxis to
give?

A. Fluoroquinolones
B. No need
C. Probiotics
D. Bismuth

Similar recall : https://t.me/BdranSpace/105?comment=233


https://t.me/smle22olds/3630

—————
—————

314- Male k/c of acute pancreatitis came with abdominal pain in RUQ In US :
gallbladder stone and
dilated CBD What is your management

A- laparoscopic cholecystectomy
B- ERCP
C- ABx
D- MRCP

Similar recall : https://t.me/BdranSpace/107


Acute pancreatitis and gallstones are interrelated conditions, and CBD (common bile
duct) dilation can indicate the risk of obstructive jaundice or cholangitis. Therefore, an
intervention is necessary to prevent worsening of the condition.

Bader H.

315- Scenario of gout case and the diagnosis given clearly. What is the characteristic
of gout crystals?

A. Rhomboid shaped negative birefringent


B. Rhomboid shaped positive birefringent
C. Needle shaped strong negative birefringent
D. Needle shaped weak positive birefringent
Bader H

Similar recall : https://t.me/silent_2023m/3699


https://t.me/c/1243365384/2886

316- Child came with their daughter parents said she had a brain surgery and since
she had polyurea
and polydipsia. Labs there was significant hypernatremia 177 Potassium 4 Glucose
normal. Cause:

A- Diabetes insipidus
B-SIADH

Similar recall : https://t.me/smle22olds/3554


https://t.me/smlemay/8553

Bader H : Diabetes insipidus (DI) is a inadequate secretion or action of ADH > excessive loss of
water. = polyuria and polydipsia.

In children can occur due to trauma or tumors affecting the pituitary gland, where ADH is
produced

Hypernatremia, or hypernatremia level, is a common complication of DI


Potassium and glucose levels may be normal in DI because they are not directly affected by the lack
of ADH
317- Case of RupturedEctopic pregnancy, (diagnosis what not mentioned, how
much the risk of mortality?

318- Patient post ERCP for dilated


CBD. Temp: 38
HR: 80
RR: 18
BP: normal
WBC: 9000
What is the diagnosis?

A- Bacteremia
B- SIRS
C- Sepsis
D- Severe Sepsis

Smimilar recall https://t.me/SmleByHamza/417

319- UTI screening in pregnancy

12 weeks

320- Open book pelvic fracuture + blood in the meatus, most appropriate?

1. Folys
2. Retrograde urothrogram

Similar Q : https://t.me/silent_2023m/7084

321- What imaging best to dignose ( renal stone or cancer? I forgot)

1.us
2.CT
3. MRI

https://t.me/silent_2023m/7085

322- Adult with Hemolytic anemia hb 9 plt normal spherosytosi what to give?

1. Blood transfusion
2.Steroid

323- 60 years old with cough and hemoptysis cxr showed pleural effusion in the rt
side, what is the cause?

1.TB
2. Cancer obstructing rt bronchus
324- A child with mild respiratory infection increase symptoms & fever,
develop respiratory distress. X-ray shows lobar infiltration, CBC shows
low HB and high Retic.

A- SCA
B- Mycoplasma pneumonia
C- pneumococcal pneumonia
Bader H .

325- A patient known to have sickle cell disease presented to emergency


department complaining of both legs
pain for three hours. Lower limb exam is normal with no sign of DVT. The abdomen and
exam reveals
spleen enlargement. Hemoglobin is 3.2 which of the following is the best next step :

A. Splenectomy
B. Hydration
C. Morphine sulfate
D. Blood transfusion

Next step hydration and pain relieve then blood transfusion as I think

326- A neonate 12 hours after normal vaginal delivery noticed to be


jaundiced. The blood film showed microcytic hyperchromic cells.
The best investigation to order is:

A) Osmotic fragility test.


B) Pyruvate kinase level.
C) Glucose 6 phosphate dehydrogenase enzyme. D)
EMA.

Hereditary spherocytes

—————
—————

327- A child was given today BCG, his mother Concerned of TB infection as her
father who. Is living in the same house just diagnosed ?
1. Reassure as he is vaccinated 2.
Do PPD
3. Do IGRA and give INH
4. Do CXR and giver (INH, rifampin, ethambutol, and pyriziomide)

2 ⬛✓
‫تأكدت في آمبوس‬

Most patients: IGRA (preferred in those who have had the BCG vaccine) or TST=PPT
=Tuberculin skin test (purified protein derivative test, Mantoux test)

◆ Interferon-γ release assay (IGRA)

Similar recall:
Q: 2 years old boy who too*k BCG vaccine came with hiis mother because of his
grandfaather had open TB and they live in the same house, the boy asymptomatic but
the mother worried what should you do?

A- nothing since he took BCG vaccine


B- PPD, X-ray and wait for the result
C-IGRA, X-ray and give INH
D-IGRA, X-ray and give 4 TB drug

Similar recall , the answer is B

328- Pregnant with asthma what happens during pregnancy?

1. increase RR
2. Increase tidal volume
3. Increase resdual volume
4. Increase functional resdual capacity

-2!
UpToDate:
- RR remains unchanged
- Tidal volume is increased up to 40%
- Residual volume is decreased
- Functional residual capacity is decreased
329- Asthmatic what to give pre exsercise?

1. Salmeterol 2.
Salbutamol
3. Budesonide

330- Cardiac patient in her 20s ask about cotraception?

1. Progestrone pills
2. Estrogen progestron patches
3. Combined ocp
4. Bilateral tubal. Ligation

331- Women completed her family asks about endometriosis management one ovary
was removed before know have (cyst) endormeterioma in the second side. Most
appropriate ?
1. Remove cyst
2. Remove ovary and cyst
3. Hestrectomy and bilateral salpingioopherectomy 3,

completed family

332- Patient ask the doctor question about the disease, doctor Explained, then he felt
that she is confused and not getting information, he ask her again the question that
she asked, she answered, what type of patient education?

1. Teach back

333- Pregnant was in labored for 2 hours, then she deliverd the baby, after
placeta delivery she developed pph, cause?

1. Prolonged delivery 2.
Precipitous delivery
334- DNR from other hospital

1- valide for 24 hrs


2- not valid dont
remember

335- Patient was having primary dysmenorrhea affected her daily function, she
was given NSAID, she was able to fuction well but she is still tot satisfied, what to
do?

1. Education
2. Follow her wishes

336- Inication of tube thoracotomy :

1. glucose less than 60


2. 90 % neutrophils

337- UTI in pregnancy.

1. Augmentin
2. Floxacillin
3. Ciprofoxacin
4. Gentamycin

1 or 3

338- Pylonephritis in pregnancy?

1. Admit with iv Abx


339- SC trait pt , what she might have

antipartm

340-

341- Hx of asthma in labor with pph doctor did massage but didnt help she stil in
bleeding Which of the following contraindications is the most appropite managment
to her at this stage ?

A- oxytocin
B-misoprstol
C-methylergonovine
D- Carboprost

oxxytocin > managment ‫ حاسني ما ادري يبغى‬100% ‫ هن اية السؤال متأكد منها‬carboprost > contraindiction
mostappriopta Or

—————
—————

342- pregnant women smoking asking about most appropriate ( ◆vno mention of
stop smoking in q) ?

A- ask the mother to smoke less cigarettes per day B-


bupropion
C- nicotine patch
D-nicotine gum

Answer is A, counseling is the first line management in pregnant smokers, then nicotine gum if unable
to quit by herself.

Explained in details in the Substance Abuse & Smoking chapter notes

343- ectopic pregnancy absolute contraindication for MTX

pt was unstable
344- cottage cheese discharge

candida

345- most common cause for PPH

atony

346- placenta abruption case

diagnosis wanted

347- pregnant DM type 1 uncontrolled , whats the complication

348- hydrops fetalis , how to diagnose

anti kell abs

349- pregnant hx of DVT what to give

enxoparine

350- how to diagnosed rupture membrane

vaginal exam

351- pooling in post fornix what does it mean

ROM

352- vaccine of 9 months


353- 3 month mile stone

354- bronchilitos ( diagnosis )

355- what is the chest finding in bronchioloitis

wheeze

356- intusspetion ( most definitive way to

dignose ) 357- chalmydia


( conjuctivitis at2 weeks then pulm symotoms )

358- breast cancer mammo BIRAD 4 , whats next ?

359- Mastitis failed abx , sus abcess how to manage ?

360- Thyorid cancer ( papillary 2x3 mass , how to ttt )

361- tension PNX ttt

362- Female with strong family hx of breast cancer , came to do BRCA screen and her
employer wants the result
What do you do as a physician

1- tell her to send request to ethics committee 2-


refuse
3- tell her to give it to the employer if its positive
4- tell the employer to contact the ethics committee

363- IDA labs , what is the diagnosis


364- 12 hrs baby jaundice how to diagnose -

osmotic fragility test

365- HBV vaccinated pt no infection hx whats the serology ? Anti HBs⬛✓


Done by Naif.
Similar: https://t.me/smlemay/1577

Similar recalls⬛J:

Q: HBV serology like 3 qs came so know it well

Q: 518- 59 y/o male C/O jaundice for 2 month P/E enlarged liver nodule. CT : cirrhotic
liver with rt 4*5 nodule which of the following risk factor strongly associated with this
condition?
A-aflatoxin
B-lead toxin
C-wilams dis
D-HBV
⬛✓

Q: patient took 3 dose HBV vaccine and but not exposed to infection will find in his
serum
A-Hbc ab
B-hbe ab
C-hbs ab⬛✓⬛
D-hbsab +hbc ab
Q: PAN associated with: HBV⬛✓

Q: Which of the following will be positive in vacinated individual who never been
infected with HBV?
HBsA⬛

HbcA
HbsAg
Forgot

HBV 30%
HCV 3%
HIV 0.3%

‫كذا صباغة الرقم‬J⬛


0,03 OR 3%

*Similar recall:*
1- https://t.me/c/1243365384/5457
2- https://t.me/medandsurg21/15
3- https://t.me/smlemay/6454
4- https://t.me/smlemay/4383
5- https://t.me/SMLE2022_23QA/5552

*◆HBV 30%*
*◆HCV 3% *
*◆HIV 0.3%*

Case of transmission of HCV:


A: 0.3
B: 0.03
C: 0.003
D: 0.0003
liver with multiple lesions, i think hcc, what is a risk factor for this?
A. Aflatoxin
B. HBV ✓⬛
C. willson disease

Similar: https://t.me/silent_2023m/2732

Took 3 doses of HB and never infected of HBV


Answer : HB sAB⬛✓

518- 59 y/o male C/O jaundice for 2 month P/E enlarged liver nodule. CT : cirrhotic liver
with rt 4*5 nodule which of the following risk factor strongly associated with this
condition?
A-aflatoxin
B-lead toxin
C-wilams dis
D-HBV
⬛✓

499- delivered bab At 35 weak and the physican ask to put pationt vaccination schedule
WHEN we arrenge it?
A. As soon as discharge
B. after 2 weak of Current date of delivery
C. After 2 weak of expected date of delivery

Answer is: As scheduled according to chronological age except HBV given at discharge
or 1 month after birthday.

12 Nov

Full term delivered healthy baby


Which vaccine should be given before discharge :

A- HBV & BCG


B- HBV& DTAP
C- MMR & BCG
D-DTAP & BCG

#p ◆
Answer: HBV only
A

393- pt with hepatitis B transmission rate


A- 0
B- 0.03
C- 0.3
D- 0.003
Answer is C.

HBV 30% ( 0.3 )


HCV 3% ( 0.03 )
HIV 0.3% ( 0.003 )

279- Full term delivered healthy baby


Which vaccine should be given before discharge :

A- HBV & BCG


B- HBV& DTAP
C- MMR & BCG
D-DTAP & BCG

Answer is A.
Note: New guidelines recommends BCG at 6 months not at birth.
But A is closest.

◆ Similar , Very important:


1- https://t.me/BdranSpace/41
2- https://t.me/octuber2022/578
3- https://t.me/SmleKillers/3936
4- https://t.me/c/1614759886/1257
5- https://t.me/c/1556741712/267
6- https://t.me/c/1222499606/27
7- https://t.me/QuickRecall/421
8- https://t.me/smlemay/12987
9- https://t.me/SmleKillers/2189
10- https://t.me/SmleKillers/1441
11- https://t.me/smlemay/10039
12- https://t.me/IMCrashSMLE/2000
13- https://t.me/c/1243365384/5840
14- https://t.me/smlemay/7613
15- https://t.me/smlemay/6722
16- https://t.me/smlemay/3751
17- https://t.me/smlemay/1577⬛✓

Q: after delivery without issue give:

— 1- HBV vaccine⬛✓⬛
— 2-BCG + MMR
— 3-HBV + Dtab
— 4-BCG + Dtab

After delivery without issue give:


A- HBV vaccine
B- BCG + MMR
C- HBV + Dtab
D- BCG + Dtab

#p ◆
A

A 25 - year - old man sustains a percutaneous scalpel injury. History revealed, hepatitis
B but is not carrier. Hepatitis B Immunoglobulin is given within 24 hours and HBV
vaccination series is initiated at the same time. What is the adequate antibody (HBs)
response after 3 dose vaccination series to ensure immunity?
A. less than 5 mmU / ml
B. between 5-9 mmU / ml
C. between 9-12 mmU / ml
D. greater than 12 mmU / ml

#med ◆
Answer: D

Young male patient has a history of exposure to HBV infection , US was done and
showed multiple nodular lesions in his liver.
His PCR for HBV is negative, what is the next step?

No need for follow up


Abdominal and Chest xray
Alfa fetoprotein test
Anti Delta antibodies⬛✓

1 -‫ واحد يشتغل بمختبر وانجرح بأدوات مستعمله فيها هيبتايتس‬C ‫كم نسبه انتقاله للعدوى‬
0
0.3
0.03
0.003

#med ◆
C
HBV 30% = 0.3
HCV 3% = 0.03
HIV0.3% = 0.003

366- Inital investigation for DVT

367- Post op complications 5 questions


‫تقريبا‬
368- smoking cessation stages

369-

diagnosis about ask and ‫الصورة دي جات‬

370- Women has no background on htn was diagnosed with it whats the best you
to educate her about the risks and complications

1- health promotion
2- health model

primary ‫ما انذكر الخيارين الثان ين بس ما كانو‬


‫وال سكندري مو من ذلين‬
-
health promotion model?

371- 62 years old male complaining of painless hematuria upon examintaion there was
a palpable mass in the rectal examination of prostate most appropriate

1- cystoscopy
2- biopsy mass

372- Patient with ectopic pregnancy presented with andominal pain , the nurse called
the doctor he was busy and asked her to give medication to the patient

1- ask another nurse to approve 2-


give methotrexate
3- give paracetamol
4- Doctor cant prescribe over phone

373- Doctor agreed that the patient needs to abort the child due to
chromosomal anomalies , the patient agrees but the father refused

-doctors consent is enough


-mothers consent prevails
-fathers consent prevails

374- Bleeding in ct brain + papilledima,, definetive mangement?

1.manitol
2.Craniotomy

Also Another question about management of brain bleeding same choices but didnt mentioned
papilledema
375- Definitive ddx in Rheumatic fever?

1. ASO TITER
2. BLOOD CULTURE

376- X linked agamglubinemia diagnostic test?

377- Burn pt, they mentioned the burn area, you have to calculate TBSA, q : calculate
IV fluid according to parkland equation?

378- 71 year old man bed ridden , diabetic complaining of chronic constipation ,
presented with left abdominal pain , vomiting and constipation for 3 days , upon
examination abdomen is distended and tenderness mainly in the LLQ and empty
rectum, all labs are normal . X ray picture of hugely dilated colon ( not coffee bean)
What is the diagnosis

- appendicitis
- Sigmoid obstructing tumor
- Sigmoid volvulus

379- Old women presented with lower gi bleeding on examination diffusely tender
abdomen pr exam showing fresh blood patient is unstable despite receiving blood
Bp70/50
What is the most appropriate management:

- laparotomy
- Angiography
- Urgent colonoscopy

380- Patient presented with lower gi bleeding labs showing low hgb , on
examination there is a friable easily bleeds caulflower mass 2cm form anal verge
what is the diagnosis

- anal cancer
- Rectal cancer
- Anal fissure
- Condyloma lata
381- 37 year old lady presented with breast lump on examination the mass is hard
, irregular with skin tethering what is the best next step?

- core needle biopsy


- Needle aspiration
- Bilateral mammography
- Mri

382- Patient present with barking cough and respiratory distress , diagnosed ad
croup given in the Q , what will you hear in osculation

- diffuse decrease in air entry


- Expiratory wheeze with prolonged expiration
- Prolonged inspiration due to sub glottis narrowing
- Prolonged inspiration due to mucus in the big bronchus

383- Patient presented with perforated duodenum will undergo surgery what is the
most appropriate

- pyloroplasty
- Pyloroplasty with truncal vagotomy
- Simple suturing with momentum
- Partial gastrectomy

384- Patient presented with abdominal rigidity and will undergo laparotomy,
bp80/55 What is most appropriate

- central line colloid


- Peripheral line crystalloid

385- Mother is worried about her child being pale and doesn’t eat red meat but
drinks alot of milk labs showing low hgb and low mcv , what to give

- trail of oral iron and follow up


- Multi vitamins with iron
- Iv iron

386- Pregnant lady lactating doesn’t eat red meat what will be deficient in her child

- foilc acid
- Vit b6
- Vit b12
387- A lady that her husband died and wants to use her husband sperm in the
sperm bank to get pregnant , which is most appropriate

- she can use it with his family approval


- She cannot use it
- She can use it with the approval of 3 consultant
- She can use it if there is previous consent of the couple

388- Patient with RLQ pain on examination pressing on the LLQ elicits pain in the
RLL what is this sign called

- rovsing

389- Patient presented with upper abdominal pain and dyspnea , patient had surgery for
perforated duodenal ulcer 3 years ago x ray showing atelectasis and pleural effusion ,
labs normal wbc ,what is the best next step

- intravenous antibiotics
- Abdominal us
- Laparoscopy maybe??

390- Young patient recently married presented with upper pubic pain and dysuria
what is the treatment:

Oral nitrofurantion
Oral b lactam
Fluroquinilones

391- female presented with pruritus and jaundice labs shoeing elevated LFT , bilirubin
and alp and positive ANA
Liver biopsy showing plasma interface hepatitis what to give:

- prednisolone
- Methotrexate
- Ursodeoxycolic acid

“ +ve ANA ( SLE) + increase liver enzyme + jaundise “ >> autoimmune hepatitis
TTT>> iv steroid

Waleed
392- Patient had tonic colonic seizures and felt from the stair , after that patient
pelt severe back pain, urinary incontinence and fecal incontinence what is most
appropriate?

- steroids
- Mri spine

Waleed

393- Patient known case of hf and htn diagnosed as stroke and was started on heparin
and patient previous medications ( furesamide and acei ) during hospital stay patient
labs showing aki what is next

- stop heparin
- Stop acei
- Stop fursamide

“ drugs avoid in AKI ( NSAIDs , ACEi , ARBs )

Waleed

394- Patient admitted to icu diagnosed as invasive aspergillosis what is the treatment

- rifampin
- Valacyclovir
- Voricinazole

Waleed

395- Pregnant 24 w presented with decrease fetal movement us showing IUFD what
is the best next step:

- iv oxytocin
- Vaginal misoprostol
- D&c
- Intra cervical insertion of something??
ELAF

396- 32 year old g3p2 known case of hypothyroidism her baby diagnosed with
single umblical artery , what is risk?

- age
- Multi parity
- Thyroid disease
( no dm or black race)

seizure & HTN DM, , smoking ‫ ومني متآكده من صحة الجابة آلن المصادر متطرقة فقط ل‬710 ‫سؤال مكرر مت الجابه عليه في سؤال‬

ELAF

397- Boy circumcised presented with vomiting and fever since today fever
39c What supports the diagnosis of upper UTI

- male gender
- Vomiting
- Duration of fever
- Degree of fever.

fever of degree high ‫ انو فيها‬UTI upper ‫هيا الصح آلنو الي يميز ال‬
usually more than 38 degree
398- Patient known case of asthma and mr present led with fatigue and dysnea on
excretion, on examination there is diastolic murmur radiating to the axilla and chest
examination is clear , what is diagnostic

- spirometry
- Trans thoracic echo
- Trans esophageal echo
- Metacholine challenge test

ELAF
399- Patient known case of asthma on sulbutamol complaining of cough and nead
to clear the throat more at night and when lying down what is next
- add ling acting b agonist
- Add ppi
- Add inhaled corticosteroid
- Oral steroid

C from similar recall & chat GPT

ELAF

400- 9 months old with repeated attacks of apnea , color change , vomiting
and repeated upper respiratory infection
On examination patient neck is flexed with dystonic movement what is the diagnosis

- appendicitis
- Esophageal reflex

ELAF

401- Pregnant 30 weeks presented with increase vaginal discharge on


examination there is pooling in the fornix what is the diagnosis
- candidiasis
- Rupture of membrane
- Bacterial vaginosis

ELAF

402- Patient RTA with raised JVP , hypotension and quit heart sound , clear lungs
what is most diagnostic

- ECG
- Echo
- Xray
- Maybe CT

ELAF

403- Patient took a medication that interacts with another drug he takes how can
we prevent this

- computerized system
- nurse should check medication
- Patient have to brig his medication
- Pharmacist should look for interaction

404- Patient presented with left side hemiparesis , absent gag reflex and
fecal incontinence of tarry stool diagnosed as stroke what is the best next
step

- elevate patient head 30 degree


- Range of motion exercise for left side
- Stool analysis for occult blood
not sure
ELAF

405- Patient with hx of dip and pip and bilateral wrist pain , morning stiffness less
than 30 min on examination there is distal phalenx non tender nodule what is the
diagnosis Labs shows -ve RF and normal wbc
- poly arthritis gout
- Hand osteoarthritis
- Reactive arthritis
- Sero negative arthritis

ELAF

406- SCD under going bone transplantation developed neutopenia , how to manage ?

1- cefepimne
2- clindamycin
3- vancomycin

ELAF

407- Pt with Prosthetic heart valve undergoing dental procedure, which antibiotic
should be given ?

Ampicllin

ELAF
408- How to calculate to get gstational age

1- crown rump length

ELAF

409- Pregnant patient known case of asthma with postpartum


hemorrhage Which of the following medication is contraindicated to give?

Carboprost

410- Which of the following medication can propose to patent ductus arteriosus ?

Indomethacin
ELAF

411- Patient with a neck injury in zone three with active leading. How do you
manage? cta

412- Patient with neck injury in zone number one with surgical emphysema what is
the most appropriate investigation?

according to hamouds file :


Neck trauma
Zone 1 -> CT-A if +ve -> Endovascular repair
If +ve bronchogram or esophagram -> open repair
Zone 2 and 3 if asymptomatic -> observe
Zone 2 symptomatic -> immediate open repair
Zone 3 symptomatic -> CT-A if +ve -> endovascular repair
Unstable (expanding hematoma, uncontrolled hemorrhage) = ligation in all zone

Penetrating trauma - AMBOSS

413- married couple , husbend is Sickle cell trait , wife unknown , what is the risk
of their children having SCD ?

1- high
2- low
3- low if the wife tested -ve

414- Picture of down syndrome infant


Child born to 35 year old mother
Has hypotonia and other sx i Dont remember
What is the dx :

1- trisomy 21

415- Pt with hx of pulmonay embolism what contracption to use ?

IUD

wafaa file 2021

416- Lots of cases about endometrosis anx dymenorrhea ( dignosis and ttt

) https://www.amboss.com/us/knowledge/Endometriosis
Wafaa file 2021

417- Case of adenomyosis


What is the definative managment

hysterectomy in women that do not want to preserve fertility otherwise myomectomy


https://www.amboss.com/us/knowledge/Uterine_leiomyoma

418- Women with shortness of breath with minimal limitation what class is she ??

NYHA II

similar recall https://t.me/silent_2023m/1812


419- Diet in ascities

Low sodium Ascites


- AMBOSS

420- Child rides a tricycle and knows his name

3 years old
https://t.me/smlemay/17303

421- What is the favorable biophysical profile ?

8/10
wafaa2021

422- pt with hx of MI and high cholestrol what to do ?

1- diet
2- excerise
3- lipid lowering meds

● Initiate pharmacologic therapy based on the patient's age, LDL level, and
ASCVD risk.
○ Statins
○ Nonstatin lipid-lowering agents: may be added to statins if treatment
goals are not met.
● Lipid disorders - AMBOSS
● Atherosclerosis - AMBOSS (ascvd )
● Initial step is always low fat low salt diet ..

423- HTN on amlodipine and losartan , still uncontrolled what to add ?


Turkis file

424- Hyperkalemia case , labs mentioned with ECG picture , what is the ECG changes
seen ?

Peaked T wave

425- child with sudden squint ? Convergence squint


Whats the initial management ?

1- refer to neurosurgery
2- reassure
3- CT for suspected space occupying lesion 4-
refer to optha next day
426- How to diagnose
trachomalacia? bronchoscope
laryngomalacia * laryngoscope
Airway malacias - AMBOSS

427- Case of a patient who had itchy skin in scalp and multiple
areas (eczema), with recurrent infections, and thrombocytopenia.

A- Wiskot Aldrich Syndrome


Congenital immunodeficiency disorders - AMBOSS
https://t.me/SmleKillers/1498

428- Paracetamol toxicity stage, he take 20 taplets of 500mg before 24h, presented
to ER with high LFT, nausea, vomiting, RUQ pain?

A. Stage 1
B. Stage 2

Safdars file

429- Child came with a potentially toxic dose of paracetamol. The physician requests
an “Acetaminophen level”. The lab technician calls to report results. He says: “two”
stops and pauses and then says ”one, three”. The nurse mistakenly writes it as “2.13”
while the real result was “213”. The patient went into irreversible liver damage, which of
the following explain the error that has happened?

A) Doctor did not check B)


Lack of communication
C) Technician didn't check if the nurse got the number correctly
D) Mishandling from the nur

https://t.me/smlemay/16720 similar recall

430- Which drug is safe in CKD stage 3

1- nitrofuinton
2- lithum
3- metformin
4- cant remember

3. if the does adjust to the correct does but contraindicated in stage 4 and 5

431- Someone who is having THOUGHTS that aliens land in his backyard when he
leave his home.Although he knows that aliens does not actually exist, be he alsways
has this thoughts. He said to the doctor that these overwhelming thougts will make him
insane. Dx ?

A- Obsession
B- Compulsion
C- Delusions
D-hallucinations

https://t.me/FM98SMLE/799

Naif:
Aliens=
‫المخلوقات الفضائيه او مخلوقات غريبة‬

‫الجابه فت رق على حسب صياغة السؤال‬


https://t.me/psychsmlechannel/129 ‫يعني لو قال انه شافهم او متأكد من وجودهم بتفرق الجابة‬
432- Pt with Loin pain , hemturia , flank mass Most appropriate to diagnose ?

CT

initial would be ultrasound

◆ Focus here:
CT abdomen and pelvis without IV contrast
CT has the highest accuracy of the imaging modalities to identify kidney stones.

c3Indication: first-line for nonpregnant patients with suspected nephrolithiasis

Ultrasound abdomen and pelvis c3


Indications: suspected nephrolithiasis in patients for whom radiation exposure should be minimized
(e.g., pregnant patients, pediatric patients, those with recurrent stones)
433- 23 yo female pregnant 28wks with fetal hydrops peak systolic flow 1.8mom on
mca doppler cause ?

- anuplide
- diaphragmatic hernia
- anti-kell antibody
- cervical teratoma
https://t.me/silent_2023m/7181
https://t.me/smlemay/3216

434- Patient with asymptomatic subseroal fibroid 5 cm , what is the most appropriate?

Observe

Uterine leiomyoma - AMBOSS

435- Patient with ovarian cyst what is the most appropriate contraception?

ocps
wafaa 2021
https://t.me/WafaOBGYN/1775

436- Patient with premature preterm rupture of membranes what to give ?

Steroids and abx


https://t.me/silent_2023m/3928

437- Female underwent gyne surgery, massive bleeding occurred intraop and the
doctor and his colleague decided she needs emergency hysterectomy to save her life
What to do?

A. Get consent from husband B.


Do it without consent
C. Ethical committee consent
438- Case of woman of tubal ligation doctor discussion from where taken consent?

A Mother consent and inform husband


B- From both consent
C- From mother only
D- from husband

439- Management for COPD mMRC 2 (Modified Medical Research Council)?

A. Combined Salmetrol + fluticasone


B. Phosphodiesterase-4 (PDE₄) inhibitors + Salmetrol + fluticasone C.
Albutarol as needed + Salmetrol+ pulmonary rehabilitation
D. Albutarol as needed + fluticasone

Chronic obstructive pulmonary disease - AMBOSS

440- Steroids 15 mg daily and on hydroxy maintaing remission. A known case of RA.
what would u do? There was an increase of blood glucose or hba1c not sure which one
of them ( but there was an increase

1) taper pred and give metho


2) taper pred and give ibuprofen
3) taper pred and give cyclosporin

https://t.me/smlemay/16832

441- A 20 years old Female, came with sudden visual impairment and unsteady gait,
speech not affected, MRI showed demyelinating lesions peri-ventricular, diagnosis?

A-Multiple Sclerosis ◆
B-Transit ischemic attack
C-myasthenia gravies
D-stroke

442- Child came with painfull red swollen hemiscrotum. On examination, mass was
palpated with -ve cough impulse. The mass was tender and extended to the inguinal
area. Left testes cannot be palpable. Which of the following is the most likely
diagnosis?

A- testicular torsion
B- Epidydomorchitis
C- Incarcerated inguinal hernia
D- Testicular appendicular torsion
443- 45 years old male or female, has a mass 5 cm in right upper limb, (MRI shows
a mass from triceps) what to do next?

A. Incisional biopsy
B. Excisional biopsy C.
Core needle biopsy
D. PET scan

444- Pt k/c ulcerative colitis , presneted with 8 times bloody diarrhea and pain , what
to give ?

1- sulfasalazine
2- IV methylpredisolone
Sulfasalazine is an oral medication used in treating ulcerative colitis. works by reducing
inflammation in the colon and preventing flare-ups. However, but takes longer time.
IV methylprednisolone, is a more potent medication and can be given when the flare-
up is “ severe “ ‼️
It helps to reduce inflammation + suppress the immune response.

445- Tender neck mass after URTI


ESR high
Lab indicated hyperthyrodism
What to give ?

1 - steroids
2- methmazole
3- Iodine
4- propythiouracil
Ref : Chat gpt ‼️not sure

446- Mode of transmtion of meningitis ?

Droplet
447- Empirical ttt of bacterial meningitis?

Ceftriaxone and vanocmycin

448- Patient known case of epilepsy on sodium valproate presented to the emergency
department with recurrent convulsants. The conversio is ns could not stop what is the
most appropriate management

diazepam
phenytoin
Phenobarbital
thiopental
Not agree with diazepam

Because of diazepam is the 1st line.

Now pt on sodium valproate


, it’s the 2nd line

◆ Refractory status epilepticus (40–60 minutes):


- Options include repeat second-line therapy or induction of coma (e.g., with IV propofol,
thiopental, midazolam, or pentobarbital).

My opinion I will go with 3rd line

line 3rd ‫وذكر ب الختيارات نوع من ال‬

line ‫ونوع‬
‫ ونوعين‬1st
‫ الن‬line 2nd

‫الرأي كل م‬

449- carcinoid at tip of the appendix . What is the appropriate management?

A. Observation
B. Chemotherapy
C. Radiotherapy
D. Right hemicolectomy
Similar recall : https://t.me/QuickRecall/600
https://t.me/Medicine_ExplainedClearly/15060
https://t.me/SMLE_Surgery/328
https://t.me/Medicine_ExplainedClearly/15059
—————
—————

450- Meig’s syndrome how to treat

1- surgery and chemo


2- surgey and radio
3- radio

Another recall : https://t.me/c/1243365384/4770


451- 62 years old male complaining of painless hematuria upon examintaion there was
a palpable mass in the rectal examination of prostate most appropriate

1-cystoscopy
2- biopsy

452- Cardio case :


(pancystolic murmur radiate to axilla )

Mitral regurge

453- Presneted presneted chest pain ECG : elevated ST in leads II III aVf
He is known case htn , dm and has hx of non hemrrhagic stroke 2 months back ? Most
appropriate?

1- fibrinolytic
2- asprine
3- PCI

454- pt in icu with low COP and high wedge pressure , what type of shock ?

Cardiogenic

Cardiogenic shock occurs when there is impaired cardiac function leading to decreased
cardiac output and inadequate tissue perfusion.
‼ ️ the low COP indicates that the heart is not able to pump blood efficiently
while the high wedge pressure indicates that there is a build-up of blood within the
heart, leading to congestion and impaired function.

455- pt underwent ERCP , has fever , High WBCs high HR , HIGH LACTATE ,
pressure mainted on IV vasopressin , what ls the dx ?

- SIRS
- Spesis
- Septic shock
456- patient took 3 dose HBV vaccine and but not exposed to infection will find in
his serum

A-Hbc ab
B-hbe ab
C-hbs ab
D-hbsab +hbc ab

Not sure ‼️
457- Child with 7 cafe au lait spots 5 axillary freckles with postivie family
hx Dx ?

Neuofibromatosis type 1
458- A case of athletic patient with type one diabetes mellitus on insulin aspart and
glargine. Patient complains of progressive post exercise hypoglycemia despite eating
meals before exercise.
HbA1c: 7.0%
What would you do for this patient?

A) Reduce dose of aspart and continue on glargine


B) Switch aspart into sliding scale regimen and continue insulin glargine
C) Stop insulin aspart and continue on insulin glargine
D) Advice to increase protein intake before exercise and continue insulin therapy

459- Celiac pt non complaint to glutin free diet , on oral iron once daily .presented
with pallor and fatiuge labs low hg 9.7 what is the most appropriate

A-Oral iron tables 3 tome a day


B-Liquefied oral iron
C-Iv iron
D-Continuo same management

Bader H. : pt is presented with acute phase u have to go ABCD first “ qustion asked about most
appropriate “ means he needs urgent ttt then go with the first line ttt
460- Pregnant with pain , passage of tissue by US there is still tissue , open
os Whats the dx ?

incomplete abortion

461- Pt with 6 times abortion in 1st trimester whats the cause ?


1- genetic
2- anatomical
3- endocrine

462- Patient k/c of COPD alert, has moderate respiratory distress.SO2 93%,
ABG showed hypoxia, hypercapnia and acidosis. What is next step?

A. noninvasive ventilation
B Decrease oxygen
C. Increase oxygen
D. Mechanical ventilation

463- A drug approved by FDA is prescribed by a doctor who is the shareholder of


the pharmaceutical company. What is the problem

Confidentiality
Conflict of interest
Justice
Privacy

464- Pt's friend ask you about diagnosis of the patients disease You refused . What
is ethical concept?

A- privacy
B- confidentiality
C- dignity

465- full term born came with respiratory distress and chest X ray showed fluid in
the fissures. He is born by cesarean section. Most likely diagnosis:

A- Transient tachypnea of newborn B-


Sepsis
C- Meconium aspiration
D- Hyaline membrane disease

466- PT complaining of dizziness after waking up, hot warm and


flushes Watery diarrhea , itching
On examination Abdomen examination normal Respiratory exam reveal wheezing
Cardiac exam (murmur maybe) I’m not sure but there is finding What you will order ?

-Amylase and lipase level


-Us for the abdomen and the pelvis
-urine 5 hydroxyindolycytic acid
467- Patient with irregular period and and infertility husband semen analysis in
normal what is the diagnosis?

- ovulatory dysfunction
- Endometriosis
- Falopian tube problem

468- Patient prsented with 6 weeks amenorrhea and severe abdominal pain , on
examination abdomen is rigid , bcg is positive , what is most appropriate

- methotrexate
- Observation
- Laparoscopy
- Analgesics

469- Child with cough , rhinorrhea and conjuctivitis , on examination there is grey
spots with white top next to second molar what is the diagnosis?

- measles

470- Patient diagnosed with subserosal 2 cm fibroid asymptomatic but worried


about fertility what is most appropriate?

- myomectomy
- Reassure
- Hysterectomy

471- 7 year old obese presented with hip pain after trauma ( xray given) what is
the diagnosis

- Slipped femoral capital


- Femoral neck fracture
- Avascular necrosis

472- Patient known case of htn on captopril complaining of dry cough , what is give

- atenolo
- Enalapril
- Nefidipine
- Losaratn
473- Pregnant at 34week in clinic presented with high bp 140/75 , no proteinuria what
tk give

- L dopa
- Hydrlazine
- Nifidipine
- Mg sulphate

474- Patient presented with aub diagnosed with functional bleeding what to give

- ocp
- Endometrial ablation
- Hysterectomy

475- Research for group of cardiac patient half will recieve new drug and half
will receive placebo , what is there right

- withdral at anytime
- Explaining and knowing the research instrument
- Explaining the results or the procedure can’t remember

476- Pediatric known case of dm with recurrent dka , health educator and told her
the symptoms of dka “ child will need to urinate more than usual , will breath deeply
and will have fruity smell , the child will have tummy pain and vomit” what did he use

- teach back
- Plain language
- Critical language

477- Patient with dysmenorrhea took nsaid with partial releive patient is not
satisfied what to do

- diazepam
- Personal health education
- Tocolytic mediaction
- ( no ocp)

478- Newly married couple wife is worried because her husband is sickle cell trait
what are the risk for future child

- high risk
- Low or nill risk if patient is negative

479- 30 year old man presented with arthritis with morning stiffness and RF +ve ( case
going toward RA) what to give

- methotrexate
- Methotrexate with HCQ
- Methotrexate with azithio
- Methotrexate and HCQ and prednisolone

480- Copd patient presented with s3 gallop and bilateral crepitation po2 was
8 What will increase survival

- heart lung transplant


- Inhaled steroid
- Longs term oxygen therapy

Alaa: according to um alqura :

Other recall:

What improves survival or reduce mortality in COPD?

Smoking cessation

Copd cor pulmonale whats best thing to improve survival ?

A. oxygen mask therapy


B. inhaled long acting beta blocker
C.Long term 02 therapy
D.NPPV
E. Diuretic

481- Patient diagnosed with CAD what is the best lipid control?

- triglyceride <150
- Ldl <70

Alaa: Full recall

DM with cardiac disease, your goal in management?

A-triglyceride less than 150

B-BP less than 140/90

C- LDL less than 70

D - HDL more than 70

482- 62 year old male know case of htn , dm and dyslipidemia and with history of
stroke on chest examination : basal crackles and gallop , calculate chadvasc2

-3
-4
-5
-6

Alaa: similar recall

Chadsvas score
64 year old male with diabetes, HTN, stroke,
In examination he has S3, pulmonary crepitations
4-
5
2 trichomonas Q one asking about diagnosis ( red spots on cervix and flagellated organism under
microscope) and another about treatment metronidazole

Patient with pelvic pain and yellow vaginal discharge and flagellated protozoa, what is the
organism? another recall A female with vaginal discharge and in examination there is red
strawberry cervix

A. Chlamydia
B. Neisseria gonorrhoeae
C. Trichomonas vaginalis

Yellowish greenish vaginal discharge Examination erythema and itching Dx e trichomonas


vaginalis Treatment ?
A) Metronidazole
B) Cefuroxime

C) Ampicillin

- Yellow-greenish offensive discharge + Vaginal itching +/- Strawberry Cervix +/- pH

> 4.5 ➡ Trichomonas Vaginalis ( Trichomoniasis) ( Metronidazole )


——

483- niacin causing flush and subside with aspirin

Alaa:

similar recall

Patient with hyperlipidemia started on anti lipid then developed facial flushing, the

doctors prescribed to him aspirin to relivethis adverse effect:

A Niacin

B Atorvastatin C
Cholestyramine
484- Ejection systolic murmur radiating to carotid

aortic stenosis

Alaa:

similar recall

55 years old came presented with chest pain and SOB , OE : systolic ejection murmur with no radiation
. ECG ST and T wave non specific changes What this pt has?
A. conestrtive cardiomyopathy
B. dilated cardiomyopathy

C. AR

D. AS

55+ M Known HTN Came with heart murmur radiating to carotid region.. What’s the best
diagnostic method?

A/ trans esophageal echo B/


ct angiography
C/ cardiac catheterization
D/ ecg

30 wk pregnant came for check up , Have mid systolic murmur radiate to carotid in left eternal border
with no diastolic sound
— physiological murmur of pregnancy
— AS
— MS
485- Soft diastolic crescendo decresendo murmur at the left sternal border

aortic regurge

Alaa:

similar recall

Young had diastole murmur heard at the left sternal border which increase when patient leaning
forwarded?
A. Mitral stenosis
B. Mitral regurgitation

C. Aortic regurgitation
D. Tricuspid regurgitation

23 years old female with history of ASD, found to have decrescendo early diastolic murmur 2/6 on the
left lower sternal border. what is the most likely diagnosis?

A. Mitral stenosis
B. Aortic regurgitation
C. Tricuspid regurgitation D. Aortic stenosis
The answer is B

Note:

The typical murmur of aortic regurgitation is a soft, high-pitched, early diastolic decrescendo murmur
heard best at the 3rd intercostal space on the left (Erb’s point) on end expiration, with the patient sitting
up and leaning forward.

486- 9 months old did not receive any vaccine what to give this visit?

487- RUQ pain after fatty meals, no fever , no jaundice >


gall stone colic

Alaa:

similar recall but asking about the management

Typical case of biliary colic: middle age women with severe RUQ pain radiates to right scapula ass/w
nausea and vomiting. Triggered by eating heavy meal lasting for 3 hours then resolved spontaneously.
All labs were normal. US showed multiple small stones with normal Gallbladder wall. Which of the
following is the most appropriate management?

A- Observation
B- MRCP
C- Abx
D- Laparoscopic cholecystectomy

Answer: D

Note from Dr.Hamzah

Cholecystitis
History of biliary colic after fatty meal, fever, and RUQ pain. No jaundice. No CBD dilatation.
Positive Murphy’s sign.
The golden period for cholecystectomy is in the first 72h, but if the patient is past it manage
conservatively then schedule for elective lap chole in 6 weeks.

488- 15 year old low hgb low mcv , mentzer exactly 13 iron deficiency anemia
or thalassemia minor?

Alaa:

similar recall

Long case Pediatric patient with pale, pica and signs of anemia. In addition they mentioned his two
siblings has the same condition. Labs: microcytic hypochromic anemia Ferritin: 9 Reticulocyte: 3%
What is your diagnosis?
A- IDA

B- SCA
C- Alpha thalassemia trait
D- Anemia of chronic disease
..‫في األرقام ٔاو الرينج الصح حقها ًطيب ذا الس ٔوال ٔاكيد فيه شيء ناقص سواء‬

:‫ٔاعتمدوا على الرينج ا لي في األختبار‬+ ‫ بعلمكم كيف تفرقون بينها بحيث تكونون فاهمين‬TIBC high + ferritin Low >
Note:

with IDA.

Normal or high ferritin + normal or low TIBC > with thalassemia .

High retics> with thalassemia. Normal or low retics > IDA. Normal

RDW > thalassemia High RDW > IDA

Family history plays an important role in diagnosing patients with clinically silent thalassemia . Alpha

thalassemia trait: mild hemolytic anemia with normal RBC and RDW(Amboss).

So, if the ferritin was normal according to the range in exam> thalassemia for sure . If retics
(3%) considered normal according to the range in exam > IDA .

thalassemia > :‫ ممكن يفيدكم‬RBCs‫اذا اعطاكم في اإلختبار عدد‬

RBCs Higher
Low RBCs > IDA

489- patient with dysmenorrhea and infertility, diagnosis?

endometriosis

Infertility for 6 y with severe dysmenorrhea not treated by naproxen:


A. Endometriosis
B. endometritis
C. Leiomyoma

Q: infertality , dysmenorrhea, dychezia , sypaniya?


-Endometriosis

- infertality , dysmenorrhea, dychezia , sypaniya? Endometriosis

Summary important keys :


Dysmenorrhea + pain just associated with period —> premenstrual dysmorphic

disorder
Dysmenorrhea + pain with period + mood change or anxiety —-> tension menstrual

syndrome
Dysmenorrhea pain associated with period and sexual —-> secondary

dysmenorrhea
Dysmenorrhea + infertility —> endometriosis
Dysmenorrhea + previous CS + multiple pregnancy —> pelvic congestion syndrome

Dysmenorrhea + symmetrical uterus enlargement or previous history of myoectomy ( history of forbid )


—> adenomiosis

———

Pathophysiology

- Pre-renal (If prolonged will lead to acute tubular necrosis)

- Increased BUN:creatinine ratio FENa


(%) (fractional excretion of Na) < 1%

- Renal

- Post-renal (Obstruction at any site from renal tubule to the urethra)

- Decreased BUN:creatinine ratio FENa


(%) (fractional excretion of Na) > 2%

——— Acute kidney injury:-

Pathophysiology
- Pre-renal (If prolonged will lead to acute tubular
necrosis) Increased BUN:creatinine ratio
FENa (%) (fractional excretion of Na) < 1%
- Renal

-Post-renal (Obstruction at any site from renal tubule to


the urethra) Decreased BUN:creatinine ratio FENa (%)
(fractional excretion of Na) > 2%

490- 62-year-old male admitted to hospital with pneumonia and


received antibiotic, developed acute kidney injury, urine analysis
showed
muddy brown casts, urine Na more than 20, which of the following is
most likely diagnosis?

A- Acute interstitial nephritis


B- Rhabdomyolysis
C- Acute tubular necrosis
D- Tumor lysis syndrome

Alaa:

Acute tubular necrosis:-

- Due to drug, toxin or prolonged pre-renal state

- Most common cause of hospital acquired AKI

- Presence of Muddy brown cast or granular cast

- Urine Na > 40

- FeNa > 2%

Urine microscopy (very important):-

RBCs Cast or dysmorphic red cells indicates glomerulonephritis


Muddy brown or granular cast in ATN

Hyaline cast in pre-renal causes

WBC cast indicates infection or interstitial nephritis Fatty

cast in nephrotic syndrome

Waxy cast in chronic kidney disease

491- Male patient with decrease urine output, hypocalcemia and very
high uric acid level above 900, creatinine is about 600, what is the
cause of this presentation?

A- Hypothyroidism
B- Urate nephropathy
C- Diabetic nephropathy
D- Membranous nephropathy

492- Patient recently took a drug, now developed rash, high


eosinophil and acute kidney injury, urine showed WBCs cast, what is
the most likely diagnosis?

A- Acute tubular necrosis


B- Pyelonephritis
C- Pre-renal acute kidney injury
D- Acute interstitial nephritis

—— Acute interstitial nephritis

- Triad of eosinophilia, fever and rash

- Presence of pyuria and eosinophiluria

- Review patient’s medications list

- Eosinophiliuria DDx:- AIN and cholesterol emboli


493- Young male patient came with acute kidney injury, raised creatinine
and BUN level, muscle pain and tenderness what is the most likely
diagnosis?

A- Tumor lysis syndrome


B- Pyelonephritis
C- Lupus nephritis
D- Rhabdomyolysis

—— Rhabdomyolysis

- Dark tea color urine with dipstick +ve for blood but no erythrocyte on UA

- Following heat exposure or crush injury

- High CK level

- Treatment:- IV fluid

494- Patient survived from crush injury in RTA, presented with high CK
level, what measure you can give to prevent AKI?

A- Sodium bicarbonate
B- IV Fluid
C- Antibiotics
D- N-acetylcystine

495- Prevention of Dye induced nephropathy in patient known case of


HTN Going to PCI and he is on lisinopril?

A- Sodium bicarbonate
B- Isotonic solution
C- Dialysis
D- N-acetylcystine
—— Contrast induced nephropathy

- Increased serum creatinine within 24 to 48 hours following


contrast exposure

- High risk for CIN:- recent AKI, eGFR < 30

- Only approved prophylaxis is IV 0.9 saline and use of iso-


osmolar or low osmolar contrast

- No benefit of N-acetylcystine, dialysis post-contrast or sodium


bicarbonate infusion

|| .. Chronic kidney disease .. ||

496- Most common cause of death in ESRD?

A- High blood pressure


B- Uncontrolled DM
C- Hypotension during HD
D- Cardiovascular diseases

497- How to differentiate chronic kidney disease from acute kidney injury?

A- High potassium level


B- Creatinine level
C- Presence of metabolic acidosis
D- Ultrasound renal showing decrease size in CKD

Small kidneys on ultrasound suggest chronicity except in:- 1-


DM
2- HIV
3- Polycystic kidney disease
4- Amyloidosis

The kidneys will preserve its size

498- Patient with an uncontrolled HTN, which one of the following can
delay progression to ESRD?

A- Restricted protein diet


B- BP control
C- Early administration of HD
D- High salt diet

499- CKD patient with GFR of 35 on Celecoxib and metformin came


to clinic for follow-up, which drug you should stop?

A- Stop celecoxib
B- Stop Metformin
C- Stop both
D- No need to discontinue any of them

Metformin is contraindicated to be started for


patient with GFR < 45

BUT if he is already on it and GFR is


declining gradually you can resume it till
GFR become < 30 then stop it

500- 55-year-old female with 5 years history of DM and HTN with good
control, all laboratory work-up within normal except for Cr:117,
what is the next investigation?

A- HbA1c
B- Urine albumin
C- Ambulatory BP
D- Kidney biopsy

Corrected by abdulrazaq alshammari


according to UTD : test for diabetic nephropathy screening is Urine albumin concentration and
Albumin-to-creatinine ratio

501- Patient with renal failure and DM, what is the best oral
hypoglycemic drug for him?

A- Metformin
B- Sitagliptin
C- Empagliflozin
D- Gliclazide

According to UTD : HF and/or DKD – For patients in whom HF or DKD (albuminuria [urine albumin
excretion >200 mg/day] and estimated glomerular filtration rate [eGFR] <60 but ≥20 mL/min/1.73 m2) [64]
predominate, we prescribe a low dose of an SGLT2 inhibitor (empagliflozin, canagliflozin, dapagliflozin).
Choice of agent is primarily dictated by provider preference, insurance formulary restrictions, eGFR, and
cost. In the setting of declining eGFR, the main reason to prescribe SGLT2 inhibitors is to reduce
progression of DKD.
corrected by abdulrazaq alshammari

|| .. Glomerular diseases .. ||

502- Patient presented to clinic with proteinuria + high cholesterol


+ edema, what is the most likely diagnosis?

A- Nephrotic syndrome
B- Nephritic syndrome
C- Chronic liver disease
D- Congestive heart failure corrected
by abdulrazaq alshammari
According to UTD : When to suspect nephrotic syndrome — Nephrotic syndrome is generally
suspected in children or adolescents who present with new onset edema. Nephrotic syndrome may also be
identified in patients who are being worked up for proteinuria or hypoalbuminemia.
Hyperlipidemia or hypertension may also prompt evaluation for nephrotic syndrome.

503- 14-year-old boy with anasarca, proteinuria (picture of minimal change


disease), what kind of casts you will see on urine microscopy?

A- Fatty
B- Waxy
C- Hyaline
D- Muddy brown cast
According to Amboss : Urine sediment microscopy [27]

● Nephrotic sediment
○ Lipiduria, fatty casts with Maltese cross appearance under polarized light

corrected by abdulrazaq alshammari

504- Patient presented with high blood pressure readings, upon


further history found to have hematuria, urine sample taken for
microscopy resulted on 6-10 dysmorphic RBCs, what is the most likely
diagnosis?

A- Cystitis
B- Pyelonephritis
C- Urethritis
D- Glomerulonephritis

According to Amboos :
Proteinuria (< 3.5 g/day) (can be in nephrotic range in severe cases )
● Hematuria with acanthocytes

● RBC casts in urine

● Mild to moderate edema

● Oliguria

● Azotemia

● Hypertension

● Sterile pyuria

corrected by abdulrazaq alshammari


505- 32-year-old male with high renal profile with palpable lower limb skin
rash and abdominal pain 2 days following an episode of URTI, protein
in urine showed nephritic range, what is the most likely diagnosis?

A- Post-streptococcal glomerulonephritis
B- IgA Nephropathy
C- SLE nephritis
D- Alport syndrome

According to Amboss : Recurring episodes of:


○ Gross or microscopic hematuria
○ Flank pain
○ Low-grade fever
○ And/or nephritic syndrome (including hypertension)
○ Usually during or immediately following a respiratory
or gastrointestinal infection [6]

IgA nephropathy presented within 3 days of URTI

Post-infectious glomerulonephritis —> 10 days after a streptococcal throat infection or longer after a
skin infection

corrected by abdulrazaq alshammari

506- 2 weeks after URTI patient came with hematuria and nephritic
syndrome picture, what is the most likely diagnosis?

A- Post-streptococcal glomerulonephritis
B- IgA Nephropathy
C- SLE nephritis
D- Alport syndrome

According to Amboss : Symptoms occur approximately 1–6 weeks following an acute


infection ( URTI , SKIN INFECTION)
● Nephritic syndrome
○ Hematuria: tea- or cola-colored urine
○ Hypertension: can lead to headaches
○ Edema (prominent facial edema)

corrected by abdulrazaq alshammari

507- Microscopic finding on renal biopsy of post-streptococcal


glomerulonephritis?

A- Thickening of basement membrane


B- Diffuse mesangial proliferation
C- Subepithelial hump
D- Foot process from podocyte

According to Amboss : Electron microscopy: immune complexes between the epithelial cell
layer and the glomerular basement membrane (referred to as subepithelial humps)
corrected by abdulrazaq alshammari

508- Patient was found to have elevated renal profile and hematuria, he
has family history of multiple members in the family with renal failure
on HD, on examination:- bilateral hearing loss, what is the most likely
diagnosis?

A- Granulomatosis with polyangiitis


B- Eosinophilic granulomatosis with polyangiitis
C- Alport syndrome
D- IgA nephropathy

According to Amboss : Initially intermittent gross hematuria (may present in infancy)


● As glomerular damage progresses, symptoms of nephritic syndrome and
chronic kidney disease occur (usually leads to ESRD between 16–35 years of
age) [1]
● Sensorineural hearing loss
● Ocular findings: retinopathy, anterior lenticonus

side note : Alport syndrome

Sensorineural hearing loss + Ocular findings (lenticonus)


- In presence of family Hx of ESRD
- Inherited defect in collagen Type 4
corrected by abdulrazaq alshammari

509- 22-year-old female presented with malar rash, joint pain, renal
impairment, urine shows RBC casts and nephritic syndrome range
of protein, which of the following is the most likely diagnosis?

A- Alport syndrome
B- IgA nephropathy
C- Minimal change diseases
D- SLE nephritis
Corrected by abdulrazaq alshammari
According to UTD : Kidney disease is typically detected in most patients with systemic lupus
erythematosus (SLE) by an abnormal urinalysis with or without an elevated plasma creatinine concentration.
The most frequently observed abnormality in patients with lupus nephritis (LN) is proteinuria [1-4]. Other
common clinical manifestations include microscopic hematuria with or without red cell casts, kidney
function impairment, nephrotic-range proteinuria or nephrotic syndrome, and hypertension [1]. As a result,
all patients with SLE should be routinely monitored for kidney disease # so i think the recall mean nephrotic
range

( Kidney manifestations of SLE )

Immune complex mediated glomerulonephritis

One of the main determinants of prognosis

Proliferative glomerulonephritis (Heavy hematuria, proteinuria, cast)

Kidney biopsy is mandatory! And it is the next step in lupus patient with
proteinuria and low C3, C4

510- RBC casts on urine microscopy and normal complement, all can cause this
except?

A- FSGN
B- Membranous GN
C- IgA nephropathy
D- Lupus nephritis

corrected by abdulrazaq alshammari

Causes of glomerulonephritis associated with low serum complements:-


Post-infectious glomerulonephritis

Subacute bacterial infection, especially endocarditis

Systemic lupus erythematosus

Cryoglobulinemia

Mesangiocapillary glomerulonephritis

511- 40-year-old male presented to ER complaining of hemoptysis


post URTI, ANA done was negative, C3 and C4 levels are normal with
proteinuria on urine analysis, what is the most likely diagnosis?

A- Alport syndrome
B- Lupus nephritis
C- Good posture disease
D- Post streptococcal glomerulonephritis

corrected by abdulrazaq alshammari

Anti-glomerular basement membrane disease:-

Antibodies against the α3 chain of type 4 collagen GBM (in


basement membranes of glomeruli and lungs)

Rapidly progressive glomerulonephritis or lung hemorrhage If


both organ affected → Goodpasture’s syndrome

Treatment:-
plasma exchange + glucocorticoids and immunosuppressants

512- Diabetic patient has non-proliferative retinopathy, his creatinine was


elevated, 24 hours urine collection for protein revealed nephrotic
range proteinuria, what is the cause of his renal impairment?

A- FSGS
B- HIV nephropathy
C- Minimal change disease
D- Diabetic nephropathy
corrected by abdulrazaq alshammari
according to amboss : diagnosis is based on the presence of albuminuria and/or reduced
eGFR and the exclusion of other causes of CKD.

513- Type of glomerulonephritis in Hepatitis C patient with


cryoglobulinemia?

A- Membranoproliferative GN
B- Membranous GN
C- FSGN
D- Diabetic nephropathy
corrected by abdulrazaq alshammari
According to Amboss : MPGN associated with HBV, HCV, and cryoglobulinemia

514- IV drug user with history of axillary mass found to have proteinuria 1.5
Gm, renal US shows kidney size of 13 cm, what is the diagnosis?

A- HIV nephropathy
B- Focal segmental glomerulosclerosis
C- Amyloid nephropathy
D- Minimal change disease Corrected
by abdulrazaq alshammari
# this is tricky question as we know HIV is associated with FSGS but here as you see in the Q the
proteniuria is 1.5 which is not nephrotic range here we call the condition as HIV nephropathy

|| .. Hypertension and renal diseases .. ||

515- Patient diabetic and having proteinuria, which drug is the best for HTN?

A- Calcium channel blocker


B- Thiazide
C- ACE inhibitor
D- Beta blocker

corrected by abdulrazaq alshammari

Keywords for HTN in MCQs:-

1- ACEI + ARBS are contraindicated together

2- ACEI + ARBS during pregnancy are contraindicated


3- ACEI is contraindicated in bilateral renal artery stenosis but can be used if unilateral

4- HTN + DM = ACEI

5- HTN + CKD = ACEI

6- Beta blockers should not be used as a 1st line ttt except in case of CAD

7- Avoid using CCB in patient with heart failure

8- Labetalol is the best for HTN in pre-eclampsia

516- Patient newly diagnosed hypertension on many medications and still not
controlled, diagnosed as secondary HTN, on examination there is bruit near the
umbilicus, what is the most likely cause of HTN?

A- Cushing syndrome
B- Addison disease
C- Conns syndrome
D- Renal artery stenosis

corrected by abdulrazaq alshammari

Renal artery stenosis

Fibromuscular dysplasia + early onset hypertension


(especially in women) + Atherosclerotic stenosis +
flash pulmonary edema + rapid deterioration of RFT
after using of ACEI + bruit near the umbilicus

517- Male patient with HTN and unilateral renal artery stenosis, what is the best
medication for this patient regarding HTN?

A- ACEi
B- CCB
C- Hydralazine
D- Methyldopa

Corrected by abdulrazaq alshammari


According to UTD : The high angiotensin II levels induced by reduced renal blood flow act to preserve the
GFR by preferentially increasing the resistance at the efferent (postcapillary) glomerular arteriole. Inhibiting
this compensatory response with an ACE inhibitor or ARB can lower GFR in the post-stenotic kidney, even
though the associated decrease in renal vascular resistance
may preserve renal blood flow

ACEI is contraindicated in bilateral

518- Patient with ESRD on HD and found to have difficult in controlling blood
pressure, which drug causing this?

A- Sodium bicarbonate
B- Empagliflozin
C- Erythropoietin
D- Iron supplement

corrected by abdulrazaq alshammari


According to UTD : The most common side effects of EPO therapy, aside from hypertension and
its related problems, are headache

|| .. Electrolytes imbalance .. ||

519- Patient with DM has history of diarrhea then developed muscle


paralysis, on examination there is tender abdomen, potassium level is
2.8, what is the next level to check?

A- Urine potassium B-
Serum magnesium
C- Serum bicarbonate
D- Urine potassium

corrected by abdulrazaq alshammari


According to UTD : Patients with hypokalemia may also have hypomagnesemia due to concurrent
loss with diarrhea or diuretic therapy or, in patients with hypomagnesemia as the primary
abnormality, renal potassium wasting

520- 80-year-old male admitted to hospital for elective cholecystectomy, found to have
potassium level of 6.6, which of the following medications can increase potassium
level?

A- Amiloride
B- Salbutamol
C- Insulin
D- Lasix

corrected by abdulrazaq alshammari


Drugs causing high K level:-
β blockers ACE
inhibitors
ARBs
Calcineurin inhibitors (cyclosporine)
Spironolactone
Eplerenone
Amiloride
Heparin
NSAIDs

521- Patient presented with normal BP + hypokalemia + hypomagnesemia + low urine


calcium, what is the most likely diagnosis?

A- Gitelman syndrome
B- Bartter syndrome
C- Liddle syndrome
D- Nephritic syndrome

Corrected by Eman
Urinary calcium excretion is increased in Bartter’s
syndrome but decreased in Gitelman’s syndrome

Liddle syndrome → Hypertension


Gitelman’s + Barter syndrome → Normotensive
522- 69-year-old male with IHD on Lisinopril and Bisoprolol, found to have
potassium level of 10, what is the next step?

A- IV calcium gluconate and ECG


B- IV insulin and dextrose
C- Sodium bicarbonate
D- IV Lasix

Ca gluconate given if : 1- ECG changes regardless the level


2- k level > 6.5 even without ECG changes

Corrected by Eman

523- 52-year-old male presented with nausea, vomiting, diarrhea for 1


week came dehydrated and Na level is 130, what is the cause of hyponatremia?

A- Hypervolemic hyponatremia
B- SIADH
C- Hypovolemic hyponatremia
D- Euvolemic hyponatremia

Corrected by Eman

524- 58-year-old male K/C of dilated cardiomyopathy on


medications, presented with confusion, he was euvolemic upon
examination, sodium level was 124, serum osmolality was low, urine
osmolality > 300, what is the most likely diagnosis?

A- SIADH
B- Diabetes insipidus
C- Hypervolemic hyponatremia
D- Hypovolemic hyponatremia

Corrected by Eman

525- Patient having sodium level of 112 with history of confusion with seizure, what
is the appropriate treatment?

A- Fluid restriction to 800 ml/day


B- 0.9% saline with targeted sodium to return to normal C-
3% saline with target of increase Na by 0.5 meq/hr D- 0.9%
saline with IV lasix

Management of hyponatremia
- Hypovolemic à 0.9 % NS
- SIADH → Fluid restriction
- Symptomatic → 3% saline Corrected
by Eman

526- Patient known case of cancer with metastasis treated by chemotherapy


presented to your clinic for follow up, he was drowsy but was vitally stable on RA and
the laboratory results showed Na
level of 111, how you will manage this patient?

A- IVF NSS
B- Restrict the water intake
C- Increase water intake
D- D5W

Not enough information about the type of hyponatremia , drowsy and very low Na (panic value below
120 requires correction) suggest that the answer is NS, even though chemotherapy agent may cause
SIADH and in this case the answer will be fluid restriction , if the Q had no
further info i would go with IV NS. double check

Corrected by Eman

|| .. Acid-base imbalance .. ||

527- 22-year-old male found to have the following ABG results, Ph: 7.32,
PCO2: 36, HCO3: 10, what is your interpretation?

A- Respiratory alkalosis only


B- Metabolic acidosis only
C- Metabolic acidosis mixed with respiratory acidosis
D- Metabolic acidosis mixed with respiratory alkalosis

According to winter formula: (1.5*10)+ 8 = 23


the compensated pco2 value should be (23+/-2), pco2 in the Q is normal and not
compensated
Corrected by Eman

Also:
Q: 45-year-old male found to have the following ABG results, Ph: 7.39,
PCO2: 16, HCO3: 10, what is your interpretation?

A- Respiratory alkalosis only


B- Metabolic acidosis only
C- Metabolic acidosis mixed with respiratory acidosis D-
Metabolic acidosis mixed with respiratory alkalosis
Same as the previous Q pco2 should be (23+/-2) here it is lower than the expected so it is not a
compensation.
Corrected by Eman

528- Female patient brought to ER with overdose of unknown drug,


Na: 138, HCO3: 10, Cl: 100, Ph: 7.32, PCO2: 16, what is your interpretation of the acid-
base disturbance?

A- Normal anion gap metabolic acidosis


B- High anion gap metabolic acidosis with respiratory alkalosis C-
Respiratory alkalosis only
D- High anion gap metabolic acidosis with respiratory acidosis

To calculate the anion gap : Na - (cl + Hco3) Normal value is 10+/- 2


So in this Q it will be 138 - ( 100+10) = 28 ,it is more than 12 so this is high anion gap
metabolic acidosis (HAGMA)
Low pco2 indicates respiratory alkalosis.

Corrected by Eman

529- What is the anion gap for the following basic screen, Na: 121, K:
3, Cl: 92, HCO3: 15?

A- 12
B- 20
C- 15
D- 14
121 - (92+15) = 14
Corrected by Eman

530- Normal anion gap metabolic acidosis is caused by which of the following?

A- Renal tubular acidosis


B- Lactic acidosis
C- Uremia
D- Diabetic ketoacidosis

DDx of high anion gap metabolic acidosis:-


MUD-PILES
DDx of normal anion gap metabolic acidosis:-
HARD-UP

Corrected by Eman

531- Question about triple acid base disorder ..


Answer was:-
Raised AGMA, respiratory acidosis with
Metabolic Alkalosis

Key points to solve Triple acid-base disorders:-

- There is only one respiratory disorder in each ABG


-Once you have metabolic disorder → calculate anion Gap
- Once you have High AG acidosis then consider that may be Triple disorders
- Once you see High AGMA → measure Delta ratio to exclude triple disorders

If Delta ration is 1-2 → Only High AGMA If


Delta ration is < 1 → HAGMA + NAGMA
If Delta ratio is > 2 → HAGMA + metabolic alkalosis
N.B:- Delta ratio:-
(measured AG – normal AG) / (normal HCO3 – measured HCO3)

Corrected by Eman
532- Diabetic patient presented with picture of DKA, VBG provided with
metabolic acidosis, low delta-delta gap and respiratory alkalosis, which of the
following is the acid base disturbance in this patient?

A- High anion gap metabolic acidosis only


B- HAGMA + NAGMA + Respiratory alkalosis
C- HAGMA + NAGMA + Respiratory acidosis
D- Respiratory alkalosis only

Depending on the delta gap if <1 it is B If


it is 1-2 it will be A
Corrected by Eman

533- 30-year-old male homeless who is having drinking alcohol with


hepatomegaly presented with abdominal pain and his laboratory
results showed high anion gap metabolic acidosis and in urine
microscopy there is calcium oxalate crystals, what is the most
likely substance?

A- Salicylate poisoning
B- Ethanol toxicity
C- Methanol toxicity
D- Ethylene glycol toxicity

Alcohol poisoning
- All has osmolar gap and CNS depression
- Ethanol:- flank pain, hematuria, oliguria
- Methanol:- vision loss, treated with Fomepizole, folic acid and dialysis if severe
- Ethylene glycol:- Anti-freeze, AKI, calcium oxalate crystal in urine, treated with Fomepizole and
dialysis if severe

Corrected by Eman
|| .. Miscellaneous .. ||

534- What is the most common stone with Proteus infection?

A- Calcium oxalate stone B-


Triple phosphate stone C-
Uric acid stone
D- Cystine stone

Nephrolithiasis
Hyperparathyroidism + hypercalciuria, Hx of Roux-en-Y surgery → Calcium oxalate
Metabolic syndrome + gout → Uric acid
Strong family Hx + young age at onset → Cystine
Chronic UTI with urea splitting organism + staghorn calculi → Struvite or Triple phosphate or
(calcium magnesium ammonium)
Corrected by Eman

535- How to prevent renal stone?

A- Increase calcium diet


B- Increase fat diet
C- Increase oxalate diet
D- Increase protein diet
Corrected by Eman

536- 32-year-old male newly involved in hemodialysis, presented with


dizziness, vomiting and nausea after first hours of HD session, what is the most likely
diagnosis?

A- Electrolytes imbalance
B- Hypoglycemia
C- Hypokalemia
D- Disequilibrium syndrome

Dialysis Disequilibrium Syndrome


Range of systemic and neurological symptoms occurs during dialysis especially in patients in whom
dialysis is initially started
— Nausea, vomiting, headache, restlessness, seizure, coma
— Due to rapid shift of urea causing rapid lowering in serum osmolarity

Corrected by Eman

537- Difference between cystitis and pyelonephritis in urine analysis?

A- WBCs cast
B- Bacteria
C- Nitrite
D- Protein

Corrected by Eman

538- 39 year old male had a pelvic fracture from MVC rollover 2 months back
that treated surgically. Before 2 hours he started to complain of sever left LL
pain.
Femoral pulses are intact. I.V heparin started and venous US is negative. What is the
the best management?

A. Catheter thrombolysis
B. Embolectomy
C. Observation
D. Amputation

Suspected DVT case ( immobility due to pelvic fracture and intact pulse) , catheter thrombolysis
used in case of acute on top of chronic PAD , embolectomy used in cases of acute limb ischemia
due to emboli as in AF , no indication for amputation.
Corrected by Eman

539- Patient with ectopic pregnancy presented with andominal pain , the nurse called
the doctor he was busy and asked her to give medication to the patient

1- ask another nurse to approve 2-


give methotrexate
3- give paracetamol
4- Doctor cant prescribe over phone

Mostly the answers are towards 4 but i think that the nurse can give paracetamol? Double check
Similar recalls:
https://t.me/silent_2023m/7425
https://t.me/smlemay/17801

Corrected by Eman

esophegeal [rupture‫جاني ت بيكال‬


After force full vomiting sudden chest and abdominal sever pain
‫حرفيا بس هالسطرين‬
Esophageal rupture

540- 62 years old male complaining of painless hematuria upon examintaion there was
a palpable mass in the rectal examination of prostate most appropriate

1- cystoscopy
2- biopsy mass

I had the same Q in my exam the mass was soft with no hard nodules , so it is mostly BPH and does
not require biopsy.
Painless hematuria in elderly is bladder cancer until proven otherwise

Corrected by Eman

541- Women has no background on htn was diagnosed with it whats the best you to
educate her about the risks and complications

1- health promotion
2- health model
‫ وال سكندري مو من ذلين‬primary‫ما انذكر الخيارين الثان ين بس ما كانو‬
Better recall:
https://t.me/silent_2023m/7424

Corrected by Eman

—————

Pictures :

Ecg hyperk , svt ,inf mi ,

X ray transient tachpnec , tof

Endometriosis

Malaria

Sarcoidosis with anemic labs asking for dx :


Low mcv loe hgb

High ferritin

Child with pica low socioeconomic class whats the highest diagnosis value : Cbcs

with peripheral view

Lead levels

—————

542- dite in HTN patients:

A-low salt
B-low fat
C-low protein
Low fat reduces BP by 11mmhg while salt by 5-6 mmhg

https://t.me/silent_2023m/7426
https://t.me/smlemay/17676
Corrected by Eman

543- Jump on 2 feet build up 6 cubes sentence with 2 words phrases

1y
2y
3y
4y

https://t.me/smlepaediatrics/213
https://t.me/smlemay/11012
https://t.me/ASMA_smle/1242
Corrected by Eman

544- Child came at clinic carrry a ball and release or kick it to the doctor.
Also when the doctor go away the child draw a stight line. Age:

A-12M
B-15M
C-18M
D-9M

Straight line at 2 years , closest answer is C


Corrected by Eman

545- Glascow and apgar score ?


546- Case of hernia the plug at lateral and inferior to the iliac tubercle what is the
type of hernia ?

A- Femoral hernia
B- Direct inguinal hernia
C- Indirect inguinal hernia

Femoral hernia : inferior and lateral to pubic tubercle


Indirect inguinal hernia : inferior and medial tubercle
Direct inguinal : abone inguinal ligament
Corrected by Eman

—————
Give now

—————

547- Patient with RA. He is having an infection currently. What


to do?

- Stop adaliumumab and give antibiotic


- Stop methotrexate and give antibiotic
- Stop both medications and give antibiotic
- Give antibiotic

Repeated from um alqurra


Corrected by Eman

548- ECG picture of: pericarditis (i think case was


constrictive pericarditis). One of the findings was increased
JVP with inspiration. What can help with you diagnosis?

- ECHO
- Right sided ECG leads

Right sided ECG will not provide extra information


Corrected by Eman

549- Patient with HF, shown to have ejection fraction of


30%. What is important to add?

- Spironolactone
- CCB

CCB is contraindicated in HF
The answer depends on the given medication in Q
Medications of HF are : ACEI > BB > spironolactone > ivabridine with or without thiazides .
Corrected by Eman

550- 30 years old healthy female came for regular health checkup. She was found
to have high blood pressure. What
tool is best to educate here about the risk factors

- health belief model


- health promotion model
- theory of reasoned action
- diffusion of innovation

Corrected by Eman

551- Calculate CHADS2 score

corrected by: Bugnah

552- A question about Hepatitis B Antibody/Antigen


profile. Anti-HBswas negative. Anti HBc positive (i think).
What do you suspect?

- Active infection
- Recent exposure
- Chronic Hep B infection
- Immunize

Bad recall didn't mention HBsAg : but for short, if there is


+ve HBsAg= infection, either acute or chrnic

+ve Anti-HBs= has immunity either by vaccine or previous infection

+ve Anti-HBc= has infection either acute or chronic

corrected by: Bugnah

553- Harsh systolic murmur at base of heart. Increased with inspiration. Dx?

- tricuspid regurg
- pulmonary regurg or stenosis (cant remember
- mitral regurg
- aortic stenosis
corrected by: Bugnah

554- Ischemic stroke patient. What is the target for BP control?

- 140/90
- 150/90
- 160/100
- 240/120
i think the answer was 220/120

corrected by: Bugnah

555- Patient with HF controlled and asymptomatic. He is on ACEI,


furosemide, simvastatin. What to add to his
medications?

- Beta blocker
- Digoxin
- Do not add any medication
corrected by: Bugnah

556- Patient with fever and pulmonary infection admitted for 5


days. He is hypotensive with low CoP, PCWP and high
SVR. What is the type of shock?
hypovolemic shock

corrected by: Bugnah

557- Post-transplant patient with fever. After 5 days, he is still


not responding to ceftazidime. Labs show low WBC. What
medication is best given?

- vancomycin
- cefuroxime
deal with them like Chemotherapy pt “both immune suppressant”
¹st cefepime
²no meropen
³rd pipra/tazo
if not respond think about MRSA and give “vnacomycin”

if mentioned <1month:give anti-fungal


corrected by: Bugnah

558- 35Y female with headache and fever for 2 days. Labs
show: high bilirubin and LDH. Platelets and Hb were low.
Coagulation profile was normal. Picture of peripheral smear
showing schistocytes. What to give

- plasma exchange
- IVIG with steroid

recall:
https://t.me/silent_2023m/2546
Schistocytes + thrombocytopenia ( schistocytes only happen in DIC / TTP /HUS ) DIC will be so sick and bleeding not only
echomiosis and HUS will have renal impairment + bloody diarrhea usually
corrected by: Bugnah

559- Weird question about a patient with synovitis in small joints of hand. Complained
of fatigue and loss of weight. ANA titer
1:320. Dx?

- SLE
- rheumatoid arthritis with vasculitis

corrected by: Bugnah


560- Plumber came with dry cough. Other symptoms suggestive of restrictive
lung disease as well. How to get a definitive
dx?

- HRCT
- DLco

corrected by: Bugnah


561- Female was in a household fire. She is having some dyspnea and lost
consciousness at the time. She is now still dyspneic, conscious but drowsy. What to
do?

- Secure airway
- ABG
- CXR

ABC Steps: Airway, breathing,Circulation> so secure the airway then do Ix


corrected by: Bugnah
562- Delirium in elderly and taking so many medications listed
in the question. What is right?

- medications are a common cause


- start antipsychotic
it depends on scenario and medications listed
corrected by: Bugnah

563- Ederly having hallucinations, insomnia, and waking up


at night. No symptoms of dementia. No tremors or other
neuro findings.

- lewy body disease


- Prion disease

corrected by: Bugnah

564- IUFD in 3rd trimester. On cervical exam: 5cm dilation


and 80% effacement

- expectant management
- augment labor
in IUFD usually takes more time so need augmentation to prevent its complication
corrected by: Bugnah

565- IUFD case. What is given?


(this question is different than
the previous)

- misoprostol pessary
- oxytocin
- laminaria
corrected by: Bugnah

566- Labour case. What can be used for cervical ripening?

- PgE2
- Oxytocin
- misporostol corrected
by: Bugnah

2 questions came.

567- Both elderly and family asks you to not tell them as it will cause them to be
more depressed and not help.

- Choices were regarding tell the patient or not as well as involving ethics commitee

568- 47 year old female hypertensive and BMI 35. What is most important screening
test to do?

- FBG or
-Pap smear corrected
by: Bugnah

569- Child having sibling with chicken pox. He is asymptomatic for 2 days. What to
do now?

- observe
- immunization
- VZIG
immunization unless she is pregnant or immunosupressant give them IVIG
corrected by: Bugnah

570- Adult admitted with pneumonia and examination shows vesicals on trunk
and extremities. What is most appropriate
- acyclovir
- 3 choices of antibiotics
corrected by: Bugnah

571- TB meningitis Rx?

- quadruple + dexa
- quadruple regimen

corrected by: Bugnah

Another opinion:-
As per UTD and Amboss ttt of TB meningitis should involve the anti-TB regimen with
adjunctive glucocorticoids (dexamethasone or prednisolone)
572- Case of hepatitis, patient complaining of pruritis and RUQ pain. Liver
biopsy showed plasma cells.
What to give?

- prednisone
- ursodeoxycholic acid

another recalls:
https://t.me/silent_2023m/5098
https://t.me/FM98SMLE/705
corrected by: Bugnah

573- Patient known case of Crohn’s disease. Had surgery years ago. CT
showed jejunonileal fistula with fluid collection.

- surgical drainage
- open drainage
- percutaneous drainage
- Fistulectomy with drainage

corrected by: Bugnah

574- Child having copious watery diarrhea. How to reach diagnosis?

- stool antigen
- stool culture
- serum viral culture

most common watery diarrhea due to Rotavirus

575- When does rooting reflex disappear?

- 6 months
- 2 months
- 4 months

corrected by: Bugnah

576- Mother brings her healthy preschool age child (written without actual age),
which is most imp vaccine?

- MCV4
- OPV
- HiB

Preschool vaccines : OPV, DTap, Varicella, MMR


577- Milestone: 6-10 words walking alone. Age?

- 19 months
- 17 months
- 12 months
- 24 months

baby nelson ¶
if there is 15months i will go with it
corrected by: Bugnah

578- RUQ pain and fever. Patient toxic according to given signs.
Liver CT shows right lobe collection with thick wall, no septate and no peripheral
enhancement. What is to be
done?

- percutaneous drainage
- metro
- surgical
- ceftriaxone
Dx is pyogenic abscess
corrected by: Bugnah

579- Penetrating injury to chest with distended JVP, decreased


breathing sounds in right, normal heart sound and sq neck
emphysema?

- tension pneumothorax
- massive hemothorax
corrected by: Bugnah

580- Child with SMA, and physian suggested DNR. What


is right?

- discuss DNR option with parents


- Obtained by 3 trustworthy doctors

first discuss with his parents

similar recall:
https://t.me/silent_2023m/7455
corrected by: Bugnah

581- newborn with microcephaly, high LFTs and jaundice what to do?

- congenital infection screen


- chromosomal abnormality

Similar recent recall(from march) :-

Newborn with microcephaly and distended abdomen exam showing hepatosplenomegaly, I think
mother was diabetic. Labs showing pancytopenia and elevated LFTs (AST/ALT). Next ?

A. Chromosomal analysis.
B. Congenital infection screening.
C. US abdomen.
D. CT abdomen.

Suhaib
Microcephaly, anemia, thrombocytopenia, hepatoslpenomegaly, jaundice -> most likely goes with
congenital rubella syndrome , so the correct answer would be congenial infection screening
(TORCH)
582- Child came with cyanotic spells. CXR shows small heart and decreased pvm.
What to do?

- fluid bolus

Suhaib
PVM : pulmonary vascular marking
Congenital cyanotic heart diseases are :-
The one that presents with decreased PVM and normal size heart is tetralogy of fallot

UTD : The classic chest radiograph of a patient with TOF demonstrates a "boot-shaped" heart with an
upturned apex and a concave main pulmonary artery segment. The heart size is often normal, and
pulmonary flow will appear normal or decreased.

Amboss
So, if question asks about next and among answers :O2, squatting, Morphine they would be correct.
If not in choices choose IV fluid Definitive
management : surgical repair

583- 34YO female came with irregular breast lump and skin tethering. No lymph
nodes. Her mother had breast cancer.
What is your next step?

- bilateral mammography
- FNAC
- biopsy

Suhaib
Next : mammography or US preferred at this age (30-39) to determine if biopsy needed by using
BIRAD system
Type of biopsy in breast masses is core needle biopsy (not FNA)

UTD :Even if the mass is clinically suspicious for cancer, it is still preferable to image before biopsy.
Further evaluation after the diagnostic mammogram depends on the lesion's BI-RADS category

UTD : 30 to 39 years of age — Either breast US or mammography can be used as the initial imaging
modality for evaluating a patient 30 to 39 years of age, although likely both US and mammography
will be performed as the two studies are complementary.
Because the sensitivity of US is higher than mammography in this age group (96 versus 61 percent)
it is reasonable to start with US as the initial modality, but with a low threshold for using
mammography if clinical suspicion is high.

So, if among answers US it would be better answer

584- Breast lumpectomy to be done in critical patient with COPD


and low pulmonary reserve. Type of anesthsia?

- pectoral block
- GA
- spinal

Suhaib
Need double check

UTD : CHOICE OF ANESTHETIC TECHNIQUE(COPD pt)


The choice of anesthetic technique (monitored anesthesia care [MAC], neuraxial anesthesia,
peripheral nerve blocks, or general anesthesia) should be guided primarily by the requirements of the
procedure, and by surgeon and patient preferences. Notably, bronchospasm may develop at any time
in the perioperative period during any surgical procedure, regardless of which anesthetic technique is
selected.

For most patients with COPD, we suggest MAC, neuraxial anesthesia, or another regional anesthetic
technique (eg, peripheral nerve block) when appropriate for the planned procedure, to avoid airway
stimulation due to laryngoscopy and endotracheal intubation or insertion of other airway devices.

● However, general anesthesia is typically preferred for patients with more severe COPD (eg,
baseline dyspnea with minimal exertion, inability to lie supine, persistent coughing) and for
procedures of prolonged duration. Also, general anesthesia is necessary for performance of
many surgical procedures (eg, abdominal laparoscopic procedures, thoracic procedures, or
surgical incisions involving the head, neck, or multiple extremities), and for those who do not
consent to a regional anesthetic technique.

-Here we have a thoracic surgery which is an indication for GA in COPD pt


-If by “low pulmonary reserve” in question he means severe COPD symptoms , it would be another
indication for GA

UTD : Oncoplastic surgery requires extensive mobilization and advancement of breast tissue, thus
requiring general anesthesia.

585- Open inguinal hernia repair in COPD patient?

- spinal
- GA
- inguinal block

Suhaib
UTD : CHOICE OF ANESTHETIC TECHNIQUE(COPD pt)
The choice of anesthetic technique (monitored anesthesia care [MAC], neuraxial anesthesia,
peripheral nerve blocks, or general anesthesia) should be guided primarily by the requirements of the
procedure, and by surgeon and patient preferences. Notably,
bronchospasm may develop at any time in the perioperative period during any surgical
procedure, regardless of which anesthetic technique is selected.

For most patients with COPD, we suggest MAC, neuraxial anesthesia, or another regional anesthetic
technique (eg, peripheral nerve block) when appropriate for the planned procedure, to avoid airway
stimulation due to laryngoscopy and endotracheal intubation or insertion of other airway devices.

However, general anesthesia is typically preferred for patients with more severe COPD (eg, baseline
dyspnea with minimal exertion, inability to lie supine, persistent coughing) and for procedures of
prolonged duration. Also, general anesthesia is necessary for performance of many surgical
procedures (eg, abdominal laparoscopic procedures, thoracic procedures, or surgical incisions
involving the head, neck, or multiple extremities), and for those who do not consent to a regional
anesthetic technique.

Me : Here the patient is undergoing Open hernia repair, in which local anesthesia is preferred (whether
patient has COPD or not) but COPD can further support the choice of local anesthesia .

UTD : Choice of anesthesia (in open hernia repair)

Anesthesia for open repair — For patients undergoing open groin hernia repair, we suggest local
anesthesia, rather than neuroaxial or general anesthesia. Local anesthesia is preferred especially in
patients with comorbidities (eg, advanced liver disease). The use of local anesthesia reduces
complications and cost in both older and younger patients .

Local anesthesia can be administered as a nerve block of the ilioinguinal and iliohypogastric nerves or
as direct infiltration into the incision site(s). Nerve block may be more difficult to administer but
causes less soft tissue edema than direct infiltration. Some surgeons use a combination of both nerve
blocks and local infiltration. Local anesthesia for open groin hernia repair is typically given in the
context of "monitored anesthesia care," which also provides intravenous sedatives for patient
relaxation and additional intravenous analgesics.

586- Pregnant with nausea and vomiting. Looks pale. Low


Hb and High MCV, cause?

- B12 deficiency
- folic acid deficiency
- IDA
- anemia of pregnancy

Suhaib
UTD : Physiologic anemia of pregnancy and iron deficiency are the two most common causes of
anemia during pregnancy

Physiologic anemia : normocytic / IDA : microcytic. But here we have macrocytic anemia .
UTD :Folate deficiency is the most common cause of megaloblastic anemia during pregnancy, often
associated with diets low in animal proteins, fresh leafy vegetables, and legumes

Between B12 and Folate : Folate deficiency is much more common than B12 deficiency B12

can be suspected if mother history of : Crhon’s disease, gastrectomy.


UTD : Vitamin B12 deficiency is a cause of macrocytic anemia in pregnancy in some
individuals, particularly those who have had partial or total gastrectomies or those with Crohn disease.

587- Antral gastric cancer. What is the best tool to stage “T” part?

- endoscopic US
- Abdominal US
- PET
- barium

Suhaib
UTD : EUS is better than CT at assessing tumor depth (T stage) and perhaps lymph node
involvement (N stage), particularly if fine-needle aspiration (FNA) is also performed.

588- epileptic with active disease, came for immunization

- defer DTP
- defer OPV
- give vaccines
- dont give any live vaccine

Suhaib
If epilepsy disease controlled -> no need to delay DTP vaccination

589- Abnormal uterine bleeding rx?

- tranexamic acid
- Oxytocin
- misoprostol

Suhaib
Here AUB(abnormal uterine bleeding) which has different management Than PPH

AUB could be acute or chronic


Acute AUB Mx (dr. wafa):-
Chronic AUB Mx :-

590- ESRD with low calcium? What is the treatment?

- calcitriol
- sevelamer
- cinacalcet
- 25 hydroxycalciferol

Suhaib
Similar recall ;-
Patient k/c of ESRD with high PTH and low calcium, what to give?
A. Calcitriol
B. Vit D 3
C. Vit D 4
D. Calcitonin

Another recall from Dr Alharbi channel:-


31-A patient with renal failure presents with low calcium, high alkaline phosphatase, and high PTH.
What is the best form of vitamin D to give this patient?
A- Calcitriol
B- Vitamin D2
Dr Alharbi : Answer: A , if you want to treat high PTH , Treatment options — Treatment options for
increased PTH include calcimimetics, calcitriol, or synthetic vitamin D analogs [5]. A combination of
calcimimetics with calcitriol or synthetic vitamin D analogs may also be used.

UTD : Circulating calcitriol (1,25-dihydroxyvitamin D), the most active metabolite of vitamin D, is
principally synthesized in the kidney. Circulating calcitriol levels begin to fall when the eGFR
is <40 mL/min/1.73 m2 and are typically markedly reduced in patients with ESKD. In addition to the
loss of functioning kidney mass, calcitriol production is also reduced by phosphate retention.

591- runny nose, rash and conjunctivitis, what is the cause?

- rubella
- coxasackie
- vaccinia

Suhaib
Since the presence of conjunctivitis Measles would be a better answer (coryza, conjunctivitis, cough),
but in some recalls no measles or rubeola (another name for measles) in answers, in this case rubella
would be the closest answer .

Similar recalls from Dr Mahdi pedia channel


47) Child complaing from coryza and conjunctivitis , macuopaplar rash started in face then the body ,
what is the cause?
- Gonccocal bactermia
- Rubella
- Toxic shouck sydrome
- Rotair syndrome
"No measals in answer 100% sure.. and it is rubella not rubeola"

Child had URTI 2 days ago, presented with erythematous rash that covers the jaw and extends to the
shoulder and back , without desquamation and occipital posterior cervical post auricular
lymphadenopathy (written in case like this)
- Rubella
- Adenovirus
- Measels
- Erythema infectiosum

592- 2 cases of tonsillitis


1. asking for treatment

Suhaib
Centor score is used to determine the next step in management of patients with Tonsillitis :-

Q from Dr Mahdi pedia file :-


593- 2. asking for causative organism: EBV
594- Mother developed infection and IgG were formed. How will it affect the baby?

- passive natural immunity


- active natural immunity
- passive artifical immunity
- active artifical immunity

Suhaib
595- Stage of APAP toxicity?

UTD ; The clinical course of poisoning is often divided in four sequential stages.

Stage I (0.5 to 24 hours) — In the first 24 hours after overdose, patients often manifest nausea,
vomiting, diaphoresis, pallor, lethargy, and malaise. Some patients remain asymptomatic.
Laboratory studies are typically normal. After massive acetaminophen overdose, central nervous
system depression and elevated anion gap metabolic acidosis may be seen, albeit rarely

Stage II (24 to 72 hours) — From 24 to 72 hours after ingestion, the laboratory evidence of
hepatotoxicity, and occasionally nephrotoxicity, becomes evident.
Initially, stage I symptoms resolve, and patients appear to improve clinically while worsening
subclinical elevations of hepatic aminotransferases (aspartate aminotransferase [AST], alanine
aminotransferase [ALT]) develop.

As stage II progresses, patients develop right upper quadrant pain with liver enlargement and
tenderness. Elevations of PT and total bilirubin, oliguria, and renal function abnormalities may become
evident .

Stage III (72 to 96 hours) — Liver function abnormalities peak from 72 to 96 hours after ingestion.
The systemic symptoms of stage I reappear in conjunction with jaundice, confusion (hepatic
encephalopathy), a marked elevation in hepatic enzymes, hyperammonemia, and a bleeding diathesis.
Signs of severe hepatotoxicity include plasma ALT and AST levels that often exceed 10,000 IU/L.

Stage IV (four days to two weeks) — Patients who survive stage III enter a recovery phase that
usually begins by day four and is complete by seven days after overdose.
596- Periorbital edema child with dark urine and proteinuria, which will support
the diagnosis?

- Impetigo before 1 month


- Urine culture
- High blood pressure

Case of Nephritic syndrome, main features :-


- Hematuria
- Hypertension
- RBC cast in urine
- Proteinuria (less than in nephrotic <3.5g/24hr)
- Edema (mild to mod)

597- Thoracentesis level?

- 4th rib (ICS not rib)


- 5th rib
- 6th
- 7th
- 8th
- 9th

UTD : INSERTION SITE


Generally, thoracostomy tubes and catheters are placed at the fourth or fifth intercostal space in the
anterior axillary or midaxillary line. In emergency situations, the landmarks are the nipple line in
males and the inframammary crease in females

Positioning
The patient is supine with the head of the bed between 30 and 60 degrees. The
ipsilateral arm is abducted and secured above the patient's head.

The safe triangle margins :-


598- Tubal ligation consent?

- Wife and inform husband

Similar recalls :-
2_ Case of woman of tubal ligation doctor discussion from where taken consent? A_
Mother consent and inform husband
B-From both consent
C-. From mother only
D.from husband

Case of woman the doctor advise her to do tubal ligation doctor discussion from where taken consent?
A. Her consent and asked to inform husband
B. From both consent is required
D. from husband
‫الخيارات جاتني زي كذا اب لضبط‬

599- Case of child with fever and developed a seizure. What to do?

- Control of fever
- Diazepam
- Head CT
- EEG

600- Brucellosis case in a man without neuro symptoms. Best management?


- Doxy + Streptomycin
- Doxy + Rifampicin

Suhaib
Amboss
Antibiotic therapy (for brucellosis)
First-line therapy: doxycycline PLUS rifampin Second-
line therapy: doxycycline PLUS streptomycin

—————
—————

601- Benign phyllodes tumor of the breast management?

A. Wide local excision

Alaa: A

similar recall

Female with benign phyllodes tumor asking about management?


A. Wide local excision
B. Wide local excision with radio
C. Mastectomy
D. Reassurance
Answer is: Depends on the size.

Patient with painful breast mass since 6months. It get increased.it is in the outer upper surface (around 9
clock) Histopathology confirmed a benign phyllode tumor 1 cm What is the best next step?

A. Simple mastectomy

B. Wide local excision with radio

Benign phyllode tumor of 3 cm, what is the treatment?

A. Wide local excision

B. Simple mastectomy

A lady complains of breast mass, biopsy done and show: phyllode tumor 5*5 What is the most
appropriate management?

A. Radiotherapy
B. Chemotherapy
C. Wide local excision

D. Simple Mastectomy

Case showed cystophylloides breast what’s the treatment?

A. Wide local excision


B. Mastectomy

C. Follow up after 6 months

Female presents with progressively enlarged breast mass. imaging showed: cystosarcoma phyllodes
What is the most appropriate management?

A - Radiotherapy
B - Neoadjuvant chemotherapy C -

Modified radical mastectomy

D - Simple mastectomy

Explanation:

Benign > WLE if small mastectomy if large

schwartz
Small phyllodes tumors are excised with a margin of normal- appearing breast tissue. When
the diagnosis of a phyllodes tumor with suspicious malignant elements is made, reexcision of the
biopsy specimen site to ensure complete excision of the tumor with a 1-cm margin of normal-appearing
breast tissue is indicated. Large phyllodes tumors may require mastectomy. Axillary dissection is not
recommended because axillary lymph node metastases rarely occur.

602- Vaginal discharge with spores under microscope ?

A. Candida

Alaa: A

full recall:

A women presents with vaginal discharge. Microscopic reveals spores. What is the most likely
diagnosis?
A. Bacterial vaginosis
B. Trichomoniasis

C. Candidiasis
D. Bacterial infection

Alaa: C from Dr: wafaa’s note.

603- Old woman with mild vaginal bleeding , site ?

A. uterus

Alaa: A

full recall

60y old lady present with lower genital bleeding, she described it as Scanty and barely stain the pad ,
what is the source of bleeding?
A. Fallopian tube
B. Ovary
C. Uterus
D. Genital tract

Alaa: C from Dr: wafaa’s note.

Note to Remember

Bleeding in elderly is endometrial cancer until proven otherwise.

604- A case about Pseudocyst (diagnosis not given), management ?


A. Endoscopic US guided drainage

Alaa: A

similar recall

-Patient post pancritis with abdominal pain found to panarctic pseudocyst with NO necrosis

A. Internal drainage
B. External dringe

Pancreatitis 5 weeks ago. Now she has epigastric tenderness and cannot tolerate food with vomiting
each time. By ultrasound you found large about 12X10 mass With thick wall and fluid inside. Labs:
346 amylase, Wbc 15k.

What is the diagnosis? pseuocyst tx.

endoscopic drainage

Case of pancreatic pseudocyst under observation for wks and there are increase in size and no fever
What’s the management?
A. Laparoscopic drainage
B. percutaneous drainage

None of them. it must be EUS drainage. but if not exist among the answers then go with B.

605- A case about bilateral inguinal hernia management ?

A. laparotomy with mesh


Alaa: A

similar recall

Pt with bilateral inguinal hernia how to manage in other recall A soldier needs to undergo
zxbilateral hernia repair. So what’s your management

Answer is: Lap with mesh


herniotomy in peds and lap. hernioplasty in adults

Direct or indirect, inguinal hernia for elective repair in 60 ys old male:

A. Open with mesh


B. Lab. Repair
C. Direct repair
Answer is: A, because there is no lap but it should be with MESH.

606- Child says Hi and feed the doll with milk ?


A. 24 month

Alaa: A

Full recall

Pediatric patient came with her mom to the clinic. She said “hi” to the doctor. Asked her mom for
milk “me”, and was feeding her doll, then she pointed to her mouth and said “mouth”. She is
mimicking her mother, how old is she:

A. 12 m
B. 15m
C. 18m
D. 24 m

607- Female wants ocp prescription but refusing doctor's pelvic examination first ?
A. refuse to prescribe

Alaa: Not sure


608- You are an Intern assisting surgeon in operation room, The surgeon made
a mistake and told him to prepare the Lt arm and the intern knows from previous
examination it's the Rt arm

A. Discuss your concern with the surgeon


B. leave the operation room to check the papers
C. Ask the nurse to check the papers
D. Call your resident to ask him what to do

Alaa: A

similar recall

Intern hear the left kidney is diseased and the surgeons say he will operate on the right in the OR what
should the intern do?

A. Inform the surgeon

609- TTN Case and another x-ray picture ... Both wants the diagnosis.

Alaa: similar recall

Baby born 27 weeks after 30 mins started having tachypnea and grunting most common cause?
xray chest preterm with signs of respiratory distress

A. RDS
B. TTN
C. Meconium aspiration

Answer is A in preterm, B in term (with CS delivery)

Newborn after CS have respiratory symptoms , CXR shows fluid in the


horizontal fissure , what’s the dx

A. TTN
B. acute distress syndrome
C. meconium aspiration

chest xray of newborn labs all normal vital signs low oxygen saturation no fever using

esocssory muscles whats the diagnosis? TTN

Baby CS respiratory distress grunting? TTN


610- A case about Bronchiolitis most appropriate management (diagnosis not given) ?

A. Supportive and hydration

Alaa: A

similar recall

18 months with a picture of bronchiolitis, developed several episodes of apnea. What’s the
appropriate management?

A. Supportive
B. Ventilatory support
C. IV methylprednisolone

Case of bronchiolitis (severe) with chest recessions what is the management?

A. Admit for fluids hydration and oxygen

(Infants and children with severe bronchiolitis require assessment in the emergency department and
usually require supportive care in the inpatient setting. Supportive care and anticipatory guidance are
the mainstays of management of severe bronchiolitis. ) → UTD

from Dr.Safdar

● First steps in patients with respiratory distress


● ABCDE
● So oxygen and IV hydration are the 1st line management
● Patients with bronchiolitis usually need supportive management as it is viral disease

611- Baby set in tripod position and can switch between prone and supine
positions and vise versa but can't hold cereals between two fingers

A. 6 months
B. 4
C. 9
D. 12

Alaa: A

from Dr.Safdar’s notes


● Sitting tripod : unsupported at 6 monthds
● Role from prone to supine 4 montns
● Reach object 4 months

can't hold cereals between two fingers → pincer grip at 9 months

612- 14 years old girl with flank pain diagnosed as Rt ureteric kidney stone 4mm
,management?

A. conservative and follow up

Alaa: A

similar recall

Patient had 4 mm ureteric colic what is the appropriate management?

A. external shock wave lithotripsy


B. conservative treatment (Reassure will pass)
C. ureterscopy
D. Surgery

613- A doctor prescribed angiotensin receptor blocker to a patient, which drug?


A. Irbesartan

Alaa: A

similar recall

Which one of these is Angiotensin II receptor blockers ?

A-Lisinopril
B-Valsartan
C-Amlodipine

614- A chest pain case Troponine high & ECG inverted T wave in ii,iii,aVF (no ecg
picture) ?

A. stable angina
B. unstable angina
C. STEMI
D. NSTEMI

Alaa: D
similar recall

60 yo F Known DM and HTN Came with chest pain for 3 hours Other info I forgot Vital signs “I don’t
remember we’re significant numbers”
Investigation:
Ecg t wave inversion in lead 5,6
trop 10
What’s the most likely diagnosis?

A/ stable angina B/
unstable angina C/
MI

Answer → this NSTEMI

76 years old male, present with retrosternal chest pain with exertion relieved by rest, on
examination S4 was heard. ECG showed T wave inversion in lead V2-VS What is the
diagnosis?
Cardiac enzymes were done already
Troponin level was 10 (normal 0.05)

A. Stable angina
B. unstable angina
C. NSTEMI
D. STEMI

Note :
Patient presents with chest pain and ST depressions on ECG with either: (-)

troponins: Angina (unstable or stable) (+) troponins: NSTEMI

615- A trauma case with increased JVP which part of the heart affected ?
A. Pericardium

Alaa: A

Full recall

45 years male came to hospital complaining of sob fever fatigue for few days, on examination revealed
high jvp, pulsus paradoxus, distended heart sound auscultation revealed clear lung, Lab Bp 90/55, Hr
120, Rr20, Temp 36.6. Ecg show: Low voltage QRS. Xray :Increase cardiac silhouette. What is the
cardiac area is involve?

A. Valves
B. Pericardium
C. Myocardium
D. Coronary artery

Note:
Pericardial diseases differentials:
-Pericarditis
-Pericardial effusion & Tamponade (present with low voltage and electrical alternus)
-Constrictive pericarditis

616- A case about systolic murmur heard over the lt sternum, cause ?
A. Vsd
B. ASD
no pulmonary stenosis

Alaa: A

similar recall

5 yr O/E pansystolic murmur at left lower sternal border

A) Vsd
B) Asd
C) Pulmonary stenosis

D) PDA

7 months old presented with history of interrupted foods associated with difficulty in breathing and
sweating for the last 4 months. Physical examination revealed normal peripheral pulses, hyperactive
precordium, normal S1, loud S2 and Pansystolic murmur grade 3/6 with maximum intensity at the 3rd
left intercostal space parasternally. The MOST likely diagnosis is:

A. Small PDA (Patent Ductus Arteriosus).


B. Large ASD (Atrial Septal Defect).
C. Aortic regurgitation
D. Mitral regurgitation.
E. Large VSD (Ventricular Septal Defect).

According to Dr. Safdar Differentiate

between the murmurs !

● VSD: PanSYSTOLIC MURMUR AT LOWER LEFT STERNAL BODER


● ASD: WIDE FIXED SPLITTING SECOND HEART SOUND PDA: CONTINOUS
MACHINARY MURMMUR
● MITRAL STENOSIS: MID DIASTLOIC MURMUR AT APEXA, raidiating to axilla
● MITRAL Regurgitation: PANSYSTLOIC MURMURA AT MITRAL AORTIC
● STENOSIS: SYSTOLIC MURMUR AT AORTIC AREA RADIATNG TO neck
COARCTATION of aorta: SYSTOLIC MURMUR AT left 2nd intercostal space, radiating to
back
● Small VSD: no symptoms , harsh murmur
● Large VSD: soft pansystolic murmur , mid diastolic murmur over mitral area and
systolic murmur over aortic area

617- Female 14 years with her mother complaining of delayed puperty and shorter,
with webbed neck and decreased concentration in school?

A. Turner

Alaa: A

similar recall

Female 16 or 17 pt come with her mother . (featre of Turner syndrom w/o name Dx)

A. Constitutional short stature


B. endocrine short stature C.
syndromic short stature

618- A case of short female with wildly spaced nipples Type of chromosomes?

A. 45X

Alaa: A

Full recall

Patient short stature , absent period wide spacing nipples karyotyping

A- 45x

B- 46xy

C- 45xx

D- 46y

619- A case about multiple air fluid level in small intestine, next step imaging ?
A. US
(I think there was no CT in the choices)

Alaa:
similar recall

Young adult with s&s of SBO (small bowel obstruction), what’s the next initial step? In other recall
sign of obstruction erect x ray show multiple air fluid level ? initial step

A. CT abdomen

B. Examine the groin area

25 years old man Hx of laproscopic appendectomy 10 years ago, present to ER with sudden
abdominal pain and vomiting , mild generalized tender, vitally stable, abdomen x ray shows multiple
air fluid level and dilated loop in certian point, what is next step?

A- Diagnostic laparoscopy
B- Laparotomy
C- CT abdomen
D- Barium swallow

620- A case about old woman complaining of heaviness sensation in lower abdomen &
difficulty in defecation , she has to push through her vagina by her fingers to complete
the defecation
(I guess posterior vaginal prolapse)
Wants the type of surgery?
(Can't remember the choices except posterior or anterior something)

Alaa:

similar recall

49- Old lady medically free with difficulty defecation and constipation, during defecation she
introduce her finger in the vagina to intiate movement, management?

A. Posterior colporrhaphy
B. Anterior colporrhaphy
C. Enterocele resection
D. Hysterectomy

621- Case of leg tense edema after surgery with absent pulses ...
CT angio

(no duplex US in the choices)


Alaa: Not sure

622- Esophageal cancer common risk factor ?

A. Smoking
B. barrett

Alaa:

similar recall

35 YO Pt with hx of GERD. came discussing or concered about his family Hx of


esophageal and gastric carcinoma.
What's is the main risk factor for esophageal carcinoma?

A. Barrettes esophagus.
B. Smoking
C. idiopathic
D. can’t remember but excluded

Note :
- Oesophageal cancer,

smoking; SSC (most common risk factor ); up 2/3, Barrette;

adenocarcinoma (strongest risk factor ); Low 1/3

623- A case about end stage cancer and the family wanting to increase the pain killers
and you explain that it could affect him ?

A. Double effect

Alaa: A

similar recall

Patient old with end stages cancer disease complaining of severe pain and his children ask you to give
him stronger pain killers you respect theirs wishes but you told them that the pain killers may fasten
the death of their father question in this case what the doctor do?
A- principles of double effects

An old patient with end stage cancer had severe pain. The family asked the physician to give stronger
analgesics but the doctor explained that although it would alleviate his pain, it may also shorten his
life. This scenario represents which principle?

A. Principle of double effect


B. Principle of totality
C. Principle of informed choice
D. Principle of subsidiary

624- Liver hemangioma…


A. avoid extensive sports

Alaa: A

Full recall

liver hemangioma (dx was written, young pt overweight/obese and smoker) conservative
management was decided.
Advice for the patient:

A. Weight reduction
B. avoid contact sports
C. lower carb consumption
D. stop smoking

Explanation : contact sports might lead to trauma which will cause rupture to hemangioma Other

recall

Smoker and obese female patient on combined OCP, at imagining there is 4x4 cm hepatic
hemangioma. What is the most important thing to advise the pt.?

A. Decrease high carbohydrate and fatty meals

B. Stop smoking

C. Eat diet rich in fiber

D. Stop OCP

625- A long case about non vaccinated child with fever and erythematous pharyngitis
which easly scratched with blood spotting upon taking a swab ?
A. Diphtheria

Alaa: A

similar recall

Pediatric patient with neck swelling, inspiratory stridor, difficulty breathing, I think also with
enlarged tonsils ), didn’t receive any vaccine, what is the dx:
A. Diphtheria
B. Pharyngitis

According to Dr.Safdar

● Diphtheria can cause fatal airway obstruction


● Gray white membrane on pharynx
● Occur in non immunized children
● Complications
● Neuropathy
● Cardiomyopathy
● Treatment
● Anti toxin Macrolide

626- A case about female 22y with pallor and anemia, CBC & Iron profile explaining iron
deficiency anemia with Hb around 8.5, previously diagnosed and not responding to
tablets , what's next ? ...

A. IV iron and follow up


B. increase dose
C. blood transfusion

Alaa: A

627- A case about male in his 30s with RTA and chest trauma with massive chest
echemosis which is increasing, fully conscious but with increasing respiratory
distress and O2 sat 82% what's appropriate management ...

A. Urgent thoracotomy
B. IV strong analgesic
C. Nebulization
D. Mechanical Ventilation

‫مالحظة يتم نسخ السؤال ولصقه هنا بالملف بنفس الصيغه وبنفس الخطا مثل ما مت نقله لنا بعدها حنا نصحح والمصحح يضع اسمه تحت السؤال‬
Naif: ABCD ; ATLS protocol
Answer is secure Airway ( Mechanical Ventilation ).
Dr. Abdullrazaq: ‫ ذا‬ATLS protocol ‫ دايم‬ABCD Alaa:

Full recall

21 years old is involved in a head-on collision as the driver of a motor vehicle. He is


noted to be severely tachypneic and hypotensive. His trachea is deviated to the left, with

palpable subcutaneous emphysema

and poor air entry in the right hemithorax. The most appropriate first treatment procedure

should be:

A. Arterial puncture to measure blood gasses.


B. Stat chest x-ray.
C. Intubation and ventilation.
D. Needle thoracocentesis or tube thoracotomy prior to any investigations.
E. Immediate tracheostomy.

similar recall

Trauma patient came complaining of chest pain. He had palpable subcutaneous emphysema and on
auscultation reduced breath sounds in the right side. What is the next management?

A. needle decompression
B. intubation

628- A case of child with scrotal pain and one testicle slightly elevated?

A. Testicular tortion

Alaa: A

629- Another case of child with sever testicular pain and scrotal tenderness ,redness
& swelling , what's next?
A. Laparotomy

Alaa: A

similar recall

-pediatric patient with sudden scrotal pain, on examination high riding testis and tender
erythema, what’s the diagnosis and management?
A. Testicular torsion, surgical exploration
630- Two typical cases about Duodenal perforation. Both need diagnosis. One with seat
belt sign & another with emphysema in the abdomen, chest & shoulder i think after
ERCP

Alaa:

similar recalls

A female get high energy accident (high velocity), with seat belt sign, on X ray have chance
fracture. What will associated with this fracture?

A. Duodenal perforation
B. Gastric perforation
C. Jejunum perforation
D. Vena cava perforation

Answer is A → Seat belt sign + Chance fracture

DM and HTN known case patient, presents with severe epigastric pain radiating to back for 6

hours, CXR shows left air under diaphragm, the most likely diagnosis ?

A. Perforated peptic ulcer


B. Acute pancreatitis

Note:

Gastric/duodenal perforation Second most common complication of PUD located anterior more
commonly than posterior, Duodenal ulcers of the anterior wall are more likely to perforate into the
abdominal cavity, causing pneumoperitoneum (free air below the diaphragm) and irritation of phrenic
nerve (e.g., shoulder pain).

631- A case about results of the appendix came back with cancer at the base
of appendix 10% I think ? (bad recall, sorry)

A. Rt hemicolectomy

Alaa: A

similar recalls

Case of appendicular carcinoid tumor 0.5 cm . What your Rx

A. Colectomy
B. CT
C. No further investigation

Answer is: Appendicectomy

Patient underwent appendectomy. You found carcinoid tumor at the tip of appendix after
surgery. What will you do next?

A. CT chest, abdomen and pelvis

Answer is: More than 2 cm do CT then right hemicolectomy

After appendectomy dr found a carcinoid mass on the tale of appendix Next?

A. CT scan abdomen and chest

Answer is : A for staging

old pt after appendectomy, found to have 0.5mm carcinoid tumor in the appendix.. what is your action ?

1. Rt. Colectomy
2. No further ttt/ follow up

Note:

● appendiceal tumor is sometimes discovered incidentally. The most common appendiceal


tumor is mucinous adenocarcinoma (it was previously thought to be carcinoid) with the
majority of these tumors located at the tip of the appendix.
● If the tumor is ≤1 cm, an appendectomy is considered appropriate definitive
management.

For tumors larger than 1 cm and located at the base of the appendix or tumors ≥2 cm and located
at the tip of the appendix, a right hemicolectomy is indicated.

632- A case of a child with diarrhea and vomiting then developed abdominal
distension and red current
jelly stool , most common site ..
A. iliocolin
Salam: A
Ileocecal invagination (most common; accounts for 85–90% of cases)(amboss)
similar recall

● A child came with vomiting and currant jelly stool , abdominal distension
● Next step
● A-Pain killer
● B-IV fluid
● C- Barium enema

633- A cast about male with reducible inguinal hernia waiting for elective surgery .
then developed pain, nausea, vomiting, bulging with redness hotness tenderness ...

A. Strangulated hernia
B. incarcerated hernia
Salam: A
explanation: If the contents of the hernia are not able to be reduced, the hernia is
considered incarcerated. A strangulated hernia occurs when the hernia contents are
ischemic due to a compromised blood supply (NCBI)

634- A case about child I guess 2 years old with cough not improving with
nebulization , upon auscultation
- localised wheezing in lt side lower part (Foreign body), what's next ...

A. bronchoscopy
B. Other choices (tracheoscopy)
C. (CT chest)

Salam: A

635- Typical case of Epiglottitis (diagnosis)


Salam: similar recall
a child came in with fever and cough then improved, after one week, he came back with high
grade fever, drooling of saliva, most likely diagnosis?

A. diptheria
B. epiglottitis
C. croup
D. bacterial tracheitis

explanation: this recall was answered as D but according to NCBI: Children with bacterial
tracheitis also generally appear more toxic and have higher body temperatures (5,8,9,17,18).
Unlike the classic description of children with epiglottitis, those with bacterial tracheitis are
usually able to swallow their oral secretions and therefore do not present with drooling.
Bacterial tracheitis preceded by viral URTI
Sign and symptoms of epiglottis:

636- A case of Croup cough and stridor asking what's helpful in diagnosis

A. clinically (There was no Lateral X-ray in the choices)

Salam: according to Dr.safdar,

● Croup usually diagnosed clinically


● However lateral next X rar can be done if croup did not improve to management or
you suspect other causes
● If lateral neck ray did not show any findings then do Ct neck

637. . Baby just delivered O+ve and mother O-ve?

A. Give anti-D to (mother


B. Give anti-D to baby
C. Give anti-D to mother and baby
D. Not give anti-D

Salam: if mother is first pregnancy (unsensitized), then give anti-D to mother only within 72
hrs of delivery. if mother already has antibodies (sensitized) no role of anti-D(amboss)
637- A case of newborn 5 days and O+ve I think with jaundice and signs of hemolytic
anemia with hb 8 and retics High & Total Bilirubin high , what's next ...

my answer ask about mother abo group


(no other choices about sphercytosis I think)

Salam: It is weird that symptoms did not appear until day 5 because pathological jaundice
appears in less than 24 hrs in newborn. Causes of hemolytic anemia: ABO and Rh
incompatibility. The diagnosis of Hemolytic Disease of the newborn requires evidence of
hemolysis in the presence of fetomaternal blood incompatibility.

638- Breast movable mass not painful not related to mensis & increasing in
size, diagnosis? ...

A. Fibroadenoma
B. fibrocystic

Salam: Fibroadenoma is the closest answer (surgery by hamza), Correct answer is phyllodes
tumor

Explanation:

Fat cyst = fat necrosis:Nontender periareolar mass with irregular borders Breast skin retraction,
erythema, or ecchymosis

Fibroadenoma:Usually, a well-defined, mobile mass, Most commonly solitary, Nontender,


Rubbery consistency, Has a relation with menstrual cycle ; often becoming more prominent and
more tender prior to a period.
Fibrocystic changes:
Premenstrual bilateral multifocal breast pain Tender or nontender breast nodules, Clear or
slightly milky nipple discharge

Intraductal papilloma: Solitary lesions (also known as central papilloma)


Most common cause of bloody or serous nipple discharge
Palpable breast tumor close to or behind the nipple or areola
Large, central lesion
Phyllodes tumor: Painless, smooth, multinodular lump in the breast, with an average size of 4–
7 cm
Variable growth rate: may grow slowly over many years, rapidly, or have a biphasic growth
pattern
639- RTA case stable with fructure ribs 3 to 7 then developed respiratory distress
and X-ray showing lung infiltrates (no picture given), diagnosis

A. lung contusion
B. flail chest

Salam: A
similar recall

61 years old women sustain MVA to here left chest brought to ER vitally stable no open wound
admitted for observation, couple of hours later developed suddenly SOB only, initial CXR
reveled 3-7 left ribs features. Repeated CXR showed well demarcated left lung infiltarate. What
is the diagnosis?

A- PE
B- flial chest
C- cardiac contusion
D- pulmonary contusions

The keyword is lung infiltrates on x ray , flail chest comes with paradoxical breathing

640- A case about female with mass at the side of the neck & high creat and urea with
low Calcium and high potassium and high phosphorus

A. tumor lysis syndrome

Salam: Think of “PUKE calcium” to remember the electrolytes affected in tumor lysis
syndrome: Phosphorus, Uric acid, and potassium (K+) are Elevated; Calcium is decreased.
(amboss)

641- Female with Breast abcess just delivered a baby a few days ago , on 4 days
course antibiotic without improvement and the doctor still suspect breast abcess

A. change antibiotics
B. encourage breastfeeding
C. stop

breastfeeding Salam: not

sure

in case of mastitis:
Patients with mastitis should continue breastfeeding to reduce the risk of a breast abscess, but
feeding should only be on-demand to avoid contributing to the inflammatory process (Amboss)
If there is no improvement with oral therapy the patient should be reassessed and vancomycin
or another antibiotic with activity against MRSA should be initiated. (NCBI)
in any breast abscess, Incision and drainage is first line followed by antibiotics, breastfeeding
is not contraindicated in the case if abscess drainage (uptodate)
642- Influenza vaccine is limited .Give to pregnant

Salam: All women who are pregnant or might be pregnant during the influenza season should
receive
the inactivated influenza vaccine as soon as it becomes available and before onset of influenza
activity in the community, regardless of their stage of pregnancy (wafa’s)

Depends on the gestational age


if 1st or second trimester-> I would go with influenza if third trimester -> I would go with Tdap
(wafa’s)

643- Baby 9 month I guess missed 6m and 9m vaccines due to traveling

A. Catch up vaccines
B. give all vaccines

Salam: A
check out catch up vaccine schedule at CDC.
https://www.cdc.gov/vaccines/schedules/hcp/imz/catchup.html#table-catchup

644- Baby during vaccination mom complaining about allergic reaction and going to ER
in the last
vaccination ...

A. Ask about previous

vaccine Salam:

Dr,safdar: As a rule of thumb:

• Any history of anaphylaxis to a vaccine is a contraindication to that vaccine

645- Female with depression and suicidal thoughts which selective reuptake
inhibitors to take

A. Seretonin
Salam: SSRI best for depression (Selective Serotonin Reuptake Inhibitors)

646- Pregnant 31 weeks cervix dilated 4 cm I guess and going into labor and the
doctor gave Oxytocin,
what's its role ...

A. Prevent infection
B. prevent PROM
C. delay the delivery till 37 week
Salam: not sure

647- Signs of Glomerulonephritis in a child, what important to ask about ?

A. Previous impetigo infection

salam: Post-streptococcal glomerulonephritis is a rare kidney disease that can occur after
bacterial infections like strep throat, scarlet fever, or impetigo. (CDC)

648- Typical Preeclampsia case ...

A. Mg Sulfate

salam:Preeclampsia: New onset of hypertension with proteinuria ( ≥0.3 g


or protein/creatinine ratio ≥0.3 (mg/mg)) orend-organ dysfunction
after 20 weeksof gestation.

management: only give Mg Sulfate if have preeclampsia with severe features (refer to Dr.
wafa topic on preclampsia as it is too long to put here)

Q- Another Eclampsia case with seizures


A. Mg Sulfate

Salam: eclampsia: preeclampsia with seizure, manage with ABC then Mg sulfate to prevent
future convulsions

649- Epileptic not taking his medication properly came with 4 convulsions in the last
3 hours and still with disturbed conscious level , what the first thing to give ...
A. Phenytoin infusion

Salam: status epiliptucs,


IV benzo
if no response, IV phenytoin

650- A case Female coming with muscle spasm in her Rt arm and other problems I
can't remember, Ca
very low, 1st thing to do ...

A. (IV

Ca) Salam: A
651- Newly diagnosed DM2 with HbA1c 8 asking about 1st line

Salam: A
Monotherapy with metformin is the first-line initial treatment for most patients.

652- HTN case ...


A. Decrease salt

Salam:
● Lifestyle measures alone may be trialed for 3–6 months in patients with: [5]
○ Elevated blood pressure BP < 130/80
○ Stage 1 hypertension (<139/89) and 10-year ASCVD risk < 10
% pharmacological treatment start when :
● Adults with SBP ≥ 130 mm Hg or DBP ≥ 80 mm Hg and ≥ 1 of the
following:

○ Clinical ASCVD (e.g., ischemic heart disease, peripheral artery disease, or


previous stroke) or congestive heart failure (CHF)
○ 10-year ASCVD risk ≥ 10% (based on the ACC/AHA Pooled Cohort
Equations; includes age ≥ 65 years and diabetes mellitus)

● All adults with SBP ≥ 140 mm Hg or DBP ≥ 90 mm Hg

653- Celiac case (without diagnosis) ...


Salam: the only treatment is gluten free diet
654- Sheigella ttt ?

A. Ciprofloxacin
B. Ceftriaxone
C. Amoxicillin)

Salam: For patients who have no clear risk factors for drug resistance, a fluoroquinolone is
a reasonable empiric option. For individuals at risk for infection with a multidrug-resistant
isolate, empiric treatment with a third-generation cephalosporin (cefixime or ceftriaxone) is
reasonable until susceptibilities are available (uptodate)

655- Prothetic valve prophylaxis before dentist ...

A. Amoxicillin or ampicillin
B. Clindamycin

Salam: A
first line prophylaxis is penicillins

656- Prothetic valve endocarditis ttt ...

A. Vancomycin
B. gentamicin
C. rifampicin

Salam: A
initial therapy is vancomycin

657- A case of aortic regurgitation , Asymptomatic & echo EF 40% without


ventricular enlargement management?

A. observation
B. total valve

replacement Salam: B

according to amboss, surgery indications are

658- A classical case of child with red urine fever and incresed WBCs , RBCs, Protein
in urine …

A. UTI

Salam: could be UTI or pyelonephritis


Safdar file: in children,

• Most common cause symptomatic and gross hematuria : UTI

• Most common cause of recurrent hematuria IgA nephropathy

659- Female not pregnant nor lactating with Uti ttt ...
A. Nitrofurantoin
Salam:A

659. in a patient not being able to urinate for 2 days with other problems and failed
catheterization trial

A. Reinsert Foleys catheter

salam: not sure (not complete question)


Note:

If the saline is not returned or any resistance to catheterization was encountered, underlying
pathology may be present and urologic consultation should be obtained.

If the patient complains of pain during catheter insertion, the catheter should be removed. If
blood appears at the meatus or on the tip of the catheter, a urethral injury may have occurred.
The procedure is abandoned and a urologic consultation is obtained. (uptodate)

659. A case of bloody urine I think with a prostatic problem found during colonoscopy.

A. Transrectal prostatic biopsy

Salam:
TRUS indication:
clinical suspicion of prostate cancer after shared decision-making with a
patient whose life expectancy is ≥ 10 years (amboss)

659. A case about epigastric pain radiating to the back but I think not acute pancreatitis

A. cholangiostomy
B. cholecystectomy

Salam: not enough info so here is an approach to gallstones


gallstones could cause back pain between shoulder blades (NCBI)

660-

↓↓
Naif:
Answer is Endometriosis⬛✓⬛

Differential diagnoses:

◆ Adenomyosis
• Definition: benign disease characterized by the occurrence of endometrial tissue within the
myometrium due to hyperplasia of the endometrial basal layer

◆ Endometriosis is a common, benign, and chronic disease in women of reproductive age that is
characterized by the occurrence of endometrial tissue outside the uterus. The etiology of
endometriosis is not yet fully established; however, retrograde menstruation is one of several factors
involved. Symptoms include dysmenorrhea, dyspareunia, chronic pelvic pain, and infertility.
Treatment is based on the individual disease manifestation and may either involve the administration
of pain relievers and hormonal therapy or surgical removal of
endometriotic tissue. Endometriosis tends to recur, but symptoms and disease spread improve after
pregnancy in many cases, as well as in menopause.

˙†_¡Etiology of Adenomyosis:
◦ Endometriosis
◦ Uterine fibroids
◦ Parity

661- Pregnant sudden expulsion of fetal parts at 20 weeks ..cause?


Rawan:
fetal aneuploidy = chromosomal abnormalities ‫اإلجابة تكون قد‬

Pregnancy loss can occur even in previously healthy pregnancies. If it occurs before 20 weeks' gestation

(∼10% of pregnancies), it is called miscarriage or spontaneous abortion. If it occurs after 20


weeks' gestation, it is called stillbirth or intrauterine fetal demise.

The majority of spontaneous abortions are due to fetal aneuploidy .

Other common causes of spontaneous abortion are maternal disease, trauma, and congenital anomalies.

Source AMBOSS

From MSD manuals → Spontaneous abortion is often caused by chromosomal abnormalities or maternal reproductive
tract abnormalities (eg, bicornuate uterus, fibroids), but etiology in an individual case is usually not confirmed

:‫للفائدة‬
662- Abortion at 8 weeks cause?

Congenital anomalies

663- Pregnant in twins prom G3p2 20 years old what is the cause of high risk
of bleeding??
- Maternal age
- Parity
- Multiple gestation

Rawan: answer is → Multiple gestation


Similar Recall:
https://t.me/Medicine_ExplainedClearly/15065 + https://t.me/Medicine_ExplainedClearly/15066

664- murmur decrease with valsulva


Rawan:

Valsalva maneuver

Most murmurs decrease in intensity during the Valsalva maneuver. Two exceptions are the
systolic murmur of hypertrophic cardiomyopathy, which becomes louder, and the systolic
murmur of mitral valve prolapse, which becomes longer and often louder.

Source UptoDate

:‫للفائدة‬

665- CSF of bacterial meningitis


Rawan:

Similar Recall:

https://t.me/silent_2023m/7413
666- Female with claudication improves by rest she is smoker and obese what to
do Reduce risk factors

- CT angio
- Duplex
- Reduce risk factors

Rawan: answer is → Reduce risk factors


Similar Recall:

https://t.me/Medicine_ExplainedClearly/15076 + Source AMBOSS

667- Patient after abdominal surgery 7days later ..fever chills ...wound is not
infected rectal examination shows ant. Pulge he is stable
- Give Iv antibiotics
- Evacuation of collection
- Laparotomy

Rawan:

668- Pic of slipped femoral capital epiphysis

Rawan:
Similar Recall:

https://t.me/c/1575629606/2+ https://t.me/smlemay/16213 + https://t.me/dec2020q/1311

:‫للفائدة‬

Assessment of slipped capital femoral epiphysis

The angle of Southwick (yellow) is the angle between a line that is drawn perpendicular to a line connecting

the ends of the epiphyseal plate (green) and a line drawn along the longitudinal axis of the femur (red).

A normal angle of Southwick is 12°. Slipped capital femoral epiphysis is classified as follows:

- Mild: 13–30°

- Moderate: 30–60°

- Severe: >60°

© AMBOSS
669- Case of placental abruption with severe bleeding next?

- Cross match
- Blood transfuse with O-ve
Rawan:

670- A disease with higher incidence in women but prevalence is equal in males
and females what is the interpret

Rawan:

671- Long case patient dm htn ...pistol shot associated with..?

Rawan:
672- Pic of failure to thrive

Rawan:

673- 4months old with fever salivation difficult breathing

Rawan:

674- Clear case of cardiogenic shock

Rawan:
675- Septic shock what is the indicator of systemic good perfusion?

Rawan:
676- 4 years old with increased thirst and hunger ...high random blood glucose what
is the cause

Increase insulin
Decrease insulin
Increase renin
Absent glucagon

comment:

- the cause is decreased insulin this is juvenile diabetes

677- Pregnant did OGTT slightly high what to do

Repeat OGTT
HbA1c
Random Bg
Control diet
678- 9 months can't sit alone otherwise normal supports heas can rotate from supine
to prone…

Asses next visit


Reassure and tell mother it's normal for age
Forgot

comment:

- this is gross motor delay

679- Deliverd twins SVD what is the cause of bleeding.

Uterine rupture
Vaginal injury
Retained placenta

680- Ectopic faraway 80km from


hospital BP 110/70
Temp 35.6
What is the causes of admission

Distance
Vital signs

681- RTA severe bleeding from nose and mouth..difficulty in breathing flail
chest decrease air entery
BP 80/50
Oxygen 80%
What to do next?

Blood transfusion
Intubation
Others doesn't make sense

682- Deodinum perforation how to treat

Repair with Omental patch

683- 31weeks pregnant placental abruption with bleeding was managed conservative in
ER what's next
Discharge and follow up as low risk pregnancy Discharge
and close follow up as high risk pregnancy Stay in hospital

684- Patient convulsions for 24 min. Was given IV diazepam but still convulsing what
to do

Esthuximab
IV phyntoin

685- 9 years old suddenly wetting bed

UTI

686- Heavy smoker no history of diseases came for follow up

Colonoscopy
AAA

- Grade A in screening is Colorectal Cancer , screening by colonoscopy ⬛✓

- colorectal cancer grade A in screening.


◆ The USPSTF recommends screening for colorectal cancer in all adults aged 50 to 75 years.

AAA is grade B
687- Sausage shaped hand?

comments:

-sickle cell disease??

-I think psoriasatic arthritis

688- Patient diagnosed with gout multiple times complained of big toe pain treated
with allopurinol not improved ..???

689- patient complaining of chest pain retrosternal for hours ESG ST elevation
patient has history of ischemic stroke since 1 months,most appropriate management
?

Thrombolysis
Pci

690- Pediatric patients injured by nail he is previously immunised what is


tetanus prophylaxi you will give?

691- Pediatric patient complained of red eye and yellow discharge what is
the management?

Topical steroid
Oral antibiotic for 5 or 6 days
Im antibiotics
Refer to ophthalmologist for ophthalmoscope

692- Pregnant case of hyperthyroidism tsh low T4 high us should diffuse


nodular enlargment bp high what is the most appropriate next step:

A. Antithyroid drug
B. Fine needle aspiration
C. Radio active scan

693- Pediatrics pt ( i forget age) admitted to PICU with subdural hematoma ( I think
but im sure it's a Brain bleed) , The father said he fell from his bed ( this all the hx ) on
PE the pt. Had ecchymosis on the buttocks and Back and he have bilateral femoral
fracture.
Asking about Dx ?

A) Hematoma
B) Pathological Fracture
C) Battered child syndrome

694- 17 years old girl complained of anorexia nervosa was admitted for initiation
nutrition shortly the blood samples revealed hypophosphatemia hypokalaemia
hypomagnesemia what is the cause of the lab investigation?

A. Wrong protocol of nutrition


B. Wrong test
C. Refeeding syndrom

695- 31 years old builder with inguinal swelling for 3 years asymptomatic the
swelling extends to scrtum with positive cough impulse what’s the management?

696- Adult patient typical case of intestinal obstruction x ray revealed y shape
mass what is the appropriate management,

A. Enema
B. Colonoscopy

697-
↓ ‫جات‬
CURP65
‫الجواب‬
outpatient managment
CHA2DS2VASore ‫تحسب‬

698-
‫سؤال عن ال‬
GCS
moderate ‫الجواب‬

699- Pregnant with negative ruppla when to give vancine ?

Postpartum

700- Pregnant and Folate dose?

-701 ‫جاني سوال ناسيه التفاصيل بس يوصف القاوت وحط خيارات من ظمن الخيارات في‬
B12 & gout

Gout is an inflammatory crystal arthropathy caused by the precipitation and


deposition of uric acid crystals in synovial fluid and tissues.
Decreased renal excretion and/or increased production of uric acid leads to
hyperuricemia, which is commonly asymptomatic but also predisposes to gout.
Acute gout flares typically manifest with a severely painful big toe (podagra) and
occur most often in men following triggers such as alcohol consumption. Diagnosis
is based on clinical presentation and, ideally, by the demonstration of negatively
birefringent monosodium urate (MSU) crystals on synovial fluid analysis

affected male more than female (2 peak incidence 30-39y &60y)


causes:
For diagnosis :
‫من ضمن الشياء المهمه‪:‬‬
‫‪ambos‬‬ ‫من ضمن السئله الي اتكررت اكتر من مرا بخصوص ال ‪ ، gout‬الجابه ‪ , C‬المصدر‬
‫‪ELAF‬‬ ‫‪s‬‬

‫‪702-‬‬
‫سوال عن انتيرن داخل‬
‫‪OR‬‬
‫والمريض بيسون هل شي كب ليته اليمين وهو المشكله باليسار شيي زي كذا ايش تسوي‬
‫؟‬
‫‪best recall with correct answer‬‬

‫‪https://t.me/silent_2023m/5607‬‬
‫‪ELAF‬‬

‫‪703-‬‬
‫؟جاء تطعيم الي قبل دال راسه ايش هو‬

‫‪OPV‬‬

‫‪correct‬‬

‫‪ELAF‬‬
‫‪704-‬‬
‫طفله عمره ‪ ٢١‬سنه جسمها كدمات‬
‫المعلومات غير كافيه‬

‫‪705-‬‬
‫طفل يتدحرج المهاره هذي ايش تكون ناسيه الخيارت بس كانه كان في شي عن‬
‫‪White matr‬‬

‫‪ELAF‬‬

‫‪706-‬‬
‫كان في سناريو عن ال‬
abortion
‫ان‬ ‫كان اذكر‬
complet
‫النه‬
Co close pass tissue

similar recall?
with description

https://t.me/WafaOBGYN/447

ELAF

707-
‫مريضه جايه وعمره كبير وعندها اعراض سن اليأس العالج >هرمون ربليسمن‬

‫‪ELAF‬‬
‫‪------------‬‬
‫جاء اساله كثير عن السوي‬
‫‪ Fast‬او ‪CT‬‬
----------
‫في كثير اساله عن تروما‬
---------

708-
‫>> سوال ولد يبلل سريره العضله المسؤله‬

Detrusor recall similar ‫العضلة المسؤلة عن سلس البول‬

correct
ELAF

709- 32 y male with red urine for 1 week with burning mecturation and can't urinate for
24H
Urinalysis normal
Culture no growth
PR enlarged tender prostate
Diagnosis:

UTI
BPH
Prostatitis

ELAF
710- 34y Pregnant G3p2 k/c of hypothyroidism her baby has single umbilical artery what
is the cause

Maternal age
Multiple gestation
Parity
Thyroid disease

‫مافي اي مصادر موضحة عالقة اي وحده من الخيارات‬

similar recall :
‫بإجابة مختلفة‬

https://t.me/silent_2023m/768

ELAF
—————
‫قريب من صيغة هذا السؤال قالك جا اول مرة ونفس الخيارات بس اخر خيار مكتوب ‪ epinephrine racemic‬انا اخترت‬
‫‪0.2 prednisolone‬‬

‫نفس صغية السؤال لاإ جابة جابها مرتين مرة قال الجواب ‪ kawash‬ومرة ‪deficiency protein sever‬‬
—————

711- The nurse repeating what the doctor said just to make sure.
A. Check back

Alaa: A

similar recall

‫ فده اسمه ايه‬iv epinephrin ‫ وهى ردت عليه اعطيه‬IV epinephrin ‫ تعطى المريض‬nursr ‫سؤال عن حالة الدكتور قال لل‬
A. call out
B. Check back

‫كان في سؤال طبيب ومعاهم المريض وهما في السعاف الطبيب كل شوي يسال المسعف‬
Air way
‫ الطبيب يسال‬clear ‫المسعف يرد‬
pressure blood
‫والمسعف يرد وهكذا‬
What is the method used
A- Call out
B- check back

-712 ‫هاذي من األسئلة الجديدة ما مرت علي وماني متذكر الصيغة بالزبط‬

Pt have CKD need dialysis what best site ?

Av
Tunnel
Non tunnel
-AV fistula
If urgent dialysis then Tunneled

checked from amboss


ELAF

713- 2 years hectic fever with lymphadenopathy and rash

Measles
Rubella
Kawsaki

description for similar recall (SMLE-B)


ELAF

714- Female after vaginal delivery and placenta still didn't come out

Wait until placenta delivers spontaneously


Start manual removal of placenta

comment:
What about active management for 3rd stage of labour ( brand Andrew menovoure ) with
Mythergin 0.2 IM + oxytocin infusion 10 IU + uterine message.

ELAF
715- Female comes for screening of breast cancer asking what type of nipple
discharge should worry her

Spontaneous and bilateral red


Spontaneous bilateral milky
Induced unilateral green
Spontaneous unilateral single duct and clear

from amboss
ELAF

716- Patient RTA GSC 8 what to do next..

Intubation
Blood transfusion
https://pubmed.ncbi.nlm.nih.gov/19272743/

ELAF
717- Female has severe menstrual pain that affects her daily activity

Progesterone only
Ocps
(No NSAIDS in choises)

ELAF

718- 60 y female urination with cough no history of frequency or urgency (no


other information)

Cystocele
Rectocele
Forgot
incontinence stress ‫ ال‬cough ‫مني متآكده بس دايما الي مرتبط ب‬

ELAF
719- 25 years has menstrual pain asks for advice to reduce pain

Do exercise
Don't drink caffeine
Bed rest
Hot path

not sure
ELAF

720- Which type of exercise suitable for menstruation pain

Cardio
Aerobics
Running

from similar recall


ELAF

721- Female on hydrocortisone takes 3 doses and takes night shifts What should
she do?

Takes medication on time regardless night shift Or


others
Bad recall can't remember properly
Similar:

Q: Female with symptoms then diagnosed with addison’s disease and started on
hydrocortisone and flurocortisone At 9:00 12:00 15:00 Then she informed the doctor
that she have night shift What is appropriate?

A- No change in the treatment plan


B- Emit using cortisone during her night shift
C- Take the first dose at 9:00 then the rest at her night shift
D- Take her first dose at the beginning of night shift then 3 hours later then 6 hours later

Answer: D

https://t.me/smlemay/17765
ELAF

722- Pregnant and has child in school asks what should she take to reduce
infection risk?

Influenza vaccine
Rubella vaccine

influenza & rubella is live attenuated vaccine which is contraindication in pregnancy


So, i don't think the answer is one of these

ELAF

723- Femal paraumblical mass negative impulse on cough skin overlying thin..what
to do?

Immediate surgery
Us
Ct
maybe CT?

theres no clear resource but similar recall showing:

ELAF

724- Frequency + urgency


Urinalysis ( 10000
shegella)

A Prophylactic antibiotic
B. Treat as uti case
not sure
ELAF
725- Epileptic not taking his medication properly came with 4 convulsions in the last
3 hours and still with disturbed conscious level , what the first thing to give?

A. Benzodiazepine
B. Phenitoin
C. Valporic acid

Naif:

‫هل تعتبر‬
Refractory status epilepticus ?

726- 45 years has HTN. And on


atorvastatin best advice?

A. Regular exercise
B. Decrease salt intake
C. Two other irrelevant answers

Naif:
◆ Statins are the first-line therapy for hypercholesterolemia.
Treatment of hyperlipidemia with statins significantly reduces the risk of mortality in patients
suffering from CAD.

hypercholesterolemia cause atherosclerosis then lead to coronary arteries disease & HTN & DM

My answer is modified lifestyle

727- Croup S causitive organism?

A.Para Influenza virus


B.Respiratory syncytial virus

Naif:
◆ Most common pathogen: parainfluenza viruses (75% of cases) [2]
Look here ⬛J

↑ ‫˙¡_†هنا فيه اح مت الت‬

728- Acute epiglottits disturbed with stridor


Best action?

A. Chest x ray
B. Bronchoscopy
C. CT chest

( I am sure no intubation or lateral neck x ray)

◆ If the diagnosis is uncertain and there are no signs of impending airway obstruction:
◦ Visualize the epiglottis to confirm the diagnosis.
◦ Consider imaging to confirm the diagnosis if:
▪ Visualization of the epiglottis is unclear or unsuccessful.
▪ Alternate diagnoses need to be ruled out, e.g., croup, abscess, foreign body aspiration.
‫اعتقد هنا لاا جابة‬
729- Colicky pain in RUQ usually after eating of fatty meal resolved spontaneously after
2 hours
No other symptoms

A. Biliary colic
B. Gastritis
C. Peptic ulcer
D. Cholangitis

true

ELAF
730- Severe Testicular pain 4 hours with elevated tender testi, Dx?

A. Testicular torsion

ELAF

731- Case of cauda equina best action?


A. CT spine

‫ اصح‬spine MRI ‫ال‬

ELAF

732- Ruptured ectopic pregnancy ttt?

A. Surgical Laparotomy
B. Reassure
C. CT abdomen

(no salpingotomy or salpingectomy in the answers)

ELAF
733- 30s y woman asking for effective and reversible method of contraceptive, which
if the following is the most effective:

A. Vaginal ring containing hormones


B. Female condom
C. Male condom
D. Spermicide
ELAF
734- Pregnant in third trimester has vaginal prolapse grade3 ,CTG
normal Vital normal what to do:

A. CS
B. Reassurance
C. induction of labor

not sure
ELAF
735- GDM on diet not controlled, ttt?

A. Insulin

ELAF
736- Baby with abscent Moro reflex what is the Dx?

I Did not remember the answers

Absence bilateral = damage brain or spinal cord


Unilateral = broken shoulder bone or an injury brachial plexus

Also
Q: Baby with abscent Moro reflex on one side what is the Dx

I selected >> Erbe's palsy

Did not remember the other choices

Naif:
https://t.me/smlepaediatrics/300

Q: an infant born via vaginal birth has an abdominal appearance of the right arm, which
hangs limply by his side and is rotated medially the forearm is extended and pronated
and the infant is unable to raise it. Which of the following muscles is still functional in
this patient?

a- triceps
b- supraspinatus
c- biceps
d- deltoid

-Erb palsy.

———
Pictures

# Dx. P. Falciparum

# Pericarditis diagnostic - - - echo #

Iron deficiency anemia >>>



737- It was decrease hb, he asked about what will be decreased also
The answers was

A. Mcv
B. Paltelet
C. Reticulocytes
D. Neutrophiles
ELAF

738- Mother brought boy 5 years old with squint and double vision?

Call neurosurgery immediately.


Refer to pediatric ophthalmology
MRI for brain occupying lesion.
Septic work up and LP

ELAF

739- Child in ER with DKA but the physician only gave hydration and discharge
and patient became worse?

Error of omission
Error of commission
Error in documentation

ELAF
740- Neurfibrmatosis case?

Autosomal dominant
Autosomal recessive
X linked dominant
X linked recessive
ELAF
741- Female with vaginal bleeding during intercourse and pain, imaging shows 9x9
cm mass heterogeneous calcification?

Watchful observation.
Laparoscopy
Forgot

not sure
ELAF

742- Old patient states that left lower abdominal is recurrent since two years, imaging
show sigmoid narrowing and mural normal?

Diverticular disease
Ischemic colitis

743- Couples asks about surrogacy


Your hospital doesn't provide it and you don't know if it's legal in ksa What to do ?

Tell them hospital doesn't provide it


Tell them it's illegal in ksa
Refer them to collegue who is connected to surrogacy centers
Tell them come after 4 weeks so you discuss it after knowing about it

744- Best investigation to confirm endometriosis

Laproscopy
US
Endometrial biopsy
Answer is Laparoscopy (confirmatory

test)⬛✓ Endometriosis:

◆ Diagnostics
• Patient history
• Physical examination
◦ Rectovaginal tenderness
◦ Adnexal masses
◦ Lateral displacement of the cervix (due to uterosacral scarring)
• Transvaginal ultrasound (best initial test)
◦ The uterus is generally not enlarged.
◦ Evidence of ovarian cysts (chocolate cysts)
◦ Nodules in bladder or rectovaginal septum
• Laparoscopy (confirmatory test): may show endometriotic implants and adhesions
745- 17 yrs old man bloody diarrhea mild pain an fever , had this pain before but milder
just mucus in stool and frequent defecation O/E mild tenderness over LLQ no guarding
Diagnosis?

Diverticulitis
Crohn's
Ulcerative colitis
Ischemic colitis

746- Child in ER with DKA but the physician only gave hydration and discharge
and patient became worse?

A. Error of omission
B. Error of commission
C. Error in documentation

747- Neurfibrmatosis case?

A. Autosomal dominant
B. Autosomal recessive
C. X linked dominant
D. X linked recessive

748- Female with vaginal bleeding during intercourse and pain, imaging shows 9x9
cm mass heterogeneous calcification?

A. Watchful observation.
B. Laparoscopy
Forgot other

749- Old patient states that left lower abdominal is recurrent since two years,
imaging show sigmoid narrowing and mural normal?

A. Diverticular disease
B. Ischemic colitis

750- Pt with symptoms and signs of acute cholecystitis ( diagnosis not given
) Best appropriate test

Trans abd ultrasonography


MRCP
ERCP
Abd CT

751- Company want to study subgroup of people for prevalence of Alzheimer's disease
, what method used by the IRB to check the ethical point

Utilitarianism
Skeptic ethics
Delogatory ‫ كذا زي شي او‬Forgot last
one

752- Mother with her child who has febrile , not vaccinated at all , mother refusing
to vaccinate him What to do ?

Vaccinate him
Treat the disease and ignore the vaccination
Educate the mother about vaccination
Refer the child to a pediatrician

753- Child blood in diper recurrent for 2 months , no abd pain or


tenderness Diagnosis

Juvenile polyp
Intussusseption
Mickles diverteculim
Forgot

754- The best of these to confirm nephrotic syndrome

Hypoalbuminemia
Hypertension
Hematuria
Forgot

‫كل الختيارات تعتبر‬


Classic manifestations& Clinical features

proteinuria ‫و الصح اللي انا اشوفه‬


Massive

‫او‬
renal biopsy
755- Woman diagnosed with cystasarcoma phylloids , tumor is 15*15 cm , management:

Radiation
Simple mastectomy
Neoadjuvant chemo
Modefied mastectomy

756- Child can skip , draw rectangle, count to 10 , age in years

2
3
4
5

757- Young man came with syx and sings of SBO , ( dx not given )
What is the most appropriate to do

Ct
Laproscopy
Examine the groin
Forgot

758- 15 months old child , reducible rt inguinal hernia, no


symptoms What to do

Mesh repair
Observe Repair
at 6 yrs
Lap exploreation

759- Female ectopic pregnancy، with severe pain , doctor is not around , nurse call
him, what can she give the pt with phone order :

Methotrexate
Pethidine
Paracetamol
Forgot

760- Pregnant diabetic


What's the best indicator of fetal wellbeing

Glucose level
GTT
HBA1c
Nuchal translucency

and also
There are 3 q about hernia In
children and adults

761- Bone age delayed with normal development child and normal
hight, Tx?

- Sun exposure
- more worke up ie:cbc,LFT, etc

762- Olive like mass with


vomiting, Tx:
pyloromyotomy

763- Cirrhosis liver (Dx given) what to do?

- water restriction
- salt restrictions
- protein restrictions
- high glycemic food restrictions

764- Cow milk consumption with low MCV and anemia, Dx:

IDA

765- Pt with dysphagia to solid and liquid and dull retrosternal chest
pain, What to do ( CA sus) :

- manometery
- endoscopy
- barium

766- Child complain of Coin ingestion, on x-ray: coin appear mid trachea, what to do?

- idk I chose Endoscopy

comments:

-Rigid Bronchoscopy

767- Child with organomegaly and pancytopenia,

- AML ( I chose it )
- ALL

768- Child with fever and sore throat, and there’s exudate membrane covering
the tonsils, and neck lymphadenopathy.

Coronavirus
H Influenza
RSV
Mononucleosis

769- Toxic looking Child, drooling, and vital unstable, What to do:

-ICU with intubation

770- Infant of IDDM


mother, High risk for what:

- respiratory distress syndrome

771- Adults with 2 days hx of Lt Abdominal pain and exaggerated bowel sound, Dx:

constipation

772- greenish brown discolouration around cornea, Dx:

idk, but u chose Wilson disease

773- Anemic pt with Low MCV, normal Iron, normal ferritin, ( in Q he mentioned "lead"
in some what ) Tx:

D-pencillamine

774- Wooping cough with pertussis

775- Staccato cough with chlamydia pmemonai

3 Qs about ↓
776- Asthma treatment and what drug u will use in step up or step down

777- Cause of coffee ground vomiting

—————
—————

778- Preterm 30 weeks , 3 hours after delivery developed tachypnea and desat , cause :

Hyaline membrane disease


Meconium aspiration
Transient tachypnea of the newborn
Forgot

779- Patient with syx and signs of pertussis. Investigation:

X ray Pharyngeal
swap Serology test
Ct chest
B?

780- Newborn , microcephaly , micrognathia , epicanthal folds ( seems like


Edwards syndrome ) what is the expected endocrine problem :

Hypothyroidism
Hyperthyroidism
Panhypopituitarism
Hypoparathyroidism

Ref: Medscape

Edward syndrome features. I


am with A
‫‪781- Picture of female genital with multiple lesions all over, history of multiple‬‬
‫?‪sexual partners, what is your diagnosis‬‬

‫‪A. condyloma lata‬‬


‫‪B. basal cell carcinoma‬‬
‫‪C. molluscum contagium‬‬

‫———‬

‫نسبة كبيرة من األسأله اوبيقايني ومديسن و الساله الباقية جراحة و اطفال‬


‫و ‪ ٦‬اسأله اثكس‬
‫و سيكاتري سؤالين قت ريبا‬

‫و فكرة عن بعض السأله جا سوال عن‬


‫‪ heart the of base the at heard murmur‬ما اتذكر‬
‫السوال كامل‬
‫جا يمكن سؤالين او ‪ ٣‬ذف اكروها ‪ murmur‬بشكل عام ال‬

‫و‪ ٤‬اسالة تقريبا من مايلستونز‬

‫سؤالين لكن ما اتذكر السوال ‪ asthma‬جا من ‪ crohn’s‬وجا عن‬


‫ما يخلو اي اختبار منها حتى لو سوال واحد وسهله تضمنونها ذاكروها واعرفو اذا ‪ GCS‬و‬
‫) ‪( mild or moderate or severe‬‬

‫‪ :‬الصور‬
‫لكن ما حصلتها عشان ارسلها ‪ECG vagina‬‬
‫‪ patient‬و صورة وحدة لـ‬
‫‪ partners multiple has She question the in written is‬و الصورة‬
‫كانت انتفاخات متعدده متوسطه الحجم الى كبيرة في‬
‫‪ minora labia and opening vaginal , vagina‬ال مو مذكورة‬
‫بالخيارات ‪ polyp‬لكن ال ‪ polyp‬كانها‬
‫برسل صوره مقاربه لها ان شاء هلال احد يعرف ‪ recall bad‬المعذرة على ال‬
‫‪ ultrasound for twins‬صورة و‬
‫برسل صورة مقاربه من قوقل لكن نسيت ايش كان طالب السؤال فيها ‪ may 20‬واذا احد اختبر اليوم‬
‫ًممكن يتذكرون الساله ويضيفون لنا ويعطيكم العافية جم عي ا واذا تذكرت شي زيادة بأرسل‬
‫فإن أحسنت فمن هلال‪ ،‬وإن أخطأت فمن فن سي والشيطان وهلال يوفق الجميع ‪ ،‬دعواتكم‬
Uterine leiomyoma

Hypoechoic ( Black ) = Fibrosis ( Leiomyoma ). Hyperechoic

( White or increase in echoginicity ) = polyp. Both came in the

exam.

‫هذا الشارت‬
‫مو فن سه بس مقارب هل بنفس النقاط‬

‫هذي مشابهه لتوأم‬


‫هذي مقاربة لصورة الـ ‪ vagina‬كل ن ال ‪ polyp‬كانت على كل المنطقه‬
‫ال ‪vagina , vaginal opening and labia minora‬‬

‫و صورة وحدة لـ ‪vagina patient‬‬


‫‪ partners multiple has She question the in written is‬و الصورة‬
‫كانت انتفاخات متعدده متوسطه الحجم الى كبيرة في ال ‪minora labia and opening vaginal ,‬‬
‫‪vagina‬‬
‫كا هن ا ‪ polyp‬لكن ال ‪ polyp‬مو مذكورة بالخيارات‬

‫تذكرت كان من ضمن الخيارات ‪ cancer‬بس م اخترتها وال اتذكر وش اخترت‬


‫‪Dx‬‬
‫?‬

‫‪maybe genital warts‬‬


CSF Result
782- Young female complain of RLQ pain , and fever , there is tenderness and guarding
. Pregancy test is negative
Wbc is high . Diagnosis :

Ovarian torsion
Acute appendicitis
Cystic rupture
Ectopic pregnancy

783- The correct interpretation of CTG of these :

Cord compression causes early deceleration Cord


compression causes variable deceleration Head
compression causes late deceleration Head
compression causes variable deceleration

784- Q of the soldier going to the south , give what drug :

atovaquone proguanil

comments:
Atovaquone is no longer used as a monotherapy to treat or prevent malaria because one-third or more
of individuals with P. falciparum infections will recrudesce (Srivastava and Vaidya, 1999).

785- Old female with history of epilepsy, had seizure and fell down the stairs.
Presented with back pain and urine incontinance and inner thigh pain.
Next appropriate management?

EEG
MRI Lumber
Forgot

786- Child came with sore throat, red eye, runny nose. History of big brother with
same symptoms Dx?

Adenovirus pharngytis
Infectious mononucleosis
Forgot

787- 40 years old female healthy, previous history of DVT?

Warfarin
Aspirin
Enoxaparin
No need

788- Old male with swelling in neck, no symptoms of hyper or hypo thyroid,
no symptoms of obstruction.
Labs calcitonin high
Dx?

Lymphoma
Anaplastic
Medullary
Papillary

789- Patient diagnosed as DM1 and started on aspart and glargine. Patient has episodes
of hypoglycemia and A1C is 7 or 6 not sure. What will you change in treatment?

Change to NPH twice daily


Stop aspart and only use glargine Decrease
the dose of aspart and glargine
Use mixtard insulin

790- 39 year old male had a pelvic fracture from MVC rollover 2 months back
that treated surgically. Before 2 hours he started to complain of sever left LL
pain.
Femoral pulses are intact. I.V heparin started and venous US is negative. What is
the the best management?

A. Catheter thrombolysis
B. Embolectomy
C.Observation
D. Amputation

791- Left Calf and left foot pain couse by any of V ?

A. S1 S2
B. L1 L2
C. L4 L5
D. L5 S1

792- 30 years old healthy female came for regular health checkup. She was found
to have high blood pressure. What tool is best to educate here about the risk
factors

A- health belief model


B- health promotion model C-
theory of reasoned action
D- diffusion of innovation

-The effect of education based on health belief model on promoting preventive behaviors of
hypertensive disease in staff of the Iran University of Medical Sciences.

793- A 66 years old patient did CT angio 4 days ago, he cMe today complaining of
oliguria and abnormal renal function,
What is he having ?

A-UTI
B-Renal cortical necrosis
C-Acute tubular necrosis
D-Renal stone
794- Mother brought baby 18 months for vaccine, states that she had sore throat 3
days before and is now better. You exam and hyperemic throat.
Appropriate action?

Give vax but Don’t give oral polio


Give vac but Don’t give dtap
Give after antibiotics duration ends Postpone or
skip (not sure exactly 4th option)

comment:

As per CDC and Safdar - Children can still get vaccines – even with a fever or mild illness. Because a
mild illness does not affect how well the body responds to a vaccine, your child can still be vaccinated
if he or she has: A low grade fever. A cold, runny nose, or cough.

Children taking antibiotics can get vaccines

I feel something is missing in the recall its not clear - id give all vaccines if sore throat

795- Question recall not complete.


Patient after cardiac or hypertensive cause stabilized and discharged. Patient came and
complaining of rash on neck, trunk and edema i think. What medication caused
reaction?

Ramipril
Aspirin
forgot rest meditations

796- 4 years baby visits for immunization but baby dx Cushing syndrome on
medication (steroid???

A. def live vaccine


B. low does of live vaccine .
C. Reived all vaccine

797- Female post cholecystectomy 7 days presents with RUQ pain and jaundice
(forgot if fever was present or not), imaging shows CBD dilation 7 mm not sure.
What is the cause?

Clips slippage
Forgot the rest
Naif:

I don’t know

But my answer is
choledocholithiasis cause obstructive jaundice.

‫اس تتن اج من بحثي في مآ بوس يعني‬


I’m not sure

798- Old patient came with history of poor cognitive function MRI shows diffuse
white matter hyperintensity

1. Alzheimer’s
2. lewy body demnetia
3.vascular cognitive disorder
4.normal hydrocephalus

Naif:
My answer is Vascular dementia
‫يعني‬
3- Vascular Cognitive Disorder✅

High Yield Smle notes ⬇️


https://t.me/c/1625198149/462

799- Case of epiglottis options were weird what the definitive next step of
management?

A. Chest xray
B. Upper git examination
C. Upper airway CT
D. Laryngyoscopy in OR room
Naif: D I’m not sure

800- Ovarian mass 7mm asked about tumor marker

A. Cea
B. Ca 125
C. Bhcg
D. Ca19

Naif:

My answer is B-CA-125✅

Tumor Markers:

1- Epithelial ovarian tumors: CA-125 is elevated in ∼ 80% of malignant tumors.


2- Germ cell tumors:
◦ Dysgerminoma: LDH, β-hCG
◦ Yolk sac tumor: AFP
◦ Immature teratoma: AFP, LDH, CA-125
◦ Choriocarcinoma: β-hCG
◦ Embryonal carcinoma: AFP, β-hCG

3- Sex cord-stromal tumors:


◦ Granulosa cell tumor: inhibin

801- Itchy polyps in the labia bleeding sometimes, asked what is it

A. Bartholin cyst
B. Bartholin carinoma
C. Squamous cell carcinoma

802- Father diagnosed with prostate cancer son goes to check Dre shows smooth
no enlargement in prostate can feel medial sulcus What next step in management

A. Reassure
B. Dre annual
C. Psa 2 weeks later
Forgot the last option

803- Patient twins monochromatic monochromatic one is breech the other


cephalic, What is the next step in management
A. Cs
B. Wait till 39 week
C. IOL vaginal

Naif:

804- Long case of patient that literally has nothing but abdominal pain all labs and
test are normal what the next step in management

A. Do ultrasound now
B. Refer to gastroenterologist

805- Patient hypertension pregnant on labetol deliver healthy baby


What will happen in future pregnancies that labetol is a risk factor
of

A. Uterine rupture
B. Adhesion placenta
C. IUGR

806- Depressed patient on paroxetine stable now she wants to conceive What is next
in management

1.Stop paroxetine because it increases contractions


2.Continue paroxetine
3.Stop because fetal damage
4.Change medication

807- Patient came with parent weird ocular movement and something wrong with
eye the parents said he missed the 13 month 18month vaccines What’s the
diagnosis

1.Orbital myositis
2.Orbital cellulitis
3.Peri orbital cellulitis
Can’t remember the last one

808- Parents smokers came with 13 months child with a lot of problems
tachycardia tachypnea failure to thrive other consultant suggests Dnr What’s your
next step in management ?

1. Ask opinion of 3 other consultants


2. Inform the parents and discuss the option of dnr
3.Do not discuss with parents
4.Ask opinion of another consultant
And 3 other physicians

809- postglumeronephritis case red Color urine after 2 weeks of strep disease
what’s the diagnostic step?

ANSWER : low c3 complement

810- Depress female on paroxten now pregnant


‫تكمل وال توقف العالج‬

I chose to continue and observe depression symptoms

811- Pregnant with vaginal bleeding student want to do pv what to tell somethings
like that
812- Female with axillary and neck pigmentation(they didn't mention obese or Diabetic)

-stria
-ackantha Nigra( i choose it)

813- Pentrating abdomen stable Next step

Ct
Fast
Laparotomy

814- 18 years 2nd degree burn on lower extremities, What to give

Ns Bolus
Ringer 5 litre first 6 hr

815- Ejection murmur radiate to carotid with


dyspnoea I am sure Didn't mention if systolic or
diastolic

2
As
Ar
M
r
Another choice mr or ms

816- Doctor prescribed angiotensin receptor block

Isbrtan

817- Infertility for 6 or 8 month with dysmenorrhoea ,what's couse

818- Old female with slowly growing mass or lesions on vagina bleed when touch

One of choice is basal cell carcinoma


819- Pt insist for hormonal replacement doctor say no indication for
her What to do

820- Lung nodule 1 cm sharb border incidentally on ct abdomen


He is Ex smoke since 20 years

821- Known endometriosis tx

822- Pregnant with headache epigastric pain high bp GA


34 Mx

823- Pt whant tube ligation or


hysterectomy Consent from whom

824- Polyhydramnios associated with


Trisomy 21 or down syndrome

825- One Question about Apgar

826- One Question about GCS

827- Female start Thyroxine 2 week before come now for check tsh high

Continue Same dose


Change dose

828- Breast feeding female whant


contraceptive Iud not in choice
There is vaginal ring and depo injection

829- Female heavy cycle whant contraceptives

830- Baby born jetring what initial investigation

Glucose i think the answer


831- Known Shigella case asking about antibiotics treatment

832- Pedia pt fever 39 wbc high with complete urine


result Nitrate was positive
What make you suspect uti

Nitrate - wbc or fever

833- Pt i think he was child discuss with his parent about dnr he suddenly arrest
before completing the decision

Do cpr and discuss later or


dont do cpr

——
Pic about bundle branch block
Pic about svt
Pic of endometriosis
Pic about red reflex
——

834- Bed ridden from rehabilitation with


obstruction Imaging show coffee bean appearance

835- A 45 year old with history of pancreatitis , then recurrent vomiting, + upper GI
bleeding After resuscitation what should be done Splenomegaly, normal portal ,
thrombo-splenic . He became
stable after injection of sclerotherapy , which of the following best way to management
?

A. Splenectomy
B. Splenorenal shunt
C. Venocaval shunt or portocaval shunt
D. None
836- child w fever vomitting and petechial rash

A-rocky mountain fever


B-meningococcemia

837- Female complaining of decreased ability to walk the same distances, feels pain
in legs, after some distance she reaches her limit and cannot walk, she feels better
when walking while leaning forward, or when walking uphill. diagnosis?

A. Spinal stenosis
B. Disc herniate

838- Pt known copd on home o2


What best predict his severity

One of choice exacerbation per year

839- Open book pelvic fracuture + blood in the meatus, most appropriate?

1. Folys
2. Retrograde urothrogram

840- Contraindications of ECV ??

A. Anteral placenta
B. Oligohydromines

841- 60 yrs old came with bleeding per rectum since 3 months Examination
bleeding 3rd degree hemorrhoids on DRE What to do

A. HEMORHOIDECTOMY
B. COLONOSCOPY

842- Asymptomatic fibroid 5cm subserous

A. Observe
B. Myomectomy
C. Hysterectomy
—————

The Answer is A
↓————— ◆◆◆◆◆◆◆◆

843-
‫جالي سؤال عن‬
‫‪gall bladder polyp‬‬
‫حجمها ‪ ٢‬سم نعمل معاها ايه‬

‫‪ gallbladder of removal surgical‬أل هن ا‬


‫اكبر من ‪١‬سم‬

‫‪844-‬‬
‫جالي صورة‬
‫‪syndrome down‬‬
‫ومحتاج نختار ‪triosomy 21‬‬

‫‪845-‬‬
‫‪ tag skin‬جالي صورة‬

‫‪846-‬‬
‫حالة زيها‬
‫‪bleeding per rectum‬‬
‫وكان‬
‫‪ unstable vitally‬ومبيوقفش‬
‫نزيف وال‪ vitals‬متصلحتش رغم نقل الدم انا‬
‫اخترت‬
‫‪fresh frozen plasma‬‬
‫وكان من ضمن الختيارات نعمل منظار ح ال‬

‫‪847-‬‬
‫‪ screening colonoscopy‬جاني ايضا سؤال راجل عنده اربعين سنة وهنعمله‬

‫هل هنبدأ دلوقتي وال عند سن الخمسين كل ‪ ٠١‬سنوات او كل ‪ ٥‬سنوات‬

‫‪848-‬‬
‫كان فيه سؤال انا جاوبته‬
‫‪ cryoprecipitate‬بس‬
‫مش مجمع السؤال‬

‫‪849-‬‬
‫كان فيه سؤال واحدة عندها‬
‫‪DM‬‬
‫وهي‬
‫‪Pregnant‬‬
‫تاخد اي نوع‬
‫ انا اخترت‬insulin

850-
‫جالي سؤال‬
8yrs old mc site of eczema

851- Pt HEAVY Smoker with symptoms of chronic limb ischemia suddenly severe
cold rt ll What makes you suspect ALI

SMOKING 40 CIG PER DAY


INTERMITTENT
CLALUDICATION
IF YOU SEE SCAR FOR BY PASS ON OTHER LEG
PAIN AT REST

-852 ‫سؤال حد كان ف‬


match
‫وسيناريو‬
Rectus sheath hematoma
‫لكن السؤال‬
next step
He was vitally stable

A. Ct abdomen
‫ تعديل‬.Exploration B

‫على الريكول‬

‫هذا جاء لي أنا حطيت خيار‬


‫ الخيار الرابع ناسيه‬hematoma conform to Aspiration
comment:

CT abdomen

Refuse -853 ‫دكتور جاب هل مريض هديه‬


Accept
Accept and report to department

854- Nurse tell you about patient in ICU (in the case she talk about his history and
recent change and tell to come see him) what is this part of communication:

Situation
Background
Assessment

855- Piqunil pregnancy:

Stop
Continue

856- Folic acid for pregnant+ she has SCA:

A. 5mg for 12 week


B. 5mg for whole pregnancy
C. 400 micro for 12 week
D. 400 micro for whole pregnancy
857- Iron dose in pregnancy

A. 1000
B. 2000
C. 3000
D. 4000

I don’t remember if with IDA or no

-think I w. 2 of Milestone 858 / ‫يرفع راسه ويحركه يمين و شمال و يبتسم‬

-y. 2 of stone Mile 859 / ‫ يكون جملة من كلمتين‬, ‫يمشي ع ورا‬

860- ‫ بعدين بسيطة مسافة يمشي‬leg pain what’s the appropriate management?

Ct angio.

861- Cord prolapse with ?

Variable deceleration

‫ م اعرف الجابه بس اخترت‬-862 ‫ام حامل وتدخن وتبي تقطع التدخين و ماتقدر وش البدائل‬
nicotine patch

863-9 mo. 1st vaccines ?

864- penetrated neck at zone 3 ?

Endovascular intervention

-865‫فيه سؤال في مارس مريضة قلب ومدري ايه وبتخلع ضرسها وش ال‬
prophylaxis ?
‫ الختيارات مافيها‬need no

-866‫فيه سؤال في ملف مايو عن مريض يكح و ال‬


spoon on sputum ‫كميته يعني‬
867-extended mediastinum ?

With aortic injury

-868 ‫ اذا الب حامل و اثنين من اعمام لاا طفال عندهم النيميا لكن الم ماعندها وراثه وما عملت‬١ ‫سؤالين عن النيميا المنجليه‬
‫؟ التست هل فيه احتماليه ان البناء يصابون‬
‫ يسأل عن النسبه‬٢ . ‫اخترت ايوه ااذ الم فحصت و طلعت حامله هل‬

869- Threatened abortion

870- I think preg. In 9 w. Bleednig with tissue passeg at home , now c os. Close ?

-w. 29 In Preg. 871 ‫ وبالفحص طلعت‬w. 24


‫ وال ببي ي ماعنده نبض و‬os c .
Dilated 4cm
‫وش العالج‬
‫ اخترت ؟‬oxytocin

872- Preg. Mono mono 1st with breech present. And 2nd with cephalic present. ?

preg. in bacteria ? ‫اخترت اني اعدل الول بيدي و تكمل والدة طبيعيه‬

of Screening 873- w. 12

874- RTA +ve FAST ?

Exploratoty laprotomy

heparin ‫ استخدمت‬-875 ‫ م اتذكر السؤال بس كانت عاملة تكميم و عندها سكر و جاها‬PE
‫اتوقع ما نفعها وكان ال‬
ptt in normal range?
‫اتذكر من الخيارات نزود الهيبارين لحد ما يصير ال‬
ptt above the normal range or switch to enoxaparin

876- Air under diaphragm and perforated dudenum what is the management?

877- Pain in 1st 3 days of period ?


Primary dysmenorrhea

878- Women with rh iso immunization, precious child had hydrops fetalis, currently
39w gestation baby has anemia what to do?

A. Amenocentasis
B. Vascular fetal transfusion
C. Maternal plasmapheresis
D. Immediate delivery

they are poor prognosis the survival rate 20% in one year

879- Pt w/ severe MS and DM, she has multiple hospitalisation because of her MS.
She recently developed Kidney failure. She decided to take DNR ( she was alter). Her
labs: potassium 6
What to do?

- Don’t do dialysis ( it revert DNA)


- Do dialysis and ignore DNR
- Give kayxzine and discuss with fam?
⁃ ‫خيار تاني بس برضو ماكان ماشي‬

880- Baby, started weaning 1 week ago. She was given fruit and juices. Had
jaundice and hepatisplenomegaly. Labs: reducing agent + in urine.

- herediatry fructase deficiency


- Galactosema ( there was no catarct )
- Antityrpsin deficancy

881- Typical scenario for borderline personality disorder (suicidal/ selfharm,


unstable mood, fear of abandonment)

882- Child took her relative meds, what to give her ( nothing extra in Qs)

- activated charol
- Gastric lavage

883- Child, look ill, fever, abdominal pain, vomiting, suprapukbuc tenderness
( lab typical for UTI), what to do?
- iv abx
- Oral Abx
- Give abx after culture results
- Unrelated thing

884- 3 months, staccato cough, post cough vomit, bilateral conjunctivitis, no


diarhea, high eosinophils. Lung crackles on exma

- pertusis
- Adenovarous pharyngitis
- Chymadia trachomaos pneumonia
- Mycoplasma pnemonia

885- Child with recent contact with his sick brother, has sore throat,
Conjunctivitis, tonsil enlarge with exudate.

- adenovirus
- Mononucleosis

886- Child with sore thorat, fever, and erythemia upon exam of the throat. Rapid Test for
strep pyogen was negative. Culture sent. What probably will be the management the
pt?

- supportive
- Steroid
- Iv abx
- Oral abx

887- Pt fall and didn’t broke anything.


What preventive measure u can take to prevent *falls*

- lighit in stair
- Give Ca to prevent fracture if fall
- Restrict water to reduce trip to bathroom
- Give antipsychotic ( i think)

———
Month repeat:
- Look alike
- vaginal discharge ( candida, thacomtaos),
- precautions airborn, contact,
- post appendectomy carcinod tumor in tip of the appendi, next? F/U
- DM, Htn, ulcer on tip of toes, next? Stop smoking and lifestyle modifications
———

888- Pt w/ symp of osteo, what to do to Dx ?

Bilateral standing knee x ray.

889- How to screen for nephropathy


‫جا سوالين‬

890- Child w/ IDDM: urine micro

albumin 891- Old with DM,


‫هنا كان في‬
Urine microalbumin and cr/bun ratio.

892- question for fibroadenoma (senatio and u choose dx)

893- pt dx with cystosarcomatus pylloid tumor - how to tx?

Simple mastecmy, or WLE

894- 13 year old, menarche 6 month ago, has irregular heavy bleeding, Dx:

A. Abnormal uterine bleed


B. Ovulatory dysfunction
C. Hypothyroidism? Not sure

maybe B?

895- mutiple cyst in breat, tender before each cycle, one is larger, what is the diagnosis.

fibroadenoma
phylliod cystic
fibroary
896- firm, mobile, mass , slowly growing over type.

fibroadenoma

897- how to tx cystosarcomatus phylloid tumor.

898- Company want to study subgroup of people for prevalence of Alzheimer's disease
, what method used by the IRB to check the ethical point

Utilitarianism
Skeptic ethics
Delogatory ‫ كذا زي شي او‬Forgot last
one

899- Child with he’s mother for routine visit knows the colors and how to
articulate words how old is he

2
3
4
5
6

900- Screening time for iron deficient anaemia

12 month

901- 17 y/O Case of DKA what is the initial management

IV fluid

902- RTA patient came with deformed swollen leg. there is pain and paresthesia
and absent Pulse. Most appropriate management:

A. fasciotomy
B. reduction of fracture

903- RTA patient has bleeding from external auditory meatus. Where is the site of
the fracture?

A. Base of skull
B. Occipital fracture
C. Mandibular fracture

‫ أسئلة مشابهه صار عل هي ا نقاش كثير‬:

Q: Patient presented with bleeding from his ear

A- mastoid bone
B- basal skull fracture

Similar:

Q: RTA patient has bleeding from external auditory meatus. Where is the site of the
fracture?

A. Base of skull
B. Occipital fracture
C. Mandibular fracture

Similar recall:
1- https://t.me/c/1752626805/4323
2- https://t.me/c/1632086392/882
3- https://t.me/smlepaediatrics/40
4- https://t.me/c/1625198149/225
5- https://t.me/silent_2023m/2062
6- https://t.me/BdranSpace/180
7- https://t.me/smle22olds/3609
8- https://t.me/c/1222499606/226
9- https://t.me/QuickRecall/488

10- https://t.me/SMLE_Surgery/588
‫وسط الرابط ع دّ ة رسايل م تاليه تابعه لنفس السؤال وقناة حلوة لجراحة‬

‫صار عليه كتير نقاش‬


B ‫ تكدم غشاء الطبل ونز ساىل شوكي من الذن‬Mastoid ‫نزف من الذن وتمزق غشاء الطبل‬

asal

Similar: 26 Apr
Q: Patient presented with bleeding from his ear ->

A- mastoid bone
B- basal skull fracture
904- Patient removed spleen some other organs was put on parental feeding central
and developed pyrexia 39.4 12hours after previous surgery later labs were insignificant
Diagnosis

1.pneumonia 2.central
line infection 3.ARDS
4.DVT

905- picture of ( TOF ) Tetralogy of Fallot ‫فالوت رباعية‬


4 diagnostic signs on CXR imaging:

1- Right ventricular outflow tract obstruction (RVOTO) due to pulmonary


infundibular stenosis; Pulmonary stenosis may be valvular and/or infundibular with
possible hypoplasia of the central pulmonary vessels.

2- Right ventricular hypertrophy

(RVH). 3- Ventricular septal defect

(VSD).

4- Overriding aorta (the aorta is displaced above the VSD).

Other cardiac defects associated with TOF:


Atrial septal defect (ASD)
PDA
Anomalous coronary arteries.
906- Question about ectopic cervical pregnancy on of the choices

stable ectopic pregnancy

907- In ER the specialist shout bp the resident said 90/50 said pulse said 90 ?

Call back

908- about chlamydia pneumonia gave history of eye pain

909- most common perforation after ERCP

910- child with cataract + hepatospelnomegaly asked about diagnostic test

911- pt after labchole complaining of mouth corner pain

912- imaging : ecg hid

913- child with tension headache what to give ?

Nothing
paracetamol
life style modifications

914- child with regurgitation of food with acid ?

give anti acid

915- mass increasing in size no skin or muscle involvement

916- case picture of endometriosis she had dyschezia + dysmenorrhea but no changes
with bleeding just diagnosis

917- Case about severe AUB, vitals unstable next step


mefenamic acid
d&c
wait

918- Woman goes to infertility clinic, how to assess tubal patency

(hysteroscopy, hysterosonography, laparoscopy and dye test)

919- What worsens mitral stenosis in pregnancy or something like that

increased stroke volume


increased platelets

920- History of PE best contraceptive

IUD

921- picture of CTG late decelerations, mother is in active labor on oxytocin


6cm dilatation, ROM 20 hours ago, what to do next

CS
stop oxytocin

922- 32 weeks PPROM 4 weeks ago, now in labor first thing to do

high vaginal swab for GBS


oxytocin
indomethacin

923- Which tocolytic causes PDA

indomethacin
nifidipine

924- milestone Knows colors, can tell story, age in months

48 months

925- Follows doctors fingers with eyes, smiles and coos


3 months, there was no option for 2 months

Other peds:

926- Child came with bilateral redness of eyes with watery discharge normal
saline drops(my answer sounded like allergic conjunctivitis), oral ABx, IV ABx,
ophthalmology review)

927- Screening time for IDA

12 months

928- picture of TTN xray, fluid in fissures asked for diagnosis

929- Question about pertussis in unvaccinated child (I think it’s from the May recalls)
——
(Also there were questions about measles>diagnosis, croup>which virus,
bronchiolitis>management)
——

930- Trauma to jugular foramen what will be lost

mastication muscles
tongue sensation
ipsilateral vocal cord paralysis

931- Pt. Post cholesytectomy 7 days with RUQ and 7mm CBD dilation

retained CBD stone


slipped cystic duct clip
subhepatic collection
932- Father diagnosed with prostrate cancer, son comes to check DRE found
smooth no enlargement can feel medial sulcus

reassure
psa in 2 weeks
annual dre

933- RA with severe symptoms not responding to methotrexate


and hydroxychloroquine, what to add

NSAIDS
Adalimumab
paracetamol

934- Case of SBO next step

NGT
surgery
CT

935- Pt. Taking medication for cholesterol developed redness? Was reversed
with aspirin, what was the medication

niacin

936- Pt. had big fall now has back pain, urinary incontinence next step

MRI spine
steroids
xray

937- 15 yo needs LP

consent from pt.


consent from parents,
consent from patient and assent from parents

938- Researchers want to make a new study to see relation of smoking exposure with
asthma, they will let a tv host call patients with asthma and ask if they were exposed to
cigarette smoke as children and let them talk about their experience. What makes this a
covert study
Patients don’t know they’re involved in a study, I don’t remember the other options but this is what I
chose

939- Women with preterm labor going for emergency c section, what should be
done before anesthesia

discuss family planning methods


check if blood results are ready I
don’t remember other options

940- question about in Saudi they want to reduce the prevalence of dengue fever,
which region should be targeted first

A. central
B. north
C. south
D. east

comments:

-If there was no west in the options, then I think south is the closest answer

-Center

941-

‫ علي بالحارث كالمه صح‬.‫هذا من قناة د‬

recall: Similar ‫ محمد الحربي‬.‫من قناة د‬


https://t.me/IMCrashSMLE/1596
24 y/o male presented with diarrhea and fatigue what electrolytes abnormality would you
suspect

A- hypokalemia ⬛✓
B- hypocalcemia
C- hyponatremia
D- hypomagnesemia

‫ ؟ ايش رايكم‬:paralysis periodic Hypokalemic Answer


‫ب الجابه‬

Naif: My answer may be wrong I’m not sure.


May be a question has an ECG findings.
I think that ECG findings will help us to determine which of electrolytes are abnormal?
Hypocalcemea ‫اجابتي هي‬
‫ واجابتي‬: ‫السؤال هذا أعتقد فيه خدعة‬
Hypocalcemea
+
Hyper( Phosphorus, Uric acid, and potassium ) ‫هو‬
‫التشخيص‬
*Tumor lysis syndrome cause ( Chemotherapy-induced diarrhea ) *

‫ علي كل كالمهم صح‬.‫من ملف عبدالرحمن القويز نفس اجابة د‬

‫ ياسر قي ول‬.‫ د‬:


I agree with you , diarrhea is one of Symptom of tumor lysis syn due to hyperkalemia .

lysis tumor ‫هو قال بالكيس انه اخذ كيمو وا كل يمو س بت هل‬
diarrhoea ‫ومن اعراضها ارتفاع بالبوتاسيم وارتفاع البوتاسيم هو اللي سبب هل‬
abnormally ، ‫والسوال كان ايش برضو يكون معاهم‬
HypoCa ‫ف اتوقع الجواب واضح بيكون‬
Hyperkalemia

Hypokalemia

—————
—————

942- Old man with HTN, S 4 sound, paroxymal nocural dyspnea, concentric
hypertrophy on ech.
What is the dx?

- diastolic dysfunction

943- Old pt w/ exertional dyspnea, systolic murmur in right 2 intercostal


space. Dx?

AS

944- Infant w. Poor feeding irritability since 2 wk, pansystolic murmur on left
sternal what is dx?

- vsd ( this one)


- pda
Q: again another senario discribing vsd, ask about management:

- dieurtic (think this)


- digoxin

945- Ankylosiny spindolytis senari, ask dx

946- Sle senario, ask dx

947- Senario of cholangitis ask next step ( no investigation done before) there was
US, and ercp, mrcp.

948- 40 yo. Nulliparous she complated breast carcinoma treatment. She planning to
get pregnant.
For your as silent doctor what are recommend time to conceive?

A. 3 months
B. 9 Months
C. 2 years
D. 5 years

949- What are the diagnostic value for Hepatocellular Carcinoma?

A. LFT
B. Alpha-fetoprotein
C. CT chest
D.!!

950- Man working in industry and exposed Radioactive liquid for all
body As silent doctor what are the 1st line treatment?

A. Remove clothes
B. Shower with warm water
C. Shower with lots soap
D. Stay away from party location A

or D
951- 27 years old male admitted due to chest pain radiate from left arm to the neck.
He is diabetic and heavy smoker. There is concern of venous thromboemboilsm.
What is the most appropriate management?

A- Aspirin
B- Encourage ambulation
C- Mechanical prophylaxis
D- Unfractionated heparin A

or D

952- ECG picture (wasn’t clear for me) with history of: patient came with
intermittent chest pain for 2 days and sweating and …..
Normal vital signs.
What is the most likely the cause?

A- Right bundle branch block


B- Left bundle branch block
C- Posterior myocardial infarction

953- 65-years old with comorbidites came with knee stiffening, increase with rest and
at night. Mild improvement with nsaids. Normal joint stability. Normal labs. What is the
most appropriate investigation?

A- Standing plain x-ray


B- MRI of knee
Remaining options are imaging also.

954- A new study conducted to investigate the effect of HIV on mortality. The target
group were recruited from hospitals and followed for 10 years to compare mortality
between negative and positive HIV patients.
What does ethics impose on this research?

A- A considerable profit
B- Frequenct appearances on social media
C- To have in its center the rights of the target group D- To
have in its center the achievement of hypothesis

955- 56-years-old female post-menopause and nullipara came with intermitting vaginal
bleeding. Vaginal examination was normal but there was bright scanty bleeding
cervix. What is the most appropriate next step?
A- Endometrial sampling
B- Pelvic ultrasound

956- What would result from the doctor not teaching a 6 years old patient how to use
space inhalers properly ?

A) increased self efficiency


B) increased adverse effects

957- Patient wit infertility for 3 years and is complaining from dymenorrhea
and dysareunia what is the most appropriate step ?

A) Hystosalpingogram B)
laparoscopy with dye

958- Patient with back pain bileteral knee and ankle pain what is the most appropriate
investigation ?

A) HLAB27
B) MRI sacroiliac joint (or lumbar)

959- Pregnant at 34 weeks with PPROM what will you give ? ( this simple and no
added details )

A) corticosteroids
B) Tocolytic
C) Antibiotic ( I think

960- Patient with PPH after delivering a 3000g baby. She delivered the baby by
spontaneous vaginal delivery after 7 hours what it the cause?

A) Grandparity
B) Macrosomia
C) Preciptous labor
D) Prolonged labor

961- What is the correct way to write an order of vitamin D ?

A) Vit D 400 IU OD
B) Vitamin D 400 unit OD
C) Vitamin D 400 units daily
D) Vitamin D four hundred units (
seriously came like this)

962- Case about a vetrenarian with fever back pain and other symptoms asking
about diagnosis?

A) Brucellosis
Don't recall other choices

963- What is the operation done for an obese with hiatal hernia ?

Roux en Y

964- Patient with cirrhosis due to autoimmune hepatitis and hepatic encephalopathy
( confusion not stated in the stem of the question ). What to avoid in diet ?

A) fat
B) proteins
C) Carbohydrates

B
In contrast, in cirrhosis patients we have to give food that is high in protein and complex
carbohydrates. And avoid low in sodium and fat and simple sugar.

Patient had hepatic encephalopathy so that protein diet should be avoid..

it may be necessary to limit protein intake in order to reduce ammonia. However, it is important to
maintain adequate nutrition, so protein intake should not be restricted too severely.

Another opinion :-
Protein should not be restricted in patients with cirrhosis even if they have hepatic
encephalopathy
Salt restriction is a mainstay of treatment in cirrhotic patients if they develop ascites (might be
forgotten in this recall)
Similar recalls :-

Cirrhosis liver (Dx given) what to do?


- water restriction
- salt restrictions
- protein restrictions
- high glycemic food restrictions

- First step in controlling ascites in cirrhotic pt:


A-Salt restriction
B-Salt and water restrictions

965- What is the treatment for DKA in emergency department ?

Answer: 2L Normal saline with 0.1/kg insulin

966- Question about acute limb ischemia with pain, altered sensation and motor skills
asking what is next ?

A) Heparin

967- Question about acute limb ischemia with pain, altered sensation and motor skills
and mentioned that there is atrial fibrilliation. What is the most appropriate ?

A) Femoral embolectomy
B) Catheter directed thrombolysis

_
Two questions about transient tachypnea of newborn scenario of a full term delivered
by C/S.
_

968- Copd exacerbation case and he asked about the management?

A. High flow oxygen


B. Albuterol
C. Aminopyhlline
D. Salbutamol and ipratropium
969- question about in Saudi they want to reduce the prevalence of dengue fever,
which region should be targeted first

A. central
B. north
C. south
D. east

South ‫جاني دب ال من‬


west ‫كتب‬

970- Patient came with right-sided pleural effusion and a positive history of coughs and
hemoptysis. There was no shift of the trachea nor the cardiac apex. What is the
diagnosis? (No mention of AFB, india or anything hinting towards TB)

A. Rupture of the esophagus


B. Heart failure
C. Tuberculosis
D. Cancer obstructing the ipsilateral bronchus

971- 3 months old boy came to clinic for vaccine has sizures and his bother told you
his immunity test is bending ( they did not mentioned why they are doing the immunty
vaccine !! ) just like this the Q

A) Don't give hil vaccine


B) Give all vaccine
C) Delay the Dtap vaccine
D) Delay all live vaccine

972- Old man came with sings and symptoms of endocarditis with murmur and
asking what is the most common organism?

A) Staph, epidermis
B) klebsiella pneumoniae
C) streptococcus species
D) S. Pnumonia

Naif:
‫السؤال هذا فيه مطب وهو الزم نحدد ال‬
Infective Endocarditis ‫هو‬
‫ هل‬acute or subacute
‫ هنا التفاصيل‬J⬛

IE may be acute (developing over hours or days) or subacute (progressive


over weeks to months). Acute bacterial endocarditis is usually caused by
Staphylococcus aureus and leads to rapid destruction of endocardial tissue, while subacute bacterial
endocarditis is most commonly caused by viridans streptococci and generally affects individuals with
preexisting damage to the heart valves, structural heart defects, or prosthetic valves.

Endocarditis ‫هذا الجدول يخليك تحدد نوع ال‬


Infective

‫ ال حسب على‬clinical
features
+
Onset of symptoms

‫وعلى حسب او ركب صمام دب يل او ال الجدول يحل المشكله‬


‫ نور‬.‫ملخص من د‬

‫ ن ور‬. ‫د‬

Chat GPT ⬛J

The most common organism that causes endocarditis is Streptococcus species, so option C is correct.

While Staphylococcus epidermidis is a common cause of endocarditis, it tends to occur more


frequently in patients who have prosthetic valves or other implanted devices.

Klebsiella pneumoniae is not a common cause of endocarditis, as it is typically associated with


respiratory and urinary tract infections.

S. pneumoniae can cause endocarditis but it is less common compared to Streptococcus species.

Therefore, when considering the most common organism that causes endocarditis,
Streptococcus species are the most likely culprit.

You might also like